Vous êtes sur la page 1sur 64

Table of Contents

Basic Principles & Rightful Position


Reliance & Expectancy Damages
Conseq. Damages & Contract Limits
Mitigation & Collateral Sources
Eco Harm Rule & Certainty
Personal Injury
Constitutional Harm
Taxes, Time & Value of Money
Punitive Damages
Preventive Injunctions
Structural Injunctions
Modifying Injunctions
Irreplaceable Losses
Balancing the Hardships
Preliminary or Permanent Relief
Declaratory Judgments
Quiet Title & Reformation
Restitution: The Innocent
Restitution: Conscious Wrongdoers
Restitution & Contract
Replevin & Ejectment
Enforcing the Judgment: Contempt
Collecting Money Judgments
Attorneys Fees
Equitable Defenses I
Equitable Defenses II

page 2
page 3
page 5
page 6
page 7
page 8
page 8
page 9
page 11
page 11
page 13
page 14
page 14
page 16
page 16
page 18
page 19
page 20
page 20
page 20
page 25
page 26
page 27
page 29
TBA
TBA

MODERN REMEDIES OUTLINE BLUMENFELD FALL 10

Overview of Remedies:
1.
2.
3.
4.
5.

Compensatory Damages
Punitive Damages
Preventive Remedies
Restitutionary Remedies
Ancillary Remedies

I. Basic Principles And Rightful Position (Compensatory Damages)


A. Basic principles/theory
a. Purposes and goals of the law what it is trying to accomplish (individual justice, corrective justice,
economic incentives, etc.?)
B. Restoring P to his rightful position
a. The stated goal of compensatory damages: the fundamental principle of damages is to restore the injured
party, as nearly as possible, to the position he would have been in had it not been for the wrong of the
other party
b. Issues:
i. Precise calculation of compensatory damages
ii. The compensatory standard itself
U.S. v. Hatahley
Facts:
Agents of government came in against orders, took horses owned by P-Native Americans
and sold them to glue factory
Procedural history:
Dispute between district court and appellate court over how we should evaluate the
damages
Analysis
Issue 1: precise calculation of compensatory damages
Appellate court: says DC pulled numbers out of the air; wants individualized
determination of damages
Federal Tort Claims Act: no jury
- Raises questions: if a jury came back with a lump sum, it would be much
more difficult to scrutinize the award
Why does it matter?
- Without precision, awards might be a windfall to some Ps while it would
stop short of restoring others
- Lends legitimacy to the system, even if it is indulging a fiction; it will get us
further in most cases than if we give up at the outset
Compensatory standard itself
Rule: the plaintiffs were entitled to the market value, or replacement cost, of their
horses and burros as of the time of the taking, plus the value of the animals during
the interim between the taking and the time they could have actually replaced the
animals
Loss of Horses = FMV or Replacement, unless Unique (which Indians claimed)
Loss of Use = Rental
Consequential Damages if Proximate Cause, Reasonable Certainty, Unable to
Replace
P&S generally no pain and suffering absent physical harm
A. Competing Economic Principles

1.

2.

Rightful Position Principle Restore the injured party with money as nearly as possible back to the position he was
in before the defendant caused the harm. Three positions 1) position plaintiff would have been in if the contract was
never made (compensating only reliance interest), 2) position plaintiff would have occupied if the contract had been
performed (compensating expectancy interest), or 3) restore defendant to the position defendant occupied before the
contract (compensating plaintiffs restitution interest). (focus on compensation and corrective justice)
Law and Economics Model Law should structure incentives to maximize profitable activity because that is how
you maximize utility. Even if there is harmful/wrongful activity, as long as those who violate the law pay damages,
that is profitable.

B. Loss or Destruction of Personal Property (Valuing Plaintiffs Rightful Position)


a.
b.
c.
d.
e.
f.

Destruction - Plaintiff must prove FMV of the property just before the harm occurred
Loss of Use Plaintiff must prove reasonable cost to rent similar property for the amount of time reasonably
necessary to repair or replace the property
Fundamental principle- P entitled to be made whole, but D is usually entitled to have P made whole in the least
expensive way.
i. Options: cost of repair v. value of item, whichever is less (lesser of 2)
Compensation is measured by the fair market value of the property at the time of the taking, unless it is too
difficult to determine or if it would result in manifest injustice.
Usually compensation is measured by diminution of market value, but there is a special purpose property
exception. With special properties, there is not generally an active market to determine diminution of market
value, so damages are measured in reproduction/restoration cost of the property minus depreciation.
Court says the rule is to value property at the time of loss. Risk of speculation that prices will increase is not
chargeable to the defendant.

C. In Re Sept 11th- LESSER OF TWO RULE


a. 99 yr lease FMV lease = $2.8billion 99 yr stream of income
b. Rule: Lesser of 2, Except special property
i.
Lesser of 2: Diminution in Value or Replacement
ii.
At time of Tort
iii.
Except Special Property: No Market Price
1. Ex: Churches, Hospitals, Non-profits
2. Replacement = proper measure
c. Analysis: Lesser of 2 $2.8 billion (paid) vs $16.2 billion
d. Special Property:
i.
Test = Unique Use No FMV for property?
ii.
Measure Value As of Date of Tort/Loss
iii.
Marketable on 9/11
e. No Loss of Use Would be DOUBLE RECOVERY
D. To recover damages for harm to personal property, must prove the reduction in the property's value or the reasonable
cost of repairing it, whichever is less. If there is evidence of both, is entitled to the lesser of the two amounts.
E. To determine the reduction in value, you must determine the FMV of the property before the harm occurred and then
subtract the FMV of the property immediately after the harm occurred.
F. To recover damages for the unique value, must prove all the following:
1.Property had some market value;
2.Property had unique value to ; and
3. had notice of unique value before the harm [or 's conduct was intentional& wrongful]
- Peculiar value refers to a property's unique economic value, not its sentimental or emotional value.
Trinity Church v. John Hancock Mutual Life Insurance Co. (Mass. 1987)
Facts:
Church was a national historic landmark, which was structurally damaged during the
construction of neighboring property owner's building. The damage to church would have
required disassembly and reconstruction. Before church filed its action, neighboring
property owner's representative agreed to waive assertion of a statute of limitations defense.
Church filed its action three days before neighboring property owner's final waiver expired.
Procedural history:
Plaintiff church filed an action against defendants, neighboring property owner and others,
for excavation-related damages. The Superior Court (Massachusetts) denied neighboring
property owner's motion for a directed verdict but directed verdicts for the other defendants.
The superior court also entered judgment for church against neighboring property owner.
3

Analysis

Disposition

Church and neighboring property owner appealed.


Church was entitled to be compensated for the reasonable costs of restoring church to
the condition it was in prior to the excavation, as church's method of damage
assessment, based upon a percentage of reconstruction cost, was consistent with the
depreciated-cost-of-reconstruction standard applicable to special purpose property
cases
Issue of present value:
Should court reduce award of future damages to their present value? No the fact
that repairs will not be done until some time in the future does not compel the
conclusion that Trinity is seeking recovery for future damages
Damage has been done presently
An injured party is not required to perform repairs in order to recover for
diminution in market value of its property
Affirmed.

-The casebook says that what were talking about is replacing the building in 150 years. Thats not proportional to 40% of the
replacement value today, its too uncertain.
-Were not talking about replacement value today. John Hancock makes argument that what were talking about is replacing the
present value of future damages, court rejects that.
-The way that the plaintiff quanitifed damage of the value was that today 40% of the useful life of the building will be taken
away. Not to pay for future damage but current damage that I will not have to deal with till the future.
-Weve identified a damage, that a building has lost part of the useful life of the building.If we were talking about diminuition
in market value this would be simple illustration: decline in useful life lowers market value.
-*challenge here is that you have the speculation about replacement in the future* the dissent goes wrong that in the situation
that when plaintiff gets substitutionary remedies they can do whatever they want. For instance, if church fell down they dont
have to rebuild it back with the damage money they receive.
-the dissent is complaining that we dont know what were going to do with the money, but that doesnt matter.
-Usually what you get is repair value. This would be impossible with this church because youd have to take the whole thing
apart to repair it.
-Lesser-of-Two rule in repair cases- diminuition in market value vs. repair cost
-How do we determine value? Value of thing lost or destroyed. Most of the time diminuition in market value, repair, and
replacement value are usually the same. In this plaintiff has no obligation to defendant other than take compensation and do
whatever she wants.
-What kinds of cases are hard: cases involving depreciation (generally speaking durable assets change value overtime, you get
back value of depreciated asset at that moment in time when the wrong occurred) where the asset is really unique like a Picasso
painting (have problems applying market measure or replacement value), Property has special value in hands of plaintiff (1.
Items highly customized to plaintiff like a prosthesis we treat them like Special Purpose Property so replacement value, 2.
Sentimental value like dog, generally sentimental value is something we dont compensate because its hard to measure 3.
Property that has greater economic value in hands of plaintiff like Larry Silverstein said WTC more valuable in hands than that
of government then we usually allow them to recover because its economic value is measurable. 3. Property has special
geographical value to plaintiff like goodwill in a particular locale then we do try to measure that goodwill and reward damages)
-Stocks and Commodities- fluctuate in value. General rules
1. In Contract, value loss at time of breach
2. In Property, value damage at time of the wrong
3. In Special cases like employment we give damages up until the date of the trial.
Difference Between Casualty and Liability Insurance:
Casualty insurance is insurance against destruction of property like home, car. Casualty is given back to you, doesnt
necessariliy have to be someone elses fault. Casualty insurance will often given you enough money rebuild your
house, which is more generous than the tort standard. In Tort- The person who dinged up your car is only suppose to
restore to were you were before the wrong, not make you better off.
Liability Insurance- insurance for if you hurt someone else and injure them that you have to pay if you are at fault.
G. Lost Profits
a. To recover damages for lost profits, must prove it is reasonably certain she would have earned profits but for 's
conduct.
b. To decide the amount of damages for lost profits, you must determine the gross amount would have received but
for 's conduct and then subtract from that amount the expenses
-When you get money damages, you dont have to restore yourself to the rightful position
4

-Often the case that replacement value and diminution in market value are the same thing.
-We run occasionally in to problems were the replacement value and diminution in market value are really different.

II.

Reliance And Expectancy Damages (Compensatory Damages)

A. Reliance = Tort (corrective justice) Where the plaintiff partially performs his obligations under the contract in reliance of
defendants representations. Plaintiff may recover reimbursement for money spent in preparing to perform the contract and
lost profits
B. Expectancy = Contract (law and economics) Plaintiff may recover what he expected from the contract or lost profits. i.e.
where the contract was not fully performed and plaintiff had expectations that were not met.
C. Sellers Breach Rule = Market Price K Price + Incidental/Consequential Damages
D. Buyers Breach Rule = Non-Acceptance or Repudiation Diff. between Market and K Price + Incidental damages, except if
inadequate (see Neri)
Difference Between Damages We Reward in Property/Tort and Contract Claims
-In tort case or property case, the harm is violation of plainitffs rights or property, so the value of the harm is what was
taken away). In contracts case the wrong in a breach of contract, the wrong is the incomplete or nonperformance of
breaching partys promise.
-Contract Law protects contracting parties expecting interest. Not completely clear why we do so.
E. Neri v. Retail Marine Corp. Disappointed Seller Sale of Goods case, expectation, reliance, and restitution damages;
volume seller , Sale of Goods UCC, Art. 2. Goods are Tangible Things You Can Move.
a.
Facts Neri went to buy a boat from Retail Marine and deposited $400 and then increased the deposition to
$4250 to expedite delivery. Neri then cancelled the order because of illness and he couldnt make the
payments. Retail Marine held the boat, incurring storage and upkeep costs of $674. Retail Marine then sold
the boat for the same price it would have sold the boat to Neri. Neri sues trying to get his deposit back and
Retail Marine () cross clams, seeking reliance damages $674 (for upkeep) and expectation damages for lost
profits ($2579).
b.
If your not volume seller- If you contract to buy boat and defendant doesnt take it your damages, you get the
profit you would have gained. Because if the contract had been performed you wouldnt still have the boat.
c.
Incidental Damages- damages you incurred specifically cause plaintiff breached which you wouldnt have
incurred otherwise.
d.
If you have to end up selling it for less that is your loss. Then your loss in that case is your damages.
e.
What do we do with partial performance by the Buyer- That will be applied to the damages, to reduce the
damages.
f.
UCC 2-718: If the buyer had taken boat and hadnt paid the contract price is value of the boat. Buyer gets
restitution by the amount greater than the liquidated damages or $500. Buyer entitled by amount his amount
exceeds $0. Buyer is entitled to restitution reduced by(1) offset to the extent of liquidated damages and (2)
benefit buyer gets under the contract.
g.
UCC 2-708(1)-does the same thing that common law does- awards difference between market price and
contract price plus incidental damages, plus any avoided costs minus avoided costs(ex. Operation costs of
factory are avoided costs.) , like not actually having to buy the thing at all. Also has part (2): If the measure
of damages provided in subsection (1) is inadequate to put the seller in as good a position as performance
would have done then the measure of damages is the profit (including reasonable overhead) which the seller
would have made from full performance by the buyer, together with any incidental damages provided in this
Article (Section 2-710), due allowance for costs reasonably incurred and due credit for payments or proceeds
of resale. due costs reasonably incurred includes cost of advertisement. due credit for payments or
proceeds of resale does not mean what it says, the courts interpreted as meaning if you junk the boat and
salvage it then that amount is credited. However, it is not applied as meaning what it says.
h.
Damages in these situations generally if Buyer does not pay(repudiates): difference between contract price
and market price OR contract price and price received on resale(has to be reasonable price on resale) +
incidental damages.
i.
If Buyer takes goods but doesnt pay then damages are contract price.
j.
Difference between contract and market in this case under 2-708 is zero.
k.
Definitions
(1)
Expectation position was in if the contract had been performed and enforced.
(a)
Here, seeks expectation for lost profits from failure to sell two boats.
(2)
Reliance position was in if the contract was never made.
(a)
stored the boat in reliance on Neri buying it and suffered storage and upkeep costs.
(3)
Restitution restores the to the position occupied before the contract. Notice that restitution
focuses on the position of the (not on the position of the )..
5

gave the deposit of $4250 and got no boat, so he wants his money back.
Expectation rule volume seller volume seller can get expectation damages on future profits (Retail
Marine only sold one boat and would have sold two boats if Neri didnt breach the contract, so the
Retail Marine is entitled to the profits from the boat it wasnt able to sell).
m.
Holding
(1)
s entitled to restitution of deposit ($4250).
(2)
is entitled to $3253 on account of its lost profit ($2579) and incidental damages ($674).
n.
Why do we enforce expectation damages?
(1)
Encourage people to enter into contracts
(2)
Theory of economic breach violation of the contract should be encouraged sometimes if the breach is
profitable. If the total benefit to the breach outweighs the benefit to performing the contract, the contract
should be breached.
2. Notes on Reliance
a.
If defendant can prove the would have lost money in the contract, then the restatement states that
the expected loss must be subtracted from any reliance recovery.
b.
A plaintiff in a losing contract can sometimes escape this by suing for restitution of the benefit she has
conferred on defendant.
-The UCC takes some liberties with the common law rule. In the context of Neri we talked about non-breaching sellers
remedies
-UCC does damage calculation somewhat differently for seller breach. 2-718(2) restitution of amount by which payments
exceed liquidated damages, OR, $500
-buyers restituoin offset yb Sellers damages: Contract-resale price (2-706) or Contract-market price(2-708(1)), unless 2808(1) inadequate then get profit and with due credit plus incidental damages(all reasonable expenses incurred because buyer
breached(. Neri is the best example of the case where 2-708(1) is inadequate because he was a volume seller. P
-Case of Seller Breaching: buyers gets to choose difference between market minus contract (2-713(1)) or cover minus contract
(2-712), plus incidentals(ex shipping charges) and consequentials (Hadley)
F. CALCULATING EXPECTANCIES
G. Hadley v. Baxendale- millers broken crank shaft, time before standardized replacement parts for mills, When part of mill
broke you had to take the piece send it to a forge and copy the piece. Hadley pays extra to get the crankshaft to the forge the
next day. Baxendale negligently gets crankshaft there late. Mill is closed entire time the thing is broken.Gets money back
for higher shipping fee. As a result of negligent failure to get it there as fast as promised, mill was closed. Every day Mill
doesnt operate he loses amount in profts, Hadley wants lost profits. Hadley loses case, Baxendale doesnt know all the
consequences of getting stuff late, court says its unfair to place risk on Baxendale when he doesnt know what risk is. They
will allow if the damages are reasonably foreseeable at time the contract is made. Unless anyone would foresee
consequential losses unless its in the mutual contemplation of the parties.Most people dont think thats the law now. Unless
its the sort of damage that due to the nature of the contract people would reasonably expect might flow from it.. It has to be
a risk the parties agree about it so the parites might price the contract correctly.
H. Consequential damages in contract cases are subject to the strict limitation of the Hadley rule. Has to be something that
anybody would have imagined unless you consentually put it into the contract.
I. Chatlos Systems v. National Cash Register Corp Chatlos bought a computer system from National for $46K that didnt
work. Chatlos sued for fair market value of the computer system as warranted ($207k). Court awards difference between
the system they got ($6K) and the system as warranted ($207K). Court says contract price is evidence of fair market
value, but is not determinative. If you dont reasonably rely on representations of a products value, then you will only
receive restitution and not compensatory damages (fake violin for $4K where a real one is worth $100K is not reasonable
reliance).
a. Rule: UCC 2-714: For breach of warranty, the correct measure of damages is the difference btwn the
fair mrkt value of the goods accepted and the value they would have had if they had been warranted.
b. FMV of Warranted Goods FMV of Accepted Goods ($200k as promised in K $6k of junk received)
i. Plaintiff must have reasonably believed they were paying $46k for a $200k computer
c. Dissent says this is a windfall for the Plaintiff (isnt it? does that matter? Punish the defendant for false
advertising, or is this merely corrective justice?)
d. A warranty is a promise that something will be true or not true. Ex. 36,000 miles warranty car company
promises car wont break in specified ways.
i. Sometimes hard to determine a warranty.
e. Court found there was a promise that the computer bought by Chatlos would do something.
f. Most of the action in chatlos rests on the facts of wether NCR gave them a warranty of functionality.
g. The more disparate the difference between what you were promised and what you get, the more suspicions it
raises if there was a warranty.
J. Smith v. Bolles expectation damages limited to CONTRACT cases
(a)

l.

c.

Facts Bolles was buying stock at $1.50 a share ($6000 total). Fair market value of what was promised was
$10 a share and fair market value of what was delivered was $0 a share.
d. Tort case- measure of damages where you were then and were you would have been had the wrong never
occurred.Damages are $1.50, a share.
e. If this had been treated as a contract case- damages would have been difference between $0 (what you got)
and $10(what you were promised)
f. Holding can get back what he paid (restitution) but not the expectation damages.
(1) Why? This was a tort case. It was a fraud case and not a breach of contract/warranty case. Expectation is
expectation of the bargain, but with fraud there is no expectation of the bargain.
(a) *** Fraud does all the to try to get punitive damages.
g. Rule expectation damages are NOT recoverable in tort cases (fraud, intentional infliction of
emotional distress) BUT are recoverable in contract cases.
h. Minority rule exception for fraud
(1) Some states have made exceptions and allow to recover the value of what was promised whether
they sue in fraud or in contract Formosa Plastics Corp. USA v. Presidio Engineers & Contractors,
Inc. TX 1998 pg. 55.
-Hawkins had it been a tort claim, the case would measured the difference between the bad hand he started with and the
worst hand he ended up with. This was a warranty case, the doctor promised a 100% perfect hand and the hand he ended up
with post-op. Hawkins is an example of a case in which the tort case are smaller than the contract damages usually tort
damages are bigger. Warranty involved yielded more generous measure than the tort.
-Minority Rule for misrepresentation case (business cases primarily): In the situation were contract measures exceed tort
damages, which most frequently happens in misrepresentation cases, there is a split of authority. Restatement says at least in
business cases, the measure of damages in representation the damages are difference between what was promised and what
you got.
-Where you have breach of contract that is also a tort you may have an opportunity to argue for whether contract
damages or tort damages are higher and argue for the higher one.

III.

Consequential Damages And Contract Limits

A. General Damages Flow directly from the wrong


B. Specials (ex: Consequential) Plaintiff may recover consequential damages that are PROXIMATE (result from the
defendants breach/wrongful conduct) and FORSEEABLE.
C. Historically suspect; To recover for this harm, the Plaintiff must prove that when the parties made the K, Defendant knew or
should have known of the special circumstances leading to such harm
D. Tort Proximate Cause
Contract Foreseeable Consequence
E. Proximate cause- has to do with something being so far down the chain that we cant hold defendant responsible for those
damages.
F. Consequential damages have been disfavored for (1) difficulty of finding certainty (2) remoteness (proximate cause) (3)
avoidable loss and duty to mitigate damages( injured party has to do things to limit there damages as reasonably as
possible).
G. Buck v. Morrow: Tex. Morrow leased Buck land, then sold it. Buck had damages.
a. The trial court held Buck (P) could recover only lost value of the lease itself, not consequential damages for
Morrows breach of lease. Ct of appeals reversed. Obvious he gets difference in price between agreed lease
and higher lease he had to get (could be argued he spent too much for higher lease, and didnt substitute but
upgraded).
b. whatever special damage naturally and proximately resulted to appellant from the sale of the land and
termination of the leasewhatever may reasonably be supposes to have entered into the contemplation nof
the parties at the time of the contract-he should recover.
c. Rule: A party may recover any consequential damages reasonably anticipated by the parties for the breach
of a lease of real property.
i. Consequential damages included higher rent elsewhere, extra cattle hand, lost cattle (cow births),
profits from cows, transportation costs, pain and suffering (not allowed for cows).
1. Notes on General and Special Damages
a) Consequential damages never awarded in eminent domain cases (50 acres)
b) There is some hostility to consequential damages b/c such damages are more speculative and
likely to have been avoidable had P been more diligent.
2. Notes on the Vocabulary of Damages
a) General damages- what P lost from the initial impact of Ds wrongdoing
7

b) Special/Consequential damages- foreseeable damages that result as a consequence of Ps initial


loss but are unique to the injured party and the situation youre facing in that particular case.
3. Outside of the Personal Injury area- when people are talking about general damages they are talking
about problems that usually arise of this kind.
a) Special damages the kind more remote, more unique to the particular injured party.
b) In the personal injury area, special damages are the kind that can be reduced to a certain sum.
Outside personal injury when they say special damages they mean damages that are less easiliy
determined and unique to the particular party.
c) General damages in the personal injury area are things like pain and suffering, emotional
distress that we leave to the jury to determine.
-So special and general mean the exact opposite of what they commonly mean in personal injury cases.
UCC- sellers damages dont include incidentals but buyers damages do include consequential damages. This is because
buyers get something different almost all the time, different product, sellers get money almost all the time.
H. Meinrath v. Singer co.
a. Meinrath sought consequential damages for Singers failure to make timely bonus payments under a written
agreement
b. Rule: Failure to pay money: A creditor may recover only interest on late payment of money due and no
consequential damages are recoverable.
(1) P contended failure of timely payments of bonuses caused his business to fail. Ct said a
creditor may not recover consequential damages for the debtors failure to timely pay sums
owed.
(2) When wrong is nonpayment of money all you get is money.
(3) Doesnt matter if these damages were foreseeable per Hadley v. Baxendale, esp since these
damages had nothing to do with the K at hand and causation is murky.
d) Amount Owed:
(1) $300,000 Bonus
(2) $770,000 Consequential Damages loss of business from D not paying him
(a)
$648,000 invested capital result of liquidation and forced sale
(b)
$122,000 decline in networth of Unicard Belgique
(c)
Moved for summary judgment. Both agree its lost. Legal question as to
whether recoverable.
c. Court says that consequential damages to Meinraths business are too speculative/remote. When money is
withheld or delayed, plaintiff is entitled only to interest and not consequential damages that stem from
the delay. Majority rule allows consequential damages when they are foreseeable i.e. causation is clear
& amount is certain. Court says that if plaintiff wanted consequential damages, he shouldve contracted for
them in liquidated damages provision.
d. Interest: pre-judgment interest- interest measured from time of the wrong, or demand is made, or suit is
made. :
i. Prejudgment interest is available at different points in time. NEED TO KNOW WHAT YOUR
JURISDICTION SAYS.
ii. Post-judgment interest- is the interest added to judgment after judgment is affirmed.
e. This time value of money interest issue frequently adds up to a lot of money. With low interest rates
defendant has incentive to carry out appeals, since he gets real cheap loan, because of low rates charged on
federal post-judgment interest.
-Exception to Meinrath loan p. 61 - If the the breach of the obligation is commitment to make loan(page 61), and lender has to
go get a loan at a higher rate of interest, then they will get measure of damages that is equal to difference in interest charges.
-Can also put consequential damages into the agreement. Usually party isnt in a position to argue for consequential damages.

Texaco v. Pennzoil (put other brief in here)


- principle of certainty at play here.
-tort involved here was tortious interference with a contract, Pennzoil is suing Texaco for inducing breach of contract.
Wrong was breach of contract that deprived them of 3/7s of the contract.
-Ways to measure contract: differnence btw/ price Pennzoil contracted for and what Texaco bought Getty shares for.
FACTS
8

Pennzoil (defendant) made a public offer to buy a controlling share of Getty Oil Company (Getty) (plaintiff) stock. In the
alternative, Pennzoil entered into an agreement with two primary shareholders of Getty, signing a memorandum of agreement
that was subject to the approval of Gettys board of directors. The memorandum was presented to the board. The board decided
that it would not recommend the public tender to the shareholders and also rejected Pennzoils agreement with the two primary
shareholders. The board made a counter-offer that was rejected by Pennzoil. Getty then began soliciting bids from other
companies. At the next board meeting, with no definite bids by other companies, the board made a second counter-offer to
Pennzoil. Pennzoil accepted and Getty and Pennzoil both drafted and issued press releases. Pennzoils lawyers began drafting
the formal agreement. The Wall Street Journal reported on the Getty-Pennzoil agreement, the Pennzoil board met regarding the
agreement, and Pennzoils lawyers were in contact with Getty regarding the agreement. Meanwhile, Getty continued soliciting
bids from other companies. Texaco, having talked with Getty, held in-house meetings, researched Getty, and hired an
investment banker to represent it in the possible acquisition of Getty. Texacos board voted to make an offer. Texaco met with
the two primary shareholders of Getty who both agreed to sell their shares to Texaco. Getty then held a board meeting and
voted to withdraw its counteroffer to Pennzoil and accept Texacos offer. Texaco issued a press release regarding its agreement
with Getty. Pennzoil contacted Getty and demanded that they honor their agreement. Getty entered into an agreement with
Texaco. Getty filed suit for declaratory judgment that it was not required to honor the Pennzoil contract. The trial court
awarded Pennzoil damages for Texacos tortious interference with the contract. Texaco appealed to the Court of Appeals of
Texas.
RULE OF LAW
A party interfering with a contract can be held liable for causing its breach when they knew of the existence of that contract and
actively caused the breach of it.
ISSUE
Can one interfering with a contract between two parties be held liable for causing the breach of that contract?
HOLDING AND REASONING(WARREN, J.)
Yes. In an action for tortious interference with a contract, a defendant must have known that a contract existed. However, it is
not required to know all the terms of the agreement. Knowledge of the contract exists where the facts surrounding the contract
are known, even if the legal significance of those facts is not known. Whether the requisite knowledge exists is a factual issue
and may be proven through circumstantial evidence. Additionally, in a tortious interference action, a defendant must have
actively caused the breach of the agreement. Simple knowledge of the other contract is not the same as actively causing breach.
There must be an active attempt to interfere or to persuade a party to breach. Regarding damages, a jury verdict cannot be set
aside unless it was made upon passion, prejudice, or other improper motive; it was not a deliberate decision by the jury; or
the award is so excessive that it would be unjust. Additionally, punitive damages may be awarded for the wanton or malicious
commission of an additional tort. New York law does not require an award of punitive damages to be proportionate to the
actual damages. However, Texas law does and the following factors will be considered to determine whether the awarded
punitive damages are reasonable: (1) the nature of the wrong, (2) the character of the conduct involved, (3) the degree of
culpability of the wrongdoer, (4) the situation and sensibilities of the parties concerned, and (5) the extent to which such
conduct offends a public sense of justice and propriety. The goal in awarding punitive damages is to punish the wrongdoer or
deter similar behavior in the future. In the current matter, there was sufficient circumstantial evidence to establish that Texaco
had the requisite knowledge of the agreement between Pennzoil and Getty. Additionally, there was sufficient evidence to
establish that Texaco actively caused the breach of the agreement. The facts show that Texaco implemented a calculated
strategy to cause the breach and was not a mere victim of Gettys solicitation for bids. Regarding Texacos prayer for remittitur,
there is evidence that the jury awarded excessive punitive damages. The award is not disproportionate to the actual damages, as
it only amounts to approximately 40 percent of the actual damages. However, the evidence does not suggest that Texaco
intended to injure Pennzoil or otherwise act in a malicious manner towards Pennzoil. Awarding one billion in punitive damages
is sufficient given Texacos actions. Accordingly, if Pennzoil files with this Court a remittitur of two billion dollars within two
days, the judgment will be reformed, with the compensatory award and a punitive award of one billion affirmed. Otherwise, the
judgment is reversed and remanded to the trial court.
-Lost value of contractual expectation plus expectation damages of subsequent damage is not controversial as the measure of
damages
-Texaco says the loss of the value is very easy to measure.
-Texaco argues the damages are $500 million. The difference between what Texaco paid and Pennzoil bargained for.
-Pennzoil argues that he purpose it bought the stock was for Gettys oil reserves. They can prove that the cost of finding the
reserves themselves is $10.50 a barrel when they could have acquired it for $3/barrel by buying Getty.
-Jury awarded difference between the $3/ barrel Pennzoil bargained for and $10 per barrel it would cost to acquire reserves
itself.
Three Takeaways
1. Enormous disparity- between Pennzoils measure of damages(differenc in cost of getting gettys oil and searching for
iol) and Texacos measure of damages(difference in agreed stock price and what Texaco paid)
9

a.

This large discrepancy raises some question: there are alternative measure of value that seem equally
plausible. Value of stock as reflected by highly sophisticated bidders for stock and assets that the firm owns.
i. There is a significant difference between looking at hthe value of what Pennzoil lost based on what
people paid for it and the value of the oil that the stock is suppose to represent. Two ways of looking
at that:
-1. If you own 3/7s of a companys stock that doesnt mean you own 3/7s of the companys assets. When youre a 3/7s
shareholder in a big complicated company talking about what kind of control you will be able to have. Pennzoil would have
some influence of the partition of the assets of the company. Juries leap from 3/7s of the stock to 3/7s of the oil even though it
seems pretty speculative that Pennzoil would have control of the oil assets. Most of the time a court would say that is too
remote and speculative to assume.
-2. Another wrinkle is the huge difference in value between the stock and the value of the assets. The shares of the company are
worth at least as much as the value of the assets usually. Its not unusual with energy companies for there to be huge disparities.
This seems implausible that Getty people were stupid to sell for such a low price. There wasnt a secret that Getty was selling,
so how Pennzoil could have gotten such a bargain doesnt really add up.
-There was some degree of speculation about how you would turn value of shares into rights and reserves.
I. Limits on the basic principle (of restoring P to his rightful position)
a. Primary tools for restricting recovery of consequential damages are (1) contractual limitations on remedies
and (2) rules about avoidable consequences, scope of liability, and uncertainty.
b. EULAs often contain limitations on remedies. They put those in there because its unclear about what the
harm to the party might be.
i. Mass Marketed products and things with sophisticated buyers like expensive equipment like
engraving equipment
ii. If there is some type of damage that seems worrisome to you put a limitation on remedy in the
contract.
iii. With these adhesion contracts- most of the time you cant negotiate the terms of. Kearney and
Trecker is difference because
J. The Parties Power to Specify the Remedy
e) Kearney & Trecker Corp. v Master Engraving Co, p.74
(1) Master engraving bought a machine tool from Kearney (D) which did not operate
properly (most of the time) and sought consequential damages for lost profit.
(2) The consequential damages from lost profit was expected, but the contract waives
those damages.
(3) These are consequential damages but they are really foreseeable. The measure of
damages they used here: because you breached the contract by delivering nonconforming
products to use we lost sales, these was breach of contract case not breach of warranty like
Chatlos.. Its within the rule of Hadley v. Baxendale. .
(4) Page 68 shows limitation on damages agreement.
(5) No issue about adhesiveness or unconscionability
(6) 2-719 UCC
(a) You can agree to limit a particular remedy, disclaim a particular remedy, and
disclaim aparticular warranty, Unless the sole remedy you provide fails in its
essential purpose.
(b) Consequential damages may be limited or excluded

unless the limitation or exclusion is unconscionable.


(7) Court says this is enforceable even though the repair or replacement sole remedy failed
its essential purpose.
(8) Rule: A consequential damages disclaimer (liquidated damages) is not invalidated
by the failure of a limited remedy provision unless it is unconscionable.
(a) Cts should only invalidate when liquidated damages are inconsistent with
intent and reasonable commercial expectation of the parties.
(b) Some cts say since there is an integral relationship btwn the exclusion and the
limited remedy, the failure of the repair or replacement provision should invalidate
the disclaimer. This and other cts say the exclusion of consequential damages
should be viewed independently, and breach of warranty does not mean that there
is also a breach of liquidated damage clause.
(9) Risk is usually reflected in a reduced purchase price.
(10) Illustrates also tension btwn freedom of K and providing fair remedy for the breach.
(11) Restitution would have been = $167,000 (purchase price)
10

(12) Jury = $57,000 in lost profits


(13) Contract:
(a) Warranty = 12 months or 4,000 hours
(b) No incidental or consequential damages = consequential damages exclusion
(c) Buyers remedy limited to repair/replace/refund = limited remedy of repair
and replacement
(14) Only damages they got was repair or replacement, or a refund. The exclusivity went
away because
(15) Some states say if you have an exclusive remedy and it goes away all the other
exclusive remedies go away with it. Then you just go to ordinary contract remedies
(16) View of Other states was that you treat exclusivity provision separately from
disclaimer of remedies. Disclaimers are unenforceable unless they are unconscionable. This
is the view of the Court here. Consequential damages are irrelevant in the instant case
because the sophisticated parties bargained for them.
f) This Tells us there is this split in the jurisdiction. Since problems come up like this with
contracts that have interstate nature and choice of law issues
g) The limitations and disclaimers that are left are going to give you a sense of whether the
remedy fails of its essential purpose. The whole contract will bear on whether the remedy available
fails of its essential purpose.
h) The other thing worth bearing in mind: understand there be a practical relationship between
limiting the remedy and disclaiming the warranty all together..
i) You can disclaim warranties allot more freely than limiting remedies.
j) If youre a litigator look at the whole package. Sometimes the exclusive remedy will fail for
failure of its essential purpose and the limitations on consequentials damages will still be enforced
or sometimes it wont be enforced. You need to know the law that will be applied.
k) Disclaimer of a remedy is like saying you wont collect consequential damages at all. Subject to
unconscionability standard
l) Exclusivity clause- says if something goes wrong your sole remedy is repair or replacement of
the product (for example). Subject to failure of essential purpose standard.
(1) The failure of essential purpose standard is a lot more forgiving than unconscionability
standard. It says if there is some reemdy that your left with then the exclusivity clause will
probably be okay.
m) Notes on Limitation of Remedy Clauses
(1) 2 ways to limit contractual liability
(a) sell as is instead of including warranties (limit substantive obligation)
(b) liquidated damages clause (limit liability for breach)
(2) Yellow pages disclaim all liability for errors or omissions (no consequential damages)
b/c some errors are inevitable and the potential liability is disproportionate to the size of the
transaction.
K. Rule limitation is invalidated ONLY when the circumstances of the transaction cause the consequential damages
exclusion to be inconsistent with the intent and reasonable commercial expectations of the parties that invalidation of
the exclusionary clause would be appropriate.
a. Policy the rule tries to balance the freedom of contract with the ability to provide a fair remedy for breach
of contract.
L. LIQUIDATED DAMAGES (limits)
M. In Re Trans World Airlines: Liquidated damages clause is reasonable if 1) it is a reasonable estimate of damages by
the parties at the time the contract was made and 2) loss is difficult to prove (where actual damages are easily
calculable, liquidated damages provisions will not be enforced). Liquidated damages clause CANNOT be punitive.
a. Law-and-Economics scholars such as Posner conflict on punitive damages. On the one hand, liquidated
damage provisions are efficient because the parties are the best ones to determine what their damages will be
in the event of a breach. On the other hand, they fear that damages in excess of lost expectancy will deter
efficient breaches.
N. The liquidated damages clause says the planes will be deemed to be worth these amounts and different times during
the life of the lease. They will charge you the difference between the termination value and a number that is either the
present value of the rent they still owe or the resale value
O. TWA says that the actual damage is a lot lower than the actual damages.

-There is a big distinction drawn between liquidated damages and a Penalty( that is something that is suppose to
incentivize you to not breach).
11

UCC 2-718:
-There are two moments you can assess the reasonableness of liquidated damages is when the parties make the
agreement and afterwards when there is a dispute. It might turn out that the actual problem that arises is that the
actual damages that occur are way smaller than what you thought they would be, or they could be a lot larger than
what you anticipated in the liquidated damages clause.
-There was a very elaborate liquidated damages clause that it will pay for violating the lease. Its the termination
value plus at the lessors option. TWA breaches and the liquidated damages are in excess of the actual damage (the
present value of the remaining rental payments.
-If liquidated damages is like a penalty of forfeiture then court will not enforce it. The main uncertainty in these
things is at what time you want to assess the reasonableness of the measure. The UCC is curious whether it is a
reasonable estimate of damages at the outset of the creation of the contract or at the time of the breach.
-NY law says that its easy to figure out what the damages are, then the actual damages are not going to be
enforced even if the liquidated damages might have seen reasonable at the time the contract was made. More and
more courts seem to be saying that if we can figure out what the damages are then even if liquidated damages were
reasonable at the time the contract was made then they wont enforce the liquidated damages clause.
-Another issue of damages disputed was the maintenance deposit. If TWA does an overhaul they get the
maintenance deposit back and if they havent done it then the lessor keeps the lessor deposit.. The deposit is $1.4
million and the overhaul costs $2million, you can look down at see that there will be a very expensive cost of
maintenance and TWA might shortly return plans right before the maintenance overhaul is required then TWA
would not have to pay for the deferred maintenance, the maintenance deposit was a way of allocating the risk of the
cost of the deferred maintenance. The court says because the maintenance deposit requirement reflected a
reasonable estimate, ex ante, of the injury that Interface would incur if TWA terminated the lease early, then the
maintenance deposit was not an unenforceable penalty.
Northern Illinois Gas Co. v. Energy Cooperative. Inc.
-one side is saying the liquidated damages clause
-Northern Illinois says the liquidated damages is the exclusive remedy. It does fail of its essential party.
-Both parties rely on UCC 2-719(b).
-The buyer (the one who breached) says 2-719(b) applies only to things that limit a remedy and liquidated damages
dont limit remedies but liquidate them ( fix them) at a particular amount.
-If you have a liquidated damages clause that underliquidates the damage then it might should be covered by 2-718,
and it does look like a limitation of a remedy and maybe should be governed by 2-719
-Understand the different ways you can interpret 2-718 and 2-719 .

Northern Illinois Gas Co. v. Energy Cooperative Rule:


Absent a contract provision to the contrary, a liquidated damages clause prevents a party in a breach of contract
action from seeking an alternative measure of damages.

IV.
Mitigation And Collateral Sources
Avoidable Consequences
A. Rockingham County v. Luten Bridge Co. (Avoidable Consequences):
a. Contract to build bridge ($18k Expected profits of $8k)
b. Luten starts and incurs $1.8k in costs when County tells him to stop - Luten builds bridge to nowhere anyway
c. Rule: If Defendant breached the contract and the breach caused harm, is not entitled to recover damages
for harm that proves could have avoided with reasonable efforts or expenditures. You should consider the
reasonableness of 's efforts in light of the circumstances facing her at the time, including her ability to make
the efforts or expenditures without undue risk or hardship. If made reasonable efforts to avoid harm, then
your award should include reasonable amounts that she spent for this purpose.
d. Holding court only gave the company damages for the part of the bridge completed BEFORE the county
decided it didnt want the bridge. County was NOT liable for the portion built after notifying that county
didnt want the bridge.
B. SJ Groves & Sons v. Warner Co.(Avoidable Consequences):
12

1.

Warner consistently failed to deliver concrete on time to SJ Groves. As a result, Groves breached their contract at the
construction project. DC awarded damages until July 12, when Groves shouldve hired someone else to avoid further
damages. Appeals decided that the decision not to hire someone else was reasonable under the circumstances, so full
damages should be awarded. Plaintiff chose from 6 reasonable options. Plaintiff gets to decide which reasonable
mitigation choice to use under the circumstances and not the defendant or the court years later.
2. Doctrine of Avoidable Consequences Defendant is not liable for avoidable consequences of his wrongdoing.
i.e. plaintiff may not recover for losses he could have avoided. BUT where both the plaintiff and defendant had
an equal opportunity to reduce damages, the defendant cannot complain that the plaintiff did not mitigate.
3. For a significant period of time Groves was stuck with Warner. IT was a government contract and Warner was
the only government approved vendor that was geographically accessible, there was nobody else they could buy
from. There wasnt anybody Groves could higher more reliably. Trap Rock gets govt certified but Trap Rock
might not have the resources and Trap Rock received some of its material from Warner, and Groves might still
have had a problem with them.
-In cases of contract for goods, if buyer breaches your agreement, then seller still has a duty to avoid consequences, so any
cost of selling the good is an incidental damage you can collect for mitigating your damages and collect those. The reasonable
costs of reasonably mitigating damages are recoverable but you still have to mitigate your damages.
-Warner is overextended themselves which is why they did such a bad job fulfilling its obligation to SJ Groves.
-App. Court adopts Groves argument saying that Avoidable Consequences works by saying that Groves could have
hired Trap Rock on July 12 but they werent sure that Trap Rock would be any more reliable than Warner. Warner said
even though we were short delivering you, you should have used another supplier. Groves said having two suppliers
could cause problems. Trap Rock was relying on Warner for some of its raw materials,, Warner could have cut off Trap
Rock if Groves had switched to Trap Rock. The App. Court said Trap Rock was an option but it was an option that
came with real reasonably and idenfitable risks.
-Court concludes it would have been reasonable to take on Trap Rock but it was also Reasonable not to.
-1. RULE: Faced with the possibility of mitigating damages the aggrieved party has the option of choosing any Reasonable
course of action and the defendant doesnt have the option of second guessing the choice if it turns out in hindsight it wasnt
the best choice. Particularly when the party that created the party is criticizing you for continuing to do business with them.
-2. Groves argued the Warner could have just have easily gone to Trap Rock as Groves could have. Court says you could not
blame plaintiff for not mitigating in a way they could have mitigating themselves. Be careful applying that argument because
its not the rule as a matter of law but is a good supporting fact if the facts do support that argument.
Employment Cases- if you get fired then you have a duty to mitigate damages by getting a new job. You can only recover in
wrongful discharge cases the amount you would have been paid in the time it would have taking you to find a new comparable
job.
Medical Treatments- If the treatments benefits outweight its risks and costs then it was probably reasonable to have done to
mitigate damages. .Difficult to make arguments about what is reasonable in light of religious objections. When someone has an
irrational reasons for wanting to do something, we dont usually talk about religious beliefs of religion as irrational. If you treat
it as a constitutional issue or as a cultural issue,
Didnt have Money to mitigate- sometimes it is the case some plaintiff argues that they didnt mitigate because they
couldnt afford to, sometimes because of the defendants breach. This is a mixed question of law and fact, usuallyl comes down
to the facts and whether what the plaintiff did was reasonable.
Offsetting benefit- if defendant did something to hurt you and it ended actually making you better off the court gives the
defendant credit. If you get fired and you get a new job thats better the difference with what you owe is a credit against your
damages.
Restatement- says your damages get offset by offsetting benefits if it they are for the interest the defendant hurt.
-Look out for offsetting benefits, and then see if they are offsetting benefits that should be allowed to count.
Collateral Sources
B. Oden v. Chemung County:
a. Injured Ironworker seeks damages for work-related injury, and Defendant seeks to diminish damages on the
basis of his Disability Retirement benefits
b. Collateral Source Rule judgment will not be reduced if there is an independent collateral source (such as
insurance policy). If an injured party receives compensation from a wholly independent source
(someone other than wrongdoer), that compensation is NOT deducted from the plaintiffs damages.
c. The plaintiffs argument was that the 141,000 only offsets the ordinary pension payments. The disability
retirement benefits does not offset lost future earning.
13

d.

In a lot of defined benefit schems if you get hurt so that you cant work, you get a disability pension that is
higher than the pension you would get if you were still working. The disability retirement beneifts replace
the the 66,000 of lost ordinary pension beneifts.
i. Collateral source rule evidence of payments made from a wholly independent source should not
come in to reduce damages.
1. It is a rule of damages and a rule of evidence. Since it wouldnt be used to offset the award
it was irrelevant and not admissible.
2. There is a lot of variation with collateral source modifications state to state.
ii. Why do we allow the to get a windfall here?
1. Plaintiff invested in insurance premiums. If you dont get the insurance money, why ever
take out insurance? Better for to get the benefitsrather than the wrongdoers.
2. We dont tell juries all kinds of things (e.g. s attorney works on contingency)
3. Subrogation if the recovers the money from the , the insurance company can go back
and get that money from that .

In collateral source cases, the payments wer e made gratuitously or in fulfillment of a promise made in exchange
for consideration paid earlier, in the offsetting-benefit cases, the payments were made for fresh consideration (after
the fact.
-The statute said here that you have to determine which benefits offsetting the corresponding the damages.
Sometimes which benefits correspond to a specific category of damages is hard to determine.
-There are about as many collateral source rules today as there are states.
-Problem we run into is that there are all kinds of collateral sources. Collateral source is payment to wrong payment
from someone other than wrongdoer.
-ex. Church fund to pay for medical expense is collateral source. Grauitous kinds of payments are subject to
the collateral source rule
-Things get messy when someone comes along and fulfills a benefit they may become subrogated to
recover some of what they get. Ex. When insurer pays you because of insured loss, insurance company steps into
your shoe to sue defendant and get their payment back.
-health insurers typically subrogated to rights of tort victim.
-IF there is a right of subrogation attached to collateral source payment there is a GOOD CHANCE STATE LAW
WILL TREAT THAT AS A COLLATERAL SOURCE. There is no risk of double payment to the victim.
-Where there is a 3rd party that is subrogated that is treated as a collateral source because the tort victim will not be
likely to keep the money.
-Truly Gratuitous stuff- you are usually allowed to keep it because charity is a good thing.
Policy Arguments: we worry about giving the plaintiff a double recovery, we also worry about wrongdoers having
to bear the costs of their wrong. Given between the choice of excess recovery to harmed person and no liability to
wrongdoer we choose excess recovery. Frequently when party is getting paid from 3 rd party source victim paid for
the insurance since they were paying premiums, so we dont wan to penalize 3 rd party from getting what they paid
for, just because the defendants payment would duplicate the insurance payment.
-Additonal concern is that most cases settle for less than the amount of the injury, and the victim
has to pay their attorney. So allowing them a double recovery allows them to be closer to being whole..

V. Economic Harm Rule And Certainty


Reasons we Limit Liability
-no proximate cause
-Notion of when you determine there is a duty
-plaintiffs harm isnt foreseeable
-categories of damages we exclude like pure economic losses.
-limits on emotional distress recoveries
-With these situations we are just figuring out how far the liability goes
A. Economic Loss Rule s cannot recover for indirect economic harm in the absence of physical harm to their persons
or their property. In the case of personal injury or strict products liability, if there is no physical injury or other
property damage, pure economic harm cant be recovered. Intentional torts are not subject to the economic loss rule.
IF the defective product destroys itself that doesnt count as property damage.
14

a.

Contract cases: Limits liability of contracting parties only to claims on the contract. Even if breach of
contract was tortious there is no tort recovery.
b. Physical Impact Requirement If the did not suffer physical impact, she cannot recover, no matter
how foreseeable the harm or how direct the causation.
c. Policy Reason Fear of unlimited liability and because plaintiff can insure against the loss
d. Exceptions: Water pollution (RI), Oil spills (US), professional negligence(since almost always the damage
is economic.)
e. Doesnt matter that the economic harm was foreseeable- still only limited to physical impact.
Pruitt v. Allied Chemical Corp.- See Casenotes book
Pruitt v. Allied Chemical Corp.
1. D spilled chemicals in the water causing many fish to die
Category of
Who They Are
Recovery Allowed
P
A
Commercial Fisherman Yes
B
Seafood Wholesalers
No (indirect
C
Seafood Retailers
N
D
Seafood Processors
N
E
Seafood Distributors
N
F
Seafood Restaurants
N
G
Boat Owners
Yes
H
Bait Shops
Yes
I
Marina Owners
Yes
2. The court thus finds itself with a need to limit liability in some way without any particular reason to exclude
anyone.
3. Economic Loss Rule: Plaintiff suing in negligence if they suffer no physical injury or impact or any property damage
then you cannot recover economic damages or lost profit
4. Law and economics: encourage efficient conduct
a. It is all about incentives
b. The whole idea of the law is to encourage the right incentives
c. If liquidated damages provision for breaching contract it would discourage people from breaching the
contract when it would be efficient for them to do so
d. Get people to do what we want them to do in order to maximize utility
5. Judge has rejected some of the plaintiffs at the pleading stage prior to discovery
6. The judge isnt really correct saying the defendant would have to pay multiple damages of the same kind. The actual
damages would be loss profits. Isnt any duplication.
a. Concern about duplicative compensation and making the wrongdoer pay more than once for the same thing.
7. We want the defendant to pay all the costs that are legitimately attributable to the accident in a way that the defendant
will internalize those costs. Only those costs that impose costs on society will be undertaken that can bear those costs.
a. If you cutoff liability for foreseeable damages that gives perverse incentives to pollute so this decision can
really be supported on law and economics grounds.
8. The judge is saying that at some point the liability will be so severe that the defendant will go out of business. The
only reason he decides the liability would be too much is just based on feelings.
9. When you encounter a situation like this- where something happened quickly- something that doesnt happen unless
someone was negligent- if youre the plaintiff you need to go out and get the facts necessary to show it was
predictable, foreseeable, that there was a duty, there was prox cause, etc.
10. Ordinarily in an environmental damage case there is damage to physical property or personal injury- The fish in the
waterway in this case were nobodys property. There was physical damage but it didnt belong to anyone yet. Under
the economic loss rule the fisherman shouldnt recover anything because its not actually their property.
B. In Re Kinsman Transit (highlighting the physical impact requirement)
a. Accident on the Freeway similarity: it was clear that mooring a boat improperly could lead to the risk of a
boat drifting away and crashing into another boat, and that both boats could crash into a bridge, which
collapsed and blocked the river, and in turn, the wreckage could flood the land adjacent to the river, as well
as prevent any traffic from traversing the river until it had been cleared. But under proximate cause, the
property owners adjacent to the river could sue (Kinsman I), but not the owners of the boats or the cargoes
which could not move until the river was reopened. The boats that lost their cargo as a result of the
accident could not sue, but the boat that was hit (physical impact) could sue.
C. Defendants Knowledge
15

(2) Evra Corp. v. Swiss Bank Corp. must be on notice of special damages
(a) Facts H-M contracted to charter the Pandora. It is a losing contract for Pandora because the rate
is locked in at a low rate and this is a two year term contract. H-M uses Swiss Bank Corp. to make
payments every month. In April 1973, H-M authorizes a payment by Swiss Bank, but Swiss Bank
loses the paperwork. Pandora cancels. This is a lawsuit against the bank.
(i) Trial court s damages are $16,000 for arbitration and $2 million in consequential damages
from the loss of the use of the Pandora.
(b) Holding not able to recover consequential damages.
(i) Why? Because the damages were not foreseeable. It is not reasonable to assume that being late
on a payment will result in $2 million in consequential damages.
(c) Rule consequential damages will not be awarded unless the defendant was put on notice of
the special damages; the kind of general foreseeability which is present in virtually every case,
does not justify an award of consequential damages.
(i) Argument that Hadley v. baxendale didnt apply here because this wasnt a contract claim (no K
between Swiss Bank and plaintiff) its a negligence case. Extent of damages in negligence is
based on foreseeability .
(ii) Posner says even though it isnt a contract case, it makes sense to apply the Hadley rule and
limit liability.
(d) This is really more of an avoidable consequence case. Hes not really mitigating damages by
checking with the bank.
(e) Really the reason is that EVRA knows a lot more about their situation than Swiss Bank does.
(f) It makes commercial sense to limit liability here, but there isnt really an existing doctrine to limit
liability in this case, but it is kind of like a contract even though its a tort case.
(g) Other ways to categorize this case: negligence case with only economic harm, case involving only
failure to pay money (no consequential damages other than interest),
(h) See starred paragraph page 115. Put this in Notes
D. Recovery of consequential damages in contract Hadley v. Baxendale 1854 pg. 72 Facts contracts case about the
pump within the windmill breaking.
i. Rule plaintiff is entitled to recover for those consequences of breach that were within the
contemplation of the parties when they made the contract
E. Reasonable Certainty

a.

-This most often comes up when we are talking about economic damages. Non-economic
damages: pain and suffering, economic loss, loss and enjoyment of life.
-requirement of reasonable certainty: most often comes up in connection with lost profits. Bigelow is great
example of the challenges inherent in this kind of exercise.
-The problem that arises often is having to imagine a counterfactual scenario that is not like the world was.
Preventitive injunctions are relatively rare. Have to imagine a world in which the wrong never happened and
figure out what we need to give the plaintiff in a position of getting herself there.
-More complicated are situations where there are lost opportunities or lost profits.
Bigelow v. RKO Radio Pictures- put in brief from Casenotes
i. In business litigation you are often presented with the problem of figuring out the rightful position.
Sometimes its hard to imagine the but-for world that would exist but for the actions of the
defendants.
ii. Antitrust conspiracy case.
1. If you have a time advantage at your movie theater you will have more customers to your
theater than somebody else.
2. The movie studios gave themselves and their theaters a timing advantage
3. Studios agreed to give their own theatres timing advantage and each others theaters timing
advantage. Independent theater owners get screwed.
iii. Two Measures of Damages- Two Common Ways Lost Profits are Measured.
1. Yardstick measure- looked at ps theatre and compared it to performance of defendants
comparable theatre during the time of the conspiracy to restrain trade
2. Before-After Measure- look at ps theatre performance during 5 years before the 5-year
conspiracy to restrain trade and compare to how it performed during the 5-year conspiracy
period. The difference is the wrong.
16

3.

Rule: this Court has sustained recovery of the full amount of defendantss profits where his
own wrongful action has made it impossible for the plaintiff to show in what proportions
he and the defendant have contributed to the profits.
iv. Problem commonly run into is to figure out what the world would be like in the absence of the law.
v. Frankfurters dissent point said that what if in absence of antitrust conspiracy the movie studios
would still legally benefit the theaters they owned, so It is the case that plaintiff might not have
suffered harm from the conspiracy or negligible damages.
vi. Other confounding factors: conditions of the economy. Example is the Brinks case note 4 page 130.
This is the bread and butter of a business litigator. In business cases there is frequently an element of
lost profit or lost economic opportunity.
b. Lost Income
i. Three Measures
1. Yardstick Method
2. Before-After Method
a. If youre going to use a before and after method- you have to have enough time
before the harm to actually come up with a reasonably reliable estimate of what
the harm to the business was.
3. Comparing going concern value of the business at the moment before the wrong and the
value after. Way to approximate value of lost profits. All evidence and calculations must be
based on facts known at the time of the wrong; actual experience after the wrong is
irrelevant.
-In a case where lost income is an issue of the case, pay attention to the problem early on and hired experts early on and have
them figure out the kind of information you need to acquire in discovery to litigate or defend the case.
Silicon Valley- news story on Sakai. Hard to use the before and after measure on this case. Similarly there was a national
market how do you figure out where there was a competitive market if anywhere. Hard to prove what damages are. Easy to
get stuck with a response from the judge that you havent proved anything with sufficient certainty.
c.

Washington v. ACS Corp.


i. Plaintiff tries to seek damages on the basis that his career as a future wrestler has been ruined by the
Defendant, and he puts forth information about how much a wrestling career would have earned
him.
ii. Rule: No remedy if the damages are too speculative. To recover damages for past lost earnings,
must prove the amount of [income] that she has lost to date.
i. To recover damages for future lost earnings, must prove the amount of [income] she will be
reasonably certain to lose in the future as a result of the injury.
ii. Holding: Court thinks these damages are too speculative for plaintiff to recover

VI. Personal Injury Damages where Value cannot be Measured in Dollars


A. Debus v. Grand Union stores
a. P was injured when boxes from store fell on her, leaving her 20% disabled. Jury awarded $346,276. D
appealed, contending allowing P to make a per diem (per day) damage argument for Ps pain and suffering is
overly prejudicial. Ct disagrees with D and affirms. Ct notes that D can refute any absurd hypothesis offered
by Ps atty.
b. Rule there is nothing inherently improper or prejudicial about per diem arguments if they are made
under the ordinary supervision and control of the trial court.
i. Policy there are sufficient checks and safeguards in the adversarial system to overcome the
objections to use of per diem arguments.
c. Notes on Pain and Suffering: states forbid per diem or unit-of-time arguments and some states only allow
them with a cautionary instruction
d. PerDiem/ Unit of Time argument- arguing that her damages per day are a certain amount and then summing
up the damages per day for all the days remaining in their life expectancy. 1/3 of states say per diem
argument okay, 1/3 say not okay, 1/3 say okay but with limiting instruction.
i. Argument on the plaintiffs side is that since its okay in this jurisdiction for attorney to give a lump
sum figure there is no problem dividing it up on a per diem basis
ii. The defendants argument is that there is a cognitive bias that causes people to not be aware of how
small amounts add up with the per diem argument.
17

iii. Per Diem argument assumes that every day is like every other day in terms of suffering. Plaintiffs
say per diem argument should be allowed since defendant is allowed to argue against it on basis,
such as this one.
*If we applied the same standard to personal injury as we do economic damages, we probably wouldnt allow damages for Pain
and Suffering.
-Since we do compensate for personal injury there is enormous variation of awards given.
B. Personal Injuries
a. Types of damages
(1) Economic damages
(a) Earnings e.g. Victim was making X money per year, and would have lived Y amount of years. Get
that number and then double (for things like raises and bonuses and inflation)
(2) Non-economic damages
(a) Pain and suffering
(b) How to evaluate the loss of a family member loss of love, loss of comfort, loss of society, loss of
solace, and loss of moral support
C. Notes on Valuing Pain and Suffering and Human life
a. 2 models of damages for pain and suffering (in light of the fact that it is impossible to FULLY compensate
for death or serious inj)
i. Corrective justice model- P should receive full value of his injuries, so that he is placed, as nearly as
may be, in the position he would have occupied if he had never been injured
1. Would award partial measure of loss to survivors in death cases, and award reason. Compensation for
pain and suffering
ii. Economic model- P should receive full value of this injuries so that D will internalize the costs and
have the optimal incentive to avoid injuring others.
iii. Both yield the same conclusiondamages should equal the full value of harm to the P.
D. Intentional Infliction of Emotion Distress
a. Limits Tests: Physical Impact, Zone of Danger, Foreseeability, Physical Manifestation
b. Pain + Suffering - CACI 3905A: To recover for future pain and suffering, must prove that she is
reasonably certain to suffer that harm. No fixed standard exists for deciding the amount of these damages.
You must use your judgment to decide a reasonable amount based on the evidence and your common sense.
For future pain and suffering, determine the amount in current dollars paid at the time of judgment that will
compensate for future pain and suffering. This amount should not be further reduced to present cash value.
Three types of Personal Injury Damage Arguments
1. Awarding the appropriate amount. This is what most standard jury instructions say,
objective measure
2. Voluntary Transaction argument- asks jurors to award amount they would ask for if they
were being paid to suffer plaintiffs injuries. Not allowed in most states either.
3. Golden Rule argument- not allowed in most every state. Asks jurors to put themselves in
victims shoes and award amount they would want to be compensated for injuries
Present Value- unusual to instruct that verdicts for pain and suffering be reduced to present value.
Wrongful Death (notes on page 141) find notes from other outlines to put here
1. Most states have survival acts. They allow survivors to bring claims that the decedent
would have. These are not wrongful death claims
2. Wrongful Death Claims- every state has a statute says who in relation to the dead person
has standing to bring wrongful death claims. Typically close family. Wrongful death claims
are not from common law but from statutory law. If your engaged to be married but not
married you typically dont have standing to bring wrongful death claim. Some places
include brothers and sisters other do not.
a. Not bringing claims because of what happened to decedent but what happened to
them.
b. By common law and by statute- the amounts you can recover for decedents death
you can recover for.
3. Wrongful death damages are economic and non-economic damages- funeral expenses are
examples of economic losses. Funeral expenss is peculiar because it is the one every state
says we should be able to recover. Thnking about it in terms of the rightful position, there
is going to be a funeral at some point. If you were actually measuring the economic harm
related to fueneral expenses, then it is a present value issue because the decedents family
18

4.

5.

6.

7.

would have to pay funeral expenses any way at some point when the decedent died from
other than tortious causes. BUT we allow all funeral expenses to be recovered anyway.
Lost Financial Support- the amount of lost financial support depends on who the
investment banker is. We figure out the present value of the support that would have been
provided to the family. The measure is what you would have gotten had the support
supplier had lived.
a. The notion of reasonable certainty that we require in business disputes- is not as
strict in wrongful death cases, since its must less sure what the decedent would
have earned during their lifetime. This is not based on how much they earned in
total but how much they would give to the family. There is a lot of room for
argument here, very little of that argument will be cut off on the grounds of being
too speculative
Services that Can No Longer Be Provided by Decedent- includes household chores. Or
income to hire people to do those types of services. Free to make arguments about what the
person was and the type of chores they would do or would hire to do household chores.
Services in many states amounts to not just household chores but parenting or peer services
of the kind that include nurturing, training, education, guidance. The kinds of things that
members of a family often provide to one another. Quite a lot of the loss service stuff goes
into the intangible nurturing, training, education, guidance area.
Loss of Society- a type of non-economic loss. Companionship, love, care. Like loss of
consortium.
a. Almost no place allows the wrongful death claimant to recover for emotional
distress or grief at the loss of the decedent even though it seems like such an
obvious result of losing a loved one. Only two or three states allow recovery for
those emotional distress or grief.
i. Reasons for that is that juries might be overly swayed by emotional
arguments
Loss of Support vs. Loss of Inheritance
a. Loss of Support- the loss of stuff that would have been used to support the family
while the decedent lived
b. Loss of Inheritance- based on how much money the decedent would have made,
how much would have been spent on support of the family, how much would be
left over, who it would have been left to, and how it would have been invested.
i. States that dont allow lost inheritance do it on the grounds the
assumptions are too speculative.
ii. Most places allow the defense to argue the bad as well as allowing the
plaintiff to argue the good.
1. For example, defendant could argue decedent was horrible and
his survivors are better off now that hes gone.
a. Plaintiffs can argue that even if person was horrible
they could have become a better person

-Typically where you have very brief period of pain and emotional distress. It is typically way overvalued on a unit of time
basis compared to long-term chronic pain and suffering.

Emotional Distress see notes p. 178: To recover in trot for either economic loss or emotional distress,s
plaintiff must show something more than negligence: intentional tort, negligence plus physical impact, or
negligence plus one of these other options for emotional distress(note 1 page 178) very limited cases
where very deliberate bad faith breach of contract will be treated like a tort rather than breach of contract.
Breaches of contracts typically doesnt include emotional distress damages, but torts will often allow
emotional distress damages. Example breach of insurance contracts will typically allow of emotional
distress damage claims, even though its a contract claim.
-Typically you will need something more than negligence to recover for emotional distress.
I. You need intentional infliction of emotional distress
19

II. If negligence results in physical injury. You can typically recover emotional distress not just from
the physical injury but from the whole experience
a. Even if there is not physical injury some states say it is enough if there is physical impact
even if you dont get hurt.
III. Negligence plus apprehension that you would be injured even though you werent
IV. Negligence plus observation of a loved one being injured
a. Some states also require that you be in the zone of danger were negligence occurred but
you didnt observe your loved one being injured.
V. False news of a death allows recovery for emotional distress
VI. Mishandling of corpse
VII.
Leaving disgusting foreign matter in a food product
VIII.
Some states say that if your emotional distress is so severe that you have physical
manifestation, they will allow recovery for emotional distress
IX. Generally, you need really serious emotional distress with proof to recover. Courts will look at
others observations of your behavior, that you went to a therapist or doctor.
-In any situation where you can show physical injury even a minor one, your still entitled to be
compensated, but not so with pure emotional distress. This is because we dont really trust emotional
distress. There is no state that doesnt have a negligence-plus standard to recover for emotional distress.
The reason we dont is because it is hard to know whats going on in someones head that doesnt have a
corresponding physical aspect. Generally in tort law, we take the plaintiff as we find them. For example,
in Levka,, the court said there not going to agree with the plaintiff that she is thin-skineed, we dont see
courts in physical injury contexts saying they wont agree with the plaintiffs assertion that they are
thinned-skin.
-very hard to measure baseline of what typical emotional distress is, harder than to measure the baseline
harm from a broken arm. Theres the risk of exaggeration.
A. Constitutional Harm
X. Typically Police Violence
XI.Section 1983: any person who is subjected to the deprivation of any rights, privileges, or immunities shall be liable for suit
in equity, and other redress.
a. Person = Individual vs. Government
b. Redress: Legal & Equitable
XII.
Bivens v. Six Unknown Named Agents (Illegal Search)
a. Damages claim against federal officers available?
i. Implied remedy for damages exists
ii. No special circumstances counseling hesitation
b. The Court, in an opinion by Justice Brennan, laid down a rule that it will imply a private right of action for
monetary damages where no other federal remedy is provided for the vindication of a Constitutional right,
based on the principle that for every wrong, there is a remedy. The Court reasoned based upon a
presumption that where there is a violation of a right, the plaintiff can recover whatever he could recover
under any civil action, unless Congress has expressly curtailed that right of recovery, or there exist some
"special factor counseling hesitation."
XIII.
Carey v. Piphus (kid suspended from school and claims Constitutional harm Ct says: prove harm)
a. May plaintiff recover damages without showing injury resulting from deprivation?
b. Held: A plaintiff must prove actual injury by a deprivation of due process in order to recover compensatory
damages as in any other tort case. Neither the likelihood of such injury nor the difficulty of proving it is so
great as to justify awarding compensatory damages without proof that such injury actually occurred. The
plaintiff must convince the trier of fact that he actually suffered distress because of the denial of procedural
due process itself. Nominal damages otherwise. No presumed damages.
i. Denial of procedural due process should be actionable for nominal damages without proof of actual
injury since the right to procedural due process is absolute. Court held that even if suspensions were
20

c.
d.
e.

f.
g.

h.
i.

XIV.

justified, respondents nevertheless will be entitled to recover nominal damages not to exceed one
dollar from petitioners.
ii. We dont know if Piphus actually did anything wrong. What we do not is that the principal
suspended him without proper process.
iii. Piphus position is that his procedural due process rights are violated.
Piphus argues that the violation of his due process rights entitle him to money because constitutional rights
are value in an of themselves without proof of actual harm
We recover damages not for a breach of duty but for the measurable injury that flows from a breach of duty.
Courts said constitutional rights are no different than any other duties we impose in our society. The court
said there is no inherent monetary value in losing a procedural due process right. Here the constitutional
violation has to cause some kind of harm, the abstract deprivation of the right without more may be a
problem it might be something we want to enjoin, but without compensable harm there is no money to
award.
Piphus counters the Courts response to his first argument by saying there is a presumption of harm if a
constitutional right is violated. Court says thats wrong, you still have to prove harm
Piphus then makes other argument that it is very upsetting and he suffered emotional distres to be deprived
constitutional rights and that should be a compensable harm. Court says he did not prove that he suffered
emotional distress. Court says that if on remand Piphus can show emotional distress flowing from the
deprivation of his due process rights. Piphus also argues that the injury is being kicked out of school. Court
says that it has not been proven that the due process violation was the cause of him being kicked out of
school since it doesnt show that he wouldnt have been suspended had a proper hearing taken place.
IN THESE CONSTITTUIONAL HARMS YOU NEED TO SHOW: 1. That constitutional right was violated.
2. That the violation caused compensable harm (hard to show when right deprived is procedural right).
Takeaway: all the same rules still apply, the fact that its the constitution doesnt matter. The same standard of
compensability of procedural rights violations also apply to substantive rights.
i. We still dont know whether there are substantive rights of which the violation is a tort, there is a
question of whether you still have to show harm (such as emotional distress).

Levka v. City of Chicago (Unwarranted Strip Search: excessive judgment?)


a. Plaintiffs damage claim was for emotional injuries
b. Facts: $50k jury verdict, shocks the conscience?
c. Defendants admitted she was negatively affected but they contested the severity of her distress
d. Rule: In reviewing a jury verdict, the court will defer to the jurys judgment unless the award is monstrously
excessive or shocks the judicial conscience
i. One factor is whether or not the award is out of line as compared with the awards from similar cases
e. Reasoning: the award in this case was inconsistent with prior awards in similar cases. In cases awarding over
$30k, the Court found that there were usually aggravating circumstances, and none existed in this case.
i. The court believed the jury was awarding punitive rather than compensatory damages.
f. NOTE: the case is about an illegal strip search. The illegal strip search was violation of constitutional rights.
g. SHE DID NOT RECOVER FOR THE VIOLATION OF THE CONSTITUTIONAL RIGHTS. She recovered
for the emotional distress that was a result of her violation of her constitutional rights. This is like typical tort
cases, where people are only compensated for harm caused to them not the fact that defendant breached a
harm.
i. This issue comes up in a lot of dignatory harms:
1. assault(in the non physical contact sense),
2. false imprisonment,
3. malicious prosecution,
4. intentional infliction of emotional distress,
5. Defamation
6. Invasion of Privacy
7. Batteries (offensive but non injurious touching, groping)
h. Levka poses the question of what were compensating in these cases where intangible constitutional rights
are being violated?
i. The Other thing the court says is that they will figure out if the damages are excessive by looking at other
verdicts and comparing them
i. This is actually a rare way of determining case. Only a handful of jurisdictions did this.
ii. This is kind of a silly way of doing things because all cases are different.
iii. Other problem with this is if you use an old verdict as the basis for determining damages, then
inflation might make that verdict must less valuable in todays money
21

j.
XV.
XVI.

iv. Mores and values change as well. Like women used to be expected to deal with a lot of abuse in the
workplace but now a lot of that behavior is considered outrageous.
v. These type of comparable verdicts damage measures can be used in arguments by lawyers but are
almost never determinative in judging whether an award was excessive.
Generally excessive damages are evaluated after a post-trial motion by a trial judge and then can be appealed
if the judge upholds them.

Zarcone v. Perry (Judge orders a coffee vendor brought before him in handcuffs because of poor quality of coffee)
a. Affirmed a jury award against a judge. Punitive damages not so high so as to shock conscience $60k
for the punitive.
Rawson v. Sears (employee is fired) Reviewing a Jury Award
a. Facts: employee is awards multi-million dollar verdict in a wrongful discharge (P&S, Punitive, Comp.)
b. Absent an award so excessive or inadequate as to shock the judicial conscience and to raise an irresistible
inference that passion, prejudice, corruption or other improper cause invaded the trial, the jury's
determination of the fact is considered inviolate
c. While bias, prejudice or passion can be inferred from an excessive damage award, a verdict will not be set
aside unless it is plainly excessive so as to suggest that it was the product of such passion or prejudice on the
part of the jury
i. My judicial conscience is not shocked though my acquired cynicism has received a rather sharp
blow.
d. Assessment of damages is the exclusive province of the jury, and it is only in the clearest cases that its award
will be overturned on review
e. It is assumed that twelve men know more of the common affairs of life than does one man: that they can
draw wiser and safer conclusions from admitted facts thus occurring than can a single judge.

Capacity?
Damages?
Punis?
State Actor
Individual
1983
Yes
City
Official*
1983
No!
State
Official*
No
No
****
Fed Actor
Individual
Bivens
Yes
(U.S.) Fed Agency
Official*
No
No
If Defendant Sued in Official Capacity Treat As If Suit against Defendants Employer
Going back to Levka- how to value the damages. We have a causation issue. And we need to know under what
circumstances recovery for emotional distress is allowed. Once we say that the emotional distress is recoverable, then what
flows from it is recoverable. IF you geniunley needed medcial assistance because you were severelly emotional distress you
can recover the cost to go to doctor, if health insurance paid for that they will have subrogation rights. Plaintiff isnt going
to be very much better off because they had to pay the doctor out of pocket or the insurance company will be in line of the
plaintiff to get their money back.Usually what we are talking about with emotional distress is the emotional equivalent of
physical pain and suffering. If the emotional distress is recoverable then they can recover for lost earnings. The med.
Expenses are recoverable, lost work is recoverable, In any of these situations we need to ask the question: What is the
plaintiff complaining about? How can we disengage what the plaintiff has suffered versus what the defendant is alleged to
have done? Is the emotional distress recoverable?
-

B. Preventative Injunctions (prevent the wrongful act from occurring)(reparative = prevent consequences)
A. Injunction = Specific Relief (equitable remedy)
a. Most common kind of specific relief. Most legal remedies involve money, and it is true that most specific
relief is injunctive. This only because of historical circumstances. There are exceptions to the general nature
of the remedies, for instance, Replevin is a specific remedy at law
b. Court orders directing the defendant to do or refrain from doing something enforceable by sanctions or
contempt.
i. More injunctions are negative than positive.
c. Seeks to maintain the plaintiff in the rightful position by preventing harm before it happens i.e. maintain the
status quo.
d. Damages compensate for things that have been caused(substitionary remedies), injunctions prevent things
from happening in the first place
22

e.

Elements for Relief:


i. Likely Irreparable Harm
1. Propensity (likelihood) Requirement
2. Ripeness
3. Not Moot
4. Fact Question - Credibility
ii. Damages Remedy Inadequate
iii. Balance of Hardships Favors Plaintiff
iv. Public Interest
B. Economic analysis of injunctions this is the answer to rightful position principle
1. Law and economics approach profitable violations of the law, trespasses, and breaches of
contract are OKAY as long as the violator compensates the victim.
2. Pure economic theory approach to injunction injunctions force people to negotiate; when
injunctions are approved, the parties are forced into negotiating to get around it.
3. Problem with pure economic theory breaks down
a. Transaction costs are too high too many parties, too many s to get everyone
together (e.g. pollution), so you cannot get all the parties together to negotiate.
b. Bilateral monopoly monopoly because the parties can only deal with each
other; there is no other place to gothese two people are stuck (e.g. neighbor
trying to steal the others timber).
ii. Contempt- violation of a court order is a crime. See note 5 starting on page 266 for discussion of
this: 3 main typesmight want to paste in definitions from internet for these terms)
1. Criminal Contempt- violating an injunction is contempt of court. Contempt is a criminal
offense if done willfully/. Plaintiff may bring the offense to the attention of the court, and
the court decides whether to ask the prosecutor to prosecute the contempt. Defendant is
entitled to most of the protections of criminal procedure
2. Compensatory civil contempt- if defendant violates injunction, plaintiff may also cite
defendant for civil contempt, civil contempt is a remedial proceeding, and plaintiff
prosecutes it himself. In compensatory civil contempt, the court grants compensation for
any harm plaintiff suffered as a result of defendants violation of the injunction.
Compensation in contempt generally includes attorneys fees and generally is awarded
w/out jury trial.
3. Coercive civil contempt- most important. The court imposes conditional penalties to
coerce defendant into obedience. These penalties are civil, but they may include fines or
imprisonment or both. There is no fixed limit to the penalties that can be imposed in
coercive civil contempt, but the penalties must be conditional. Defendant must be able to
avoid them. The court often specifies what steps defendant must take to purge himself of
contempt and thereby avoid coercive penalties. The effect is to make the original
injunction more specific, and sometimes more demanding.
C. Almurbati v. Bush (Ripeness of Injury Requirement)RIPENESS CASE- SAYS INJUNCTION FACTORS SHOULD BE
BALANCED
a. 6 Bahraini nationals classified as enemy combatants
b. Fear: released to hostile foreign country
c. Injunction sought: No transfer without 30 days notice
i. Facts: Media Reports and Unidentified US officials
d. To obtain injunctive relief, petitioners must show that the threatened injury is not merely remote and
speculative.
e. In determining whether to grant a motion for a preliminary injunction, a court must consider four factors: (1)
whether the petitioners have demonstrated that there is a substantial likelihood that they will prevail on the
merits of their claims; (2) whether the petitioners have shown that they would be irreparably harmed if
injunctive relief is not awarded; (3) whether the issuance of injunctive relief would not substantially harm the
other parties; and (4) whether awarding the relief is in the public interest. These factors should be balanced
against one another and if the arguments for one factor are particularly strong, an injunction may issue even
if the arguments in other areas are rather weak. Thus, injunctive relief may be warranted where there is a
particularly strong likelihood of success on the merits even if there is a relatively slight showing of
irreparable injury. However, a party seeking injunctive relief must demonstrate at least some injury since the
basis for injunctive relief in the federal courts is always irreparable harm.
4. RULE on Ripeness: Before an injunction is issued, there must be a ripe threat of
injury. Thus, the threatened harm must be imminent or even immediate.
23

f.
g.

Ripeness injunction sought must prevent imminent irreparable harm


Rule an injunction should be issued only to prevent imminent irreparable injury. Injunctions will not
be issued just to allay fears and apprehensions or soothe anxieties of parties.
h. If your counsel on either side of an issue over an injunction. One of the most common ways of proving that a
defendant is likely to do a bad thing is to show that a defendant has already done it.
i. Does the Plaintiff have to show propensity or likelihood by the fact the defendant already did it?
No. They can show a threat by the defendant to do the wrong.
ii. Sources of Ripeness:
1. Article 3(I) of the constitution: There is a constitution requirement of ripeness in order for
federal courts to have the jurisdiction to do anything, and there was a requirement from
english common law that a equity will not adjudicate unless there is a very good reason.
i. Burk disagress with this decision by the judge. The judge was much too trusting of what the Governments
assertion that its policy is not to send people to countries where they might be tortuted. Burk thinks judge
also screwed up by discounting the value of the evidence offered by the plaintiffs with the NY times citing
their jailors saying theyll be sent to Saudi Arabia and Yemen. The kind of credibility your suppose to give to
testimony from defendants about what will happen in the future was not given in this case.
j. The issue in this case (and with Ripeness in general): is how likely is the bad thing to happen? It is a
likelihood that often is accompanied by temporal immediacy but the immediacy is really required.
D. Ripeness - Uncertain consequences (Ripeness required for a Nuisance = Reasonable Certainty?) Similar to Almurbati
a. Nicholson v. Connecticut Half-Way House, Inc. Conn. 1966 pg. 252
i. Facts half-way house being built in a nice residential neighborhood.
Neighborhood seeks an injunction to stop its building because of concern
about increased crime and decreased property values. Nicholson is
concerned that there will be unreasonable harms that flow from the
lawful harm.
ii. Rule fears and apprehensions based on speculation do NOT justify
an injunction.
iii. Application we dont know what these criminals can or will do and so
an injunction is inappropriate. At this point, there s an insufficient factual
showing that will make any unreasonable use of its property.
iv. Factors to consider in determining Reasonable Certainty:
1. Nature of Activity
2. History of Defendants similar conduct
3. Importance of Proposed use?
v. This case involved a prophylactic injunction. The difference between a
prophylactic and preventive injunction, is that preventive injunction are
orders to refrain from specific unlawful acts. Prophylatic acts are orders
to refrain from acts that are legal.
vi. The court is engaged in the same crystal balling that any other court
would do before issuing an injunction. Its making a prediction about
what the likely affects of this lawful conduct are.
vii. What you are enjoining is legal with prophylactic injunctions: that is a
step further from ordinary preventive injunction.
viii. If the court concludes that this isnt ripe (the apprehension of imminent
harm is not great enough to justify the relief) that doesnt mean it is over.
The only thing that has been determined that at that moment in time there
is no basis to grant injunction. Facts could change in the future.Unlike
other kinds of disputes that are retrospective in nature, this is a moving
target that can be relitgated again and again.
b. Ripeness is part constitutional issue and part discretionary issue. Before court will
instruct someone to do something there has to be a reasonable apprehension of
imminent harm.
c. Why dont we consider everything enjoined: court orders individuate courts
command. Implicitly the court is saying they are more worried about you doing
the bad thing than they are every one directly.
d. What Ripeness is Not:
i. It is not enough to show that the plaintiff fears to irreparable. /There has
to be enough risk that the defendant is going to do something that the
status quo will be disturbed.
24

E. Mootness Voluntary Cessation Court says party seeking an injunction of a recurring violation has a heavy
burden of showing that there will be recurring violations.
a. Rule must prove that the case/injunction is not MOOT; e.g. has to show defendants propensity
i. Burden of proof has the burden of proving propensity, but the defendant has the burden of
proving mootness and voluntary cessation is generally not enough
ii. United States v. W.T. Grant Co. - voluntary cessation is a factor
iii. Facts Hancock (working for Layman Bros) had a deal that when they took over a company, they
would put one of their people on the board. This is a violation of the Clayton Act because these are
ALL department stores and he is on the boar at each store.
1. Government seeks injunction (a) getting him off the boards and (b) preventing him from
doing this in the future.
a. Hancock resigns from the boards and then argues no harm imminent because he
isnt on the board and no imminent danger in the future.
i. Lower court agrees.
iv. This is antitrust case under sec. 8 of Clayton act, to prevent interlocking directorates that restrict
competition. This case is not just about interlocking directorates but interlocking directorates that
restrict competition.
v. Rule necessary determination is that there exists some cognizable danger of recurrent
violation, something more than the mere possibility which serves to keep the case alive.
1. 3 factors to consider
a. bona fides of expressed intent to comply;
i. is this just an attempt to avoid the injunction or is it evidence of an
expressed intent to comply with the law
b. the effectiveness of the discontinuance;
i. How effective was the action taken? Can it be undone easily.
c. The character of past violations.
i. The more egregious the violation, the more likely that voluntary
cessation doesnt prevent an injunction.
vi. Bottom line defendants voluntary cessation may defeat an injunction IF that action proves
the defendant now lacks the wherewithal, knowledge, and propensity to cause imminent harm.
1. This is really a credibility issue.
vii. Like ripeness mootness has two sources: art. 3, section which says there has to be a case or
controversy for federal jurisdiction, mootness is also a matter of equity in determining whether a
court will issue an injunction.
viii. Court critical questions court asks:
1. Has this case become so moot that it flunks the case or controversy test and we dont have
the authority to issue an injunction? The court says voluntary cessation of an alleged wrong
does not moot the controversy. There is a controversy here because the DOJ is not
convinced there not going to do it again
ix. Dissent says there is no indication that Lehman brothers would refrain from getting its own people
on the boards of directors its customers, and on that basis he would issue an injunction. This is a
propensity inquiry, the majority looks at the proclivity of Hancock, Douglas looks at the propensity
of Lehman brothers to get its people on boards of directors. Hard to enjoin Lehman in this case since
they are not a party
x.

F.

Scope of the injunction THE SCOPE OF THE PAST VIOLATION DETERMINES THE SCOPE OF THE
REMEDY AGAINST FUTURE VIOLATIONS.
a. Marshall v. Goodyear Tire & Rubber Co. - Scope of injunctions based on past violations
i. Facts Secretary of Labor suing Goodyear under ADEA for lost wages and damages. District court
granted a nationwide injunction, which is challenged for being overbroad given that this was only
one incident in the store with regard to one employees dischargeReed.
1. Qui tam where individual goes to the government and ask the
government to bring the case.
ii. Rule scope of past violations determines the scope of the remedy
against future violations.
25

iii.

iii. Application here, the injunction was probably not appropriate (the case
was remanded for that determination) because this was only one guy in
one store and doesnt seem to justify a nationwide injunction.
1. Actions cannot support a nationwide injunction (We are not
going to give a nationwide injuction (broad scope of injuction)
for the bad action of a single store (narrow scope for infraction))
a. No finding of company or policy practice
b. Encouraging more local oversight is not sufficient
c. Scope of Relief must be measured to the past violations
iv. Compensatory injunction notice part of this injunction is compensatory
because the is seeking reinstatement and back pay.
e. Ripeness is being used here to limit the scope of injunctive relief.
f. Saying that they should be enjoined from age discrimination in all of their stores is
beyond the scope of evidence.
g. Why are we concerned about overly broad injunctions: we want to tailor the
injunction to the wrong. Also, here there was a real practical issue, that goodyear
cant control how everybody acts in its organization. Even if goodyear didnt
everything reasonable to prevent age discrimination, if someone screwed up then
goodyear would still be held in contempt of court. Nationwide injunction puts this
company at a risk that is not fair.
h. When Federal Courts sit as courts of equity, they need to tailor the injunction to
the wrong that is reasonably apprehended.
i. Where there has a been determination of facial invalidity of a government law or
policy: the prospective relief can reach more broadly than the plaintiff, and it
could take the form of saying the rule does not apply to anybody or that they must
withdraw the rule.
i. A similar set of concerns has to do with defining the scope of wrong, in
terms of defining what the wrong is and what the defendant should be
kept from doing.
j. Obey the Law injunctions are not okay, this is because the defendant has to be
given a clear idea of what is required of him
k. F. R. Civ. Pro. 65(d)(!) requires every injunctive order to state the reason why it
issued, state its terms specifically, describe in reasonable detail the act or acts that
are restrained in the order.
Three Types of Injunctions
1. TRO- temporary restraining injunction. IF it is an emergency can get this injunction. This
is done on very short notice sometimes day or less. Done sometimes Ex Parte. Under Rule
65 of FRCP, they cant last more than 14 days. If after 14 days, the parties prepare for a
hearing on a preliminary injunction. TRO lasts only as long as the case lasts or less.
Frequently two weeks notice.
2. Preliminary Injunction- lasts as long as the case lasts. Preliminary injunction are
appealable even though are interlocutory.
3. Final/Permanenet Injunction- a final order given at the conclusion of the case. Permanent
injunction given after whatever procedure the rules of civil procedure make available in
that jurisdiction.

G. Pepsico v. Redmond
a. Facts: Plaintiff PepsiCo, Inc., sought a preliminary injunction against defendants William Redmond and the
Quaker Oats Company to prevent Redmond, a former PepsiCo employee, from divulging PepsiCo trade
secrets and confidential information in his new job with Quaker and from assuming any duties with Quaker
relating to beverage pricing, marketing, and distribution. The district court agreed with PepsiCo and granted
the injunction. We now affirm that decision.
b. Inevitable Use + Dishonesty = Richmond shall not assume responsibilities and Richmond is forever disclosed
from Disclosing Trade Secrets (courts will be intrusive to prevent harm if equity calls for it)
c. Determination that Redmond, given the job he was going to do, it was inevitable he would use trade secretes
he learned. The inevitable disclosure doctrine has been rejected in more states its been accepted in.
d. We still have to ask how reasonably likely is it the trade secret will be disclosed?
e. Injunctions are very common in these employee-raid cases, where employee/s goes to work for a competitor
26

f.

Most trade secret information has a shelf life. The relief should last only as long as the information will be
misused.
g. Courts want to tailor the injunctive the relief to the possible harm.
h. In cases like this were the possibilities of bad consequences has a shelf life, timing is everything. 6 month
injunction in this case was a fairly long time for the soft drink business.
-court makes practical judgment about the shelf-life of the information hes going to need in his new job and what is the
commercially salient time period. 6 months was the guess by the court about how long pepsicos concern will last given the
nature of the information, and how rapidly it gets stale.
-This matters because if you are Pepsi or Quaker oats, you are thinking about what kind of relief you can get and
whether it matters.
- Court has tremendous amount of discretion in setting those balances based on time by looking at evidence presented.
-Courts provide expedited appeals for these types of preliminary injunction. The more there is public concern, or the belief the
trial court got it wrong the quicker the court of appeals will review it.
-getting a preliminary injunction or a TRO, can effectively end the case especially in cases where there is a large time element
involved.
-It is important to keep in mind, that different legal regimes can make the law itself as opposed to the remedy seem more
prophylactic is.
-See Note 9 page 291.- prophylactic injunction preventing lawyers from representing client before they did anything wrong.
-common types of prophylactic measure like notice in Almurbarti, or require training for employment discrimination like in
Goodyear. Rather than by saying dont do something, these prevent the wrong by ordering something thats less restraining.
-Remember sometimes injunctions are used as legal remedies as opposed to equitable remedies. Like Habeas Corpus
-There are no rules out there that if you have a type of injunction there are particular limiations on the form those injunctions
can take in that form.

Reparative Injunctions
A. Forster v. Boss- see casenotes brief.

The Forster were arguing two types of rights, tort rights, and property rights. The Forster argues that because
they had two rights violated they get two remedies. Rule:

o
o

No matter how many rights are


violated you only get one remedy for one wrong.
Emotional distress damages. If they had to dock their pay somewhere else and pay for it that is a classic
consequential harm . The Forsters lost sight of the fact they had other things they could have gotten
remedies for.
The court decides it will hold you in the market as long as it would take an engineer to reverse-engineer the
product after 3M releases in the market.

B.

One of the striking things in this case is the degree the court goes to tailor the injunction to the plaintiffs
rightful position.
Winston research Corp. v. Minnesota Mining & Manufacturing Corp.

o
o

See casenotes brief.

Even though they cant be prevented from using it now, they can be prevented from profiting from their
wrong. Its a reparative injunction, not a preventive injunction, or prophylactic injunction. You can
talk of it as a preventive injunction, which is using the trade secret by selling objects that incorporate
the trade secret. Reparative in the sense in that 3M is being repaired from the wrong caused to it by
Winston being kept from profiting from it. Other way reparative damages could be given is if Winston
sold their device then 3m could get damages for profits that are rightfully theres.

The difference between this and PepsiCo, is that in PepsiCo they want to prevent Redmond from using their
trade secret, in this case the defendants have already used the trade secrets.

C.

Winston says there is no longer a wrong to repair since 3m has released their product, and thereby disclosed
their trade secret.
Bailey v. Proctor

This case deals with lopsided investments. If there is a big return the stock holders get all of it, but the
bondholders are llimited to their interest even though they invested most of the money of the company. As
the managers and shareholders you have an incentive to invest the investment companys money in riskier
investment. The bondholders would prefer to see something with a modest return.
27

Company went into receivership and the court issued the orders. Court is exercising its equity jurisdiction to
protect the previously unprotected people.

Bailey comes in to run the company after all the bad things had already happened. So he says the company
should come out of receiversip. SEC says they should disband the company. The company did make a good
investment that did make them enough money to afford to pay off its creditors. Bailey says just because the
old management screwed up doesnt mean he shouldnt be allowed to run the company.

Court says even though legally the company can continue to operate. The court says under the receivership
that it has plenary powers to do whatever is equitable and shut the company down.

The debenture holders have a rightful position is that they have a right to invest in an investment company
that isnt looted. But here we dont have any reasonable apprehension the company will be looted again, the
court doesnt care. The court says because it has broad equitable discretion, they are going to give the
debenture holders a better deal than the one they actually bought.

This is clearly not a reparative injunction, or an overly reparative injunction in that it gives the
plaintiffs a better deal than they got at the outset. There is nothing compensatory about this. The
remedy in Bailey is really a prophylactic injunction in that it prevents fraud and mismanagement of
the type that was common I nthis type of company.

o
-Winston and Bailey, show us two equally valid ways to fashion an injunctive remedy. Sometimes when the courts are
confronted with a wrong that has occurred. And they are authorized to formulate specific reparative injunctive relief..
-equitable power refers to powers traditionally held by courts of equity. Powers to provide for things that the law courts dont
provide for that prevent
-Just because youre a court of equity doesnt mean you have a roving commission to do good, it has to be specific
remedy related to the specific wrong before you.
-affirmative injunctions are often though of as less constraining that broads negative injunctions.

C. Structural Injunctions (where you restructure the institutions that are violating the law)
A. Two Rationales for Structural Injunctions:
1. Putting people in the rightful position they wouldve been in but for the violation.
2. If you want to solve the problem that is not limited to one specific case, you retain jurisdiction until equity is done.
However, the law must induce compliance by shared values and goals. So, at some point, courts lose their legitimacy
when they go too far. Structural injunctions take courts to the limits of their power.
3. Structural Injunction Tension is between rightful position and equitable discretion regarding scope of relief.
1.
Structural injunctions are based on the equitable-discretion model.
-The broader that judicial power to issue these kinds injunctions makes it seem like the court is infringing more on what the
executive is suppose to do.
-Federal courts ordering States to do something: We start to get nervous when a federal court say of the many ways you, as a
state, can comply with the constitution, we want you to pick this one.
B. Brown v. Board of Education II court ordered implementation of desegregation at all deliberate speed.
C. Prisons
a. cruel and unusual punishment Hutto v. Finney 1978 pg. 307
i. Facts conditions in the AK prison violated the 8th and 14th amendment.
The conditions included unlimited confinement (solitary confinement),
overcrowding in the cells, punitive confinement, infected mattresses, and
grue (1000 calorie a day food).
ii. District court remedy
1. 30 day limit for isolation (not unlimited as it was in the past)
2. limited number of people in cell everyone had to have a bunk.
3. Had to have own mattress (no more sharing of infected
mattresses)
iii. Holding court upholds the district courts injunction.
iv. This case was about whether the prison conditions violated the 8th
amendment. Yes.
v. The district court said together the violations constitute an 8th amendment
violations. They asked the state to come back with a plan about how they
were going to fix those things.
28

1.

b.

The problem we run into is that elected officials often dont care
about people. These people include prisoners.
2. It took six years for the state to come up with a plan.
vi. This was a section 1983 civil rights case. The courts are encouraged to
award defendants fees, especially if defendants act in bad faith.
vii. The court says solitary confinement or in close quarters is not inherently
inconstitutional, but we you take into account there is a starvation diet,
overcrowded cells, and then subject people to that for an indeterminate
period of time that is unconstitutional.
viii. The problem beautifully illustrated in this case that underlies most
structural injunction casesit isnt just one or two things you can
isolate and say its illegal/unconstitutional. IT might be the totality of
things that are unconstitutional/illegal.
1. The problem with a large government institution is that its
enormous and complicated an involves lots of people, and may
have many interacting rules and policies.
ix. Whenver federalism and separation of powers concerns are presented by
the court being instrusive are overcome by the intransigent behavior of
the offending state or agency.
x. This isnt a remedial injunction and its not a preventive injunction.
b. Rehnquist in the dissent- may be saying that federal courts have no power to issue
prophylactic injunctions against state governments because of federalism
concerns. He might be a saying a strictly tailored injunction could be okay. But
The Fed courts have no power to tell a state to comply with the constitution
beyond the bare minimum the constitution requires
i. The problem with this is characteristically in institutional litigation the
situation is so complicated that there are myriad ways to achieve
minimum constitutional compliance
c. Hard Question- in a situation where youve actually concluded that it is a
pervasive problem and you have an intransigent institution to make progress, do
you have broader power notwithstanding the constitional constraints to direct
some solution to the problem even if there are many solutions
Access to libraries for prisoners Lewis v. Casey 1996 pg. 313
i. Prelude constitutional requirement to have access to library to be able to plead for their
constitutional rights.
ii. Facts AZ there was a problem with the prisoners using the libraries. There were 2 instances where
prisoners who couldnt read, couldnt use books. If in jail and you want to exercise your rights, and
want to read, assistance must be made available.
iii. Rule the remedy must be consistent with the scope of the harm.
1. Narrowly drawn and extends no further than necessary to correct the violation
2. Least intrusive means
3. Substantial weight to any adverse impact on public safety
iv. Holding the remedy went beyond the scope of the harm.
1. Overbroad all prisons (the violation only applied to two prisons)
2. Overbroad went beyond the violation (which was just illiterate prisoners); the injunction
went beyond assistance to illiterate prisoners and instead covered all forms of library
deficiencies.
3. There are ripeness concerns here. Just because you prove a few prisons have the problem
then the remedy should be applied to all prisoners.
4. For class action you have to show the class is subject to the harm, not just a few members.
v. Scalia seems to be pushing what Rehnquist did in the dissent in Hutto, which is that no prophylactic
injunctions by a federal court is allowed against a state ever. But this is unclear.
vi. What we can fairly say is that after Lewis: there is a majority opinion that cites federalism and
separation of powers as restraints on the federal court when it awards equitable restraints on state
courts.

D. US v. Virginia
a. VMI was state-support military college for men only. Ct of appeals said uncon unless VMI has comparable
college for girls. VWIL was not equal, and was declared uncon by s ct. Ordered injunction to allow women
into school.
29

i. Rightful Position Principle- put in position they would have been in but for the violation
ii. Rule: the remedy must closely fit the constitutional violation
1. Defining the violation:
a. Majority Categorical exclusion of women
b. Concurrence Failure to provide comparable education
iii. VMI decides to coeducate, even though the court doesnt order them to.
iv. Rehnquist says its really up to Virginia to come up a solution and when they come back to us will
say if its okay or not.
v. Discretion is a double-edged sword for the court- sometimes it means you give the plaintiff more
than the suffered in harm.

D. Modifying Injunctions
-Fed. R. Civ. P 60(b):
(b) Grounds for Relief from a Final Judgment, Order, or Proceeding. On motion and just terms, the court may
relieve a party or its legal representative from a final judgment, order, or proceeding for the following reasons:
(1) mistake, inadvertence, surprise, or excusable neglect;
(2) newly discovered evidence that, with reasonable diligence, could not have been discovered in time to move
for a new trial under Rule 59(b);
(3) fraud (whether previously called intrinsic or extrinsic), misrepresentation, or misconduct by an opposing
party;
(4) the judgment is void;
(5) the judgment has been satisfied, released, or discharged; it is based on an earlier judgment that has been
reversed or vacated; or applying it prospectively is no longer equitable; or
(6) any other reason that justifies relief
-Remember that a permanent injunction- is a final judgment that says based on the judges finding of the facts: the judge will offer some
constitutional relief.

-all permanent injunctions and preliminary injunctions except temporary restraining orders are appeleable.
A. Grounds for relief from a Final Judgment, Order:
a. The judgment has been satisfied, released, or discharged; it is based on an earlier judgment that has been reversed
or vacated; or applying it prospectively is no longer equitable
i. Strong presumption against relitigating the issues.
ii. Strong resistance to having defendant coming back and saying an injunction was wrong
iii. Sometimes the basis on which the injunction was issued has changed.
-Very limited degree of how final judgment can be reversed
B. US v. Swift Co
a. Nothing less than a clear showing of a grievous wrong evoked by new and unforeseen conditions should lead us
to change what was decreed after years of litigation with the consent of all concerned
b. United Shoe in 1968, said that if a degree has failed to achieve its essential purpose then the Court can
STRENGTHEN an order. Relaxing the restraints of an injunctive order are really harder but tightening the
restraints are easier.
C. Rufo v. Inmates of Suffolk County
a. US v. Swift does not apply to decrees stemming from institutional reform litigation
b. Party must establish a significant change in facts or law that warrants a revision of the decree
i. Modification must be suitably tailored.
D. Agostini v. Felton: An injunction is modifiable for significant change in law when the precedents on which it isbased have
been seriously undermined or eroded. Only the Supreme court can decide that oneo f its own decisison has been so
undermined. The court held that the proper procedure in such cases is or the district court and court of appeals to entertain
the motion to modify, because there is a claim that the law has changed, but to deny relief, becase those courts cannot
conclude that the law has actually changed.
E. Horne v. Flores (State claims changed circumstances warrant a relief from a prior injunction)
a. Facts: the case was brought by English Language Learner (ELL) students against the state board of education and
state superintendent on the grounds that the Nogales Unified School District had failed to teach the students
English, which was vital to their success. Injunction requiring adequate funding to state program.
b. Fed. R. Civ. P. 60(b)(5) permits a party to obtain relief from a judgment or order if, among other things, applying
the judgment or order prospectively is no longer equitable. Rule 60(b)(5) may not be used to challenge the legal
conclusions on which a prior judgment or order rests, but the rule provides a means by which a party can ask a
30

F.

court to modify or vacate a judgment or order if a significant change either in factual conditions or in law
renders continued enforcement detrimental to the public interest. The party seeking relief bears the burden of
establishing that changed circumstances warrant relief, but once a party carries this burden, a court abuses its
discretion when it refuses to modify an injunction or consent decree in light of such changes.
c. Rule: Significant fact/legal change Enforcement Harmful to Public Interest
d. Holding: Supreme Court says district court failed to consider 1) new ELL method, 2) No child left behind, 3)
management reform, 4) overall increases in educational budget. Consider those 4 things on remand, and see if
those 4 things are enough together to modify the injunction.
e. Takeaway: we can say its impossible to read the case as not signaling a significant relaxation of the standards for
either modifying or vacating an injunction. Well there are still finality concerns with a final judgment, we will
turn up the federalism and separation of powers concerns so that the concerns about finality will be somewhat less
heavily weighted in the balance of whether to modify or vacate an injunction. So there will be less concerns about
finality, less concern about bring up issues that were brought up before, and there will be less concern with
bringing up issues that should have been brought up before.
i.
f. (although Rule 60(b)(5) may not be used to challenge the legal conclusions on which a prior judgment or order
rests, but the Rule provides a means by which a party can ask a court to modify or vacate a judgment or order if
a significant change either in factual conditions or in law renders continued enforcement detrimental to the
public interest; the Rule thus serves a particularly important function in institutional reform litigation
Consent Decree- A consent decree is an injunction entered by agreement of the parties, subject tot the approval of the
court. Once entered, it is a final judgment and a permanent injunction just as much as if it had been fully litigated.
a. There is a little bit of flexibility with modifying a consent decree, this is to make sure the court has not been
bamboozled into thinking it had more power than it actually has.

E. Irreplaceable Losses
A. Irreparable Injury to Party Seeking Injunction
1. Rule So, for an injunction (equitable relief), must show irreparable injury: legal
damages are NOT adequate. IF cannot buy what lost with the monetary damages,
then a legal remedy is NOT sufficient.
a. Courts will NOT grant an equitable remedy IF a legal remedy would be
ADEQUATE.
b. How does this play out? Compare some equitable remedy that will prevent a
threatened injury, and money damages that will compensate for the injury after it
has occurred. IF the money damages will be adequate, then the injury is not
irreparable and will not be prevented.
c. General rule for property damages are NEVER an adequate remedy for the
loss of real estate or damage to property.
2. Pardee v. Camden Lumber Co timber (real property) as irreparable injury W.VA. 1911
pg. 363
a. Facts this appeal comes from an order dissolving an injunction awarded to
prevent the cutting of timber on a tract of land.
b. Old rule coming on s land to cut down tree, to get an injunction beforehand,
needed to show:
i. had titled to the land
ii. Trespass by the
iii. Some reason why couldnt collect damages from the .
c. New Rule to get an injunction to prevent from cutting down s timber on s
tract of land, only needs to prove:
i. has title to the land
ii. Trespass by the
d. Plaintiff almost didnt get the remedy he wanted in this case.
e. Pardee says to enjoin Camden Lumber from coming and taking my trees
f. Inability to Pay0 makes damage remedy ineffectual
g. If Pardee says money will not be enough to compensate him, the court will say
thats right, because the property was special to him.
i. It should be clear that to a significant degree we are indulging a fiction
because almost any plaintiff will be able to come up with a theory that
money will never be sufficient.
31

h.

Economic considerations are working there way into concerns about the
effectiveness and efficiency of legal remedy.
i. One thing an injunction does is that it leaves people free to make the deals that are
the most efficient
Legal remedy is inadequate unless $ can replace loss.
j. Rule on irreparable injury in this case standing timber is everywhere
regarded as part of the real estate upon which it grows. The legal remedy is
adequate, if the trespass amounts to nothing more than the trampling of the
grass, but adequacy of the remedy in such cases does NOT argue efficacy in
those cases in which part of the real estate is actually severed and carried
away, to the injury and detriment of the inheritance.
k. Land is unique, unlike horses, cattle, sheep which are replaceable. $ cant
reconvert lumber into growing trees
3. Law and Economics Theory says that trees should be cut down if that maximizes
profitability. Injunctions are good because they force the parties to bargain with each other
and historically that means that if the plaintiff refuses to allow plaintiff to cut, he valued the
trees more than the plaintiff.
4. But law and Economics Theory Breaks down where there are high transaction costs.
Under these scenarios, damages are appropriate:
a. Where there are many parties involved
b. Bilateral Monopoly where there are only two people, there are also high
transaction costs because there are no market alternatives for either side to go to.
***Economic Theory says: Where transaction costs are high, court should use damages (legal relief). Where
transaction costs are low, court should use injunctions (equitable relief).
-When were talking about irreparable harm we mean harm that can not be adequately repaired with damages.
-Whenver you order specific performance youre putting people in a bilateral monopoly
B. Brook v. James A. Cullimore & Co. Plaintiff sued defendant in replevin for the property used as collateral for a mortgage
note. Prevailing party gets choice between money and property. So, if the return of the property sought by relevin is
possible, it must be returned. Cullimore had the right to Brooks property. Replevin is legal and not equitable, so no
irreparable injury needs to be shown.
a. Rule: A successful litigant in a replevin action has a right to the subject property, rather than cash.
b. This ensures an equitable remedy.
c. You could not seek replevin or trover in the alternative.
d. If you seek replevin youre seeking return of the specific piece of collateral
i. With replevin it is a legal remedy, which means you dont have to show there is no adequate remedy
at law to get it. No worry about whether the property is unique or not. Just have to show right to
possess the property.
e. Trover: you are seeking the value of the collateral.
C. Continental Airlines Inc. v. Intra Brokers, Inc Difficulty of calculation may prove legal remedies inadequate
a. Facts Continental Airlines had discount vouchers it gave to passengers. These were not allowed to be sold,
but allows a broker to sell them. Continental () decides not to use the vouchers anymore and tells Intra
Brokers () to stop using them.
i. seeks an injunction to stop from selling the discount vouchers.
b. Issue is there an irreparable injury?
i. Continentals Argument this is undermining their business by undermining their authority and
control over vouchers.
c. Rule the difficulty and expense of proving amount of economic harm may indicate legal remedies are
INADEQUATE.
i. Where the injury defies calculation, damages will not provide a remedy at law.
ii. Court says there is irreparable harm to Continentals right to conduct business the way it wants
to. (issue of control)
d. Court says Continental is entitled to price discriminate and to set prices as they see fit.
i. The power to control the price that you charge is less clearly a property right than the trees in
Pardee, but Court says your pricing and the way you conduct your business is something were
going to protect by injunction.
D. Specific Performance injunctive remedy available in contract cases that directs breaching party to perform contract
to its terms.
32

a.
b.
c.
d.

Most of the itme we are protecting the expectancy interest in the contract, so most of the time contract
remedies are the payment of money.,
In a lot of jurisdictions people will say in order for a contract to be specifically performed, the contract has to
have a level of clarity to it that is greater than usual
Often, in order for there to be specific performance the performance can not be impossible
In some jurisdictions, plaintiff must plead and prove that plaintiff is ready, willing, and able to perform its
part of the contract.
In the situation where the contract is not performed on both sides, we look to give plaintiff
difference between what it would get under the contract and what it would pay.

E. Campbell Soup v. Wentz Campbell entered a contract to buy Chatenay red carrots from Wentz. The price went up, so
Wentz sold the carrots to someone else for more money. Campbell sued Wentz for specific performance. Campbells said
they couldnt get the carrots from anyone else and having regular carrots would lead to irreparable injury to its reputation.
But they couldve covered for three times the contract price. Court says, in times of scarcity, you can get specific
performance for things that are normally not unique because legal relief is inadequate. Specific performance is
often granted when it is difficult to cover. Here the goods were not available on the open market, so they were unique.
Interesting thing Campbells did while the case was pending was pay the farmers $30 and then pay the $60 difference
between the contract price and market price into court till it was determined whether the contract was enforceable. This is a
case too where consequential damages would probably be available because campbells would have to pay to find other
farmers, and hire a broker, risk that the carrots they cover with arent as high a quality and they cant produce as much soup
a. Big issue in this case is whether there is a damage remedy or a specific performance remedy.
b. In this case when you think about it, damages are sufficient because there is a market for these carrots. Its not
a problem where its difficult to determine the value so there is a real serious risk that you miscompensate.Yet,
the court says carrots are hard to get and maybe impossible to get under the historical circumstances. Here
what was taken from them was the predictability and stability of the arrangement. The court says that in
either all situations or at least this one, were there is scarcity, equitable relief should be granted liberally.
i. Three things weird about this:
1. Campbells has already engineered the situation so that they have the carrots and the money
is with the court. There is no problem of Campbell having to scramble to get the carrots
now, no scarcity issue
2. Into the contract, Campbells had built in a liquidated damages clause Once you built a
liquidated damages clause into your contract, arent you really saying heres what Ill
accept in dollars for a breach. So the subject of the contract is really unique and special,
theyve specified the amount of money that will substitute for the carrots
3. Court ends up saying the contract was unconscionable, but determines that specific
performance would have been warranted had the contract been enforceable.
c. Scarcity is the sort of thing that would suggest to economists that there should be a damage remedy because
it would be hard for the parties to reach an agreement, but is usually one of things that weight toward courts
granting injunctions/specfici performance
i. Uniqueness is a big argument people argue when they want a specific remedy.
d. Otherways to argue uniqueness: scarcity, time constraints (you need something immediately need TRO or
preliminary injunction, cant get damages and wait for months); if youre a bulk consumer like Campbells
its a lot harder to cover; if you have a very volatile market (may create damages so big defendant cant pay
them and hard to determine the date on which a good would be priced which could make damages hard to
determine)
e. UCC takes the view that specific performance ought to be available a lot more often than common law
suggests. 2-716
f. Even though equitable relief is suppose to be only awarded when there is no adequate remedy at law, and
there is a likelihood of irreparable harm.But Pardee and Campbell shows that the harm doesnt have to be
only to an objectively unique good but a subjectively unique good, like the land in Pardee or the carrots that
Campbell uniquely values, even though there is a ready market for the carrots and they are not scarce.
g. Rule a party may have specific performance of a contract for the sale of chattels IF the legal remedy
is INADEQUATE.
h. Inadequacy of a legal remedy is to be determined by an examination of the facts in each particular
instance.
i. Specific performance (even of ordinary goods) in times of scarcity, time constraints, or sheer size of the
contract makes it difficult or impossible to cover EXAMPLES:
33

i. 4,000 tons of cryolite (used to make alumnimum) when there was a limited global supply (Kaiser
Trading Co v. Assoc. Metals & Minerals Co.)
ii. jet fuel when fuel was in short supply during 1973 war (Eastern Airlines v. Gulf Oil Corp.)
iii. sale of tomato crop when reliable supply was not available on the open market (Curtice Bros Co v.
Catts)
iv. gravel, when there was no other source of gravel (Frame v. Frame).
j. Courts are more likely to grant specific performance if the breaching party breaches opportunisticallysituation where court determines that breaching party took advantage of non-breaching party. Efficient breach
seems indistinguishable from opportunistic breach.
i. This stems from two views of contract law : 1. That it is merely meant to facilitate pareto optimal
exchanges, no moral component beyond efficient allocation of resources 2. Moral sense- that
contracts involve a promise and breaking a promise is a blameworthy act.
k. Whenever youre talking about equitable remedies, talking about whats fair is effective.
F. Van Wagner Advertising Corp. v. S&M Enterprises - uncertain damages are irreparable
c. Facts Van Wagner built a billboard on the side of a building. Michael owned the
building and Asch rents the billboard space from Van Wagner. Michaels eventually
sells the building to S&M Enterprises, who cancels the lease with Van Wagner.
Van Wagner seeks an injunction to stop S&M Enterprises from canceling the
lease.
d. Is there irreparable injury?
i. YES Van Wagner injury was irreparable because of the prime
location of the sign.
ii. NO Michaels (court agrees) the injury is not irreparable.
e. Rule
i. A2: physical location is unique the fact that the subject of the
contract may be unique as to location for the particular advertising
purpose intended by the parties does not entitle a plaintiff to the
remedy of specific performance.
ii. Damages should be uncertain a breach of contract is redressable by
specific performance when there is uncertainty in valuing the
damages.
f. Application the value of the commercial billboard space can be readily
determined and can be fixed with reasonable certainty and without imposing an
unacceptably high risk of under compensating the injured tenant. Billboard space
= CERTAIN damages and not irreparable
g. Van Wagner wants the injunction because it gives them tremendous leverage over
S & M, since S &M would then have to honor the lease, then S & M cant tear
down the building like they want to.
h. S & Ms breach in this case is an example of an efficient breach.
i. This court took all the same arguments made in the Campbell soup case and came
to completely the opposite conclusion.
j. Part of whats going on in the time between Campbells and Van Wagner, is the
rise of the law and economics movement.
k. Uniqueness ends up being more associated with property than it does with
contract rights

F. Balancing The Hardships


G. Whitlock v. Highlander Foods (Balancing Hardships) (balance of Equities)
a. 18 inch below ground encroachment onto Plaintiffs property. P noticed soon after construction and
withdrew permission to use his property. P sues Defendant, seeking to remove the encroachment.
b. Holding: ordinarily, in deciding whether to order a defendant to remove an offending structure, the trial
court must balance the hardship to the defendant against the benefit to the plaintiff; if the former is great and
the latter slight, the court will ordinarily leave the plaintiff to his remedy at law. If the encroachment is
deliberate, the court may issue the injunction without considering the relative hardships.
c. Court says it will not balance the hardships (i.e. expense and difficulty of removing the encroachment)
in favor of defendant where the act was intentional. Intent can include failure to take adequate
precautions before building i.e. reckless building.
d. We dont in this case have any answers about what was factually true.Ultimately, the court doesnt
make any finding about who was the bad guy.
34

e.

IF we look at the hardships in this case, its really expensive to take the wall down at this point, and the
encroachment is minor because its only underground. Theres nothing the plaintiffs doing that the
encroachment prevents. So The defendant has a better case here on the basis of hardships
f. The plaintiffs argument for injunction is that the defendant trespassed and trespassers are
traditionally ejected. The plaintiff is also making the policy argument that if damages are sufficient
that the infringer has the right to take the plainitiffs land for his own private purposes and make
payment. Like a private right of imminent domain.
g. Encroachment cases are hard because they are usually a major problem to fix for the encroacher and
are a very minor problem for the plaintiff-landowner.
h. The court points out that it is not enough just to balance the hardships, you have to take the other
equities into account (the other creiteria for getting an injunction) like did the defendant do it on
purpose.
i. Summary: we will look up at balance of the hardships to defendant vs. plaintiff, and vice-vesa for
damage remedies. An intentional breach of contract will not be as serious as an intentional tort, taking
into account that at some point some intentional breach of contract may be an opportunistic breach of
contract that we look down on even though the difference between opportunistic breach and efficient
breach seems kind of arbitaray
j. Even when the damages will turn out to be nominal or trivial, they will usually not be taken into account
when considering the balance of equities.
k. There is some ways that the balance of equities reflects the law-and-economics consideration- if the specific
relief that normally would be merited is too wasteful then it would be denied. (tearing down a wall because
of a 18-inch encroachment would be too wasteful, when they get just pay money for an easement)
H. Where there is unreasonable delay from Plaintiff in bringing the claim is called laches
I. One way you can think about bad faith that would otherwise upset a clear balance of hardships- is that if the defendant
did act in bad faith the court will likely order the inefficient remedy, i.e. iinjunction in cases like minor encroachment
on land.
J. Undue Hardship
a. This is the laws most explicit embodiment of the economic approach. If a wrong is too expensive to correct,
defendant can pay damages instead.
i. It is not sufficient that the injunction costs a little more than it
saves the ; the injunction must impose hardship greatly
disproportionate to the benefit.
b. Boomer v. Atlantic Cement Co. NY 1970 pg. 405 cement plant was pouring dust on homes of s,
despite best technological efforts. Court held the was a nuisance but REFUSED to enjoin operation of the
plant because of undue hardship on , who invested $45 million and employed more than 300 workers.
ii. Holding court gave s $185,000 for reduced value of their homes.
iii. Rule to prevent an injunction, the hardship must be
disproportionate to any benefit will derive from the injunction.
c. Relevant factors in evaluating undue hardship
i. Defendants culpability;
ii. Relationship between the parties
iii. s diligence or acquiescence
iv. Whether denying relief would be a public or private benefit.
d. What happens when the court denies an injunction because of undue hardship? Answer plaintiff gets
damages instead.
i. The damages could be nominal or could be substantial.
G. Cooperative Insurance Society v. Argyll Stores Ltd. (too difficult to enforce injunction/Administrative Burden)
Safeway had a clause in contract that said it had to stay open during normal operating hours. Safeway wanted to give
up store because it was losing money. Court said you cant make Safeway stay open, but you can get money damages
to business other tenants lose. Court doesnt give injunction because too much difficulty for court to enforce and
too much harm to defendants.
a. Why did the shopping center owners want Safeway to stay open? IF Safeway stays open(an anchor tenant) it
brings in business for all the other businesses.
b. The process of determining whether or not Safeway was not in contempt of the injunction, if the injunction
where to be granted would be very unadministerable.
c. This case illustrates the problem with positive injunctions.
d. There is also a balance of equities here: If the defendant operates this at the levels it did before it breached
the lease it will continue to lose money. If you compare the amount of money the defendant will lose by
operating and the amount the shopping center will get from the lease it is a big burden on the defendant.
35

a. Rule the most frequent reason given for declining to order someone to carry on a business is that it would
require constant supervision by the court. Supervision would in practice take the form of rulings by the court, on
applications made by the parties, as to whether there had been a breach of the order.
I. eBay v. Merc Exchange (High priority) Merc Exchange has a patent that eBay needs (essential to its business). Merc
Exchange normally only charges $100 for the patent, but because it is essential to eBay and eBay is rich, Merc Ex holds out for
$ millions/billions. Court applies 4-part test for an injunction and denies the Injunction and provides $ damages for patent
infringement.
Four-Part Test for Injunction in federal courts:
1. That it has suffered an irreparable injury
2. That remedies available at law, such as monetary damages, are inadequate to compensate for that injury;
3. That, considering the balance of hardships between the plaintiff and defendant, a remedy in equity is warranted;and
4. That the public interest would not be disserved by a permanent injunction.

Important case: even though people talked about it as a patent case, the supreme court used it as an
opportunity to clarify rules of equity in federal court
Court says there are no special considerations for granting injunctions in Patent cases. The same
considerations for granting an injunction under general equity principals are to be taken into account.
-Roberts concurrence suggesst that when patents are infringed an injunction should issue, not because they are patents but
because they are property rights.
-Kennedys concurrence (joined by Souter, Breyer) says we have to apply the remedy that makes sense. He says the four-factor
test is the appropriate test in this case is correct. He says the unusual fact that the patent holder in this case hadnt used it should
be taken into account.
-It was determined in this case that the patent is good and that eBay did infringe the patent. Ebay says that infact this payment
impacts a very minor part of their business
-The overwhelming presumption was that if a patent is valid and found to be infringed, the court should enjoin the
infringement.
-Issue comes up that when the patented device is a very small part of the overall business or product, enjoining its use can
impose huge costs on the infringer by having to completely retool their business or reengineer their product to not use the
patented device.
-Case is screwy cause the court screwed up the injunction standard for federal courts, but the test applied here is the law.
-This case is telling us about injunctive relief, that at least 4 justices are telling us there is a lot more room to do what makes
sense, we may decide that a property right may end up being enforced by damages.
-A patent right that involves a right to be exclude may also be enforced by damages.
-In particular, as Kennedy talks about, the difference between sloppy encroachers as opposed to deliberate
infringers, Where it is a non-practicing entity as opposed to a patent holder who exercises the patent, you
should seriously consider not granting an injunction. (Not granting an injunction for patent infringement was
pretty much unheard of before eBay.). This view is not limited to patent cases, but all cases where injunctive
relief is at issue. So you can argue it in other property right/injunction cases.
-This case now suggests you really need to write brief in four-factor format
-When there is a bilateral monopoly there is a lot of room for bad negotiationg, that may make an injunction untenable,
according to law and economics view.
-An injunction in this particular case would give mercexchange a power to get money in excess of the actual market power of
itself. That would be a good reason to think about not granting an injunction.
Willing v. Mazzocone see case brief in book. Case about Rule Against Prior Restraints
Mrs. Willing marching in front of lawyers office saying he stole money
Mazzocone tries to shut her down. Ancient English rule says equity will not enjoin a libel.
In the United States have a presumption that enjoing defamation is highly problematic.
o Because even when we have a good idea that a certain kind of speech will be harmful (especially because we
had a trial saying her statements were false) because of the 1st amendment we have a strong presumption
againt enjoining speaking or publishing.
o Standard for enjoining speaking for publishing comes fromt Pentagon Papers case about Daniel Ellsberg.
This case tells us there are really strong policies that oppose the granting of injunctions.

36

In this case this woman would never be able to pay a judgment, any assessment of damages would be insufficient to
keep her from disparaging the attorney. So remedy at law is inadequate, and the harm is irreparable, but despite this,
because it involves speech were not willing to enjoin speech outside a commercial context.
o Even outside the commercial context the rule against prior restraints is very powerful.
The rule the court cites in this case that they do not consider the ease of collection of damages is not considered when
assessing the inadequacy of damages is a very small minority rule.
Multiplicty of Suits problem notes page 439- Sometimes its impossible to discount future harm. The Willing case is a good
example of that because you wont know how long shell keep disparaging you, Whether you would have to bring multiple
lawsuits over a period of time is a good reason to argue that damages are not sufficient and an injunction is what you need.

H. Preliminary Or Permanent Relief


-very frequently people rush to court because there is something going on and might be going on during
the case that is threating to upset the state of things and leave someone worse off in a way that is unfair.
-After the close of the case the court can decide to vacate a preliminary injunction. This doesnt happen
often because usually the balance of hardships is so obvious that the prelim injunction was warranted.
-The major difference is that when we issue a preliminary injunction we know pretty much everything
were going to know about the facts. Under the discovery rules, the parties have gathered all the rules they
are legally entitled to gather, so after discovery we have all the information were going to get.
-The preliminary injunction happens on short notice and the facts may not have been argued so
there is a risk it is wrong.
-The preliminary injunction is on an incomplete record, happens on short notice, and they last for a
temporary period of time.
-While there was never a traditional four-factor test for permanent injunction there was a standard fourfactor test for preliminary injunctions
-The ruling on a preliminary injunction is not binding on any one for any purpose. It does not mean that
the preliminary injunction was proper. It is just a temporary provisional set of findings that doesnt bind
any one. You can come back during the pendency of the case and challenge it (based on changed facts or
changed law).
A. Three Procedural stages of Injunctive Relief:
1. Temporary Restraining Order issued at the beginning of the lawsuit and get 24 hours notice. Lasts 10 days, up to
20. Preserves status quo to prevent immediate harm until the preliminary injunction hearing.
2. Preliminary Injunctions issued during the case before trial. Must give 21 days notice. Lasts until final judgment.
Preserves status quo pending judgment.
3. Permanent Injunctions issue at end of litigation. Lasts a long time. Prevents irreparable harm from end of
litigation onwards.
A. Temporary Restraining Order Preserves status quo from the complaint until the time when a preliminary injunction
can be ordered. It lasts for a maximum of 10 days and can be extended 10 days. (Usually you must give 15 days notice
until the preliminary injunction must be ordered).
B. Preliminary Injunctions
1. Preliminary injunction the goal is to preserve the status quo until judgment and then at that point, the court can
determine if a permanent injunction is appropriate.
2. Granting or denyinf TROs are not appealable
3. Granting or denying of Preliminary injunctions are immediately appealable.
4. Two requirements for preliminary injunction
i. Strong likelihood of success Moving party must show there is a strong likelihood that they
will prevail at the end of the case.
ii. Bonding requirement moving party must post a bond (sum of money or insurance bond) in
the event the court is wrong in granting the preliminary injunction, and then the bond will go
to the damages of the party wrongfully enjoined.
1. Purpose To pay the damages to the party that was wronged by an injunction improperly
granted (and ultimately rejected by the final judgment).
2. Two kinds of mistakes made by courts
37

a.

3.

Erroneous denial rejected a preliminary injunction when it should have been


granted
b. Erroneous grant grant preliminary injunction when it should NOT have granted
it.
i. Compromise approach aggrieved party can get damages for
erroneously granted preliminary injunction. BUT, aggrieved party
can ONLY get the money from the bond (not more) and is not
entitled to whole bond if damages were less serve from erroneous
grant of preliminary injunction.
4 equitable factors relevant in decision to grant preliminary injunction
a. Likelihood of success on merits Strong showing of likelihood of success on
the merits
b. Irreparable Injury Possibility of irreparable injury to the moving party if
the injunction is not issued
c. Balance of hardships favors the moving party the hardship in not granting the
injunction (on moving party) outweighs the hardship in granting the injunction (on
responding party).
i. We dont know who the wrongdoer is in the preliminary injunction stage,
SO we balance the hardships in light of the relative likelihood of success
on the merits.
d. Advancement of a public interest (only in certain cases).

B. Winter v. NRDC (High Priority)


a. This case is very hostile to preliminary relief
b. Sonar affecting whale and other marine life.
i. Merits Serious Question
ii. Irreparable Harm Possibility
iii. Balance of Hardship + Public Interest Speculative for the Navy
iv. The four part test is a SLIDING SCALE
c. Burk thinks the majority came out the way they did in this case because they thought the statute in question
was stupid.
d. This case is unusual in that it isnt a negative injunction, but a a positive injunction, an injunction ordering
them to do something.
e. Because NEPA requires Environmental Impact Statement then there is a high likelihood of success.
f. Robert for the majority: says an injunction should not be granted on the mere possibility of irreparable harm
but with the likelihood of irreparable harm
g. Ginsburg and Souter dissent: Ginsburg says that having the report before you make the decision is required.
The irreparable harm is not doing the impact statement. She says the traditional test is a a sliding scale, she
agrees likelihood of success is great.
h. The court says we dont care how likely success is, if there only a possibility of irreparable harm then that is
not enough to grant a preliminary injunction..
i. By the court suggesting that this is a four-factor enumerated test instaed of a balancing test, the court is
saying that a preliminary injunction will be harder to get in close cases
j. The majority says to get prelim injunction there has to be likelihood of irreparable harm, but even if there is
irrerable harm we have to look at the public interest. You could say the court has taken what was a balancing
test and turning it into an enumerated test where they have to win all factors. It is unclear here whether they
are balancing the factors or concluding that because the plaintiffs didnt win on one of the factors they failed
the whole test.
k. In Ginsburg view, she says they are not changing the standard in the majority they are just balancing.
l. If you were arguing for an injunction you would say its a balancing test like Ginsburg says If you were
arguing against the injunction you would say Ginsburg is wrong, and the it is an enumerated standard were
all four factors have to be met.
Rationale for preliminary injunction- the outcome of preservation of status quo analysis should be that we are
tyring to put all the parties in the best situation based on the likelihood of success vs. the balance hardships
C. Coyne-Delany Co. v. Capital Development Board (bond requirement) Capital did not allow companies bidding on a
construction project to use Coyne plumbing materials in their bids. Coyne sued for denial of the right to bid. Court said
38

that there is no property right in the right to bid. Court held the prevailing party can only get damages in the amount of the
bond ($5000), even though they have been damaged more. A defendant who prevails, but is injured by the preliminary
injunction is limited in damages to the amount of the injunction bond. If defendant is worried about being limited,
argue for a higher bond. There is a financial impact of requiring a bond: shuts out consumers, workers, from getting a pi.
-no substantive due process right to bid on government contracts.
-The very strong likelihood of success that Coyne-Delaney had goes away and they lose the case.
-The defendant comes forward and said that theyve been hurt by the bidding process not being able to go forward
with their projects. They have to rebid the job, and now the low-bid on the next round of bidding is $56,000 higher. So
the harm from issuing the incorrect injunction is $56,000..
-Generally, Injunction bonds are the sole and exclusive rule for wrongful injunctions.
-That means if youre the enjoined party you to need make sure the bond will cover your potential losses.
-In this case the bond is for $5000, so the defendants only get that amount, instead of the full $56,000.
-They have right to appeal preliminary injunction and that the bond was set wrongfly. Defendants didnt
appeal preliminary injunction.
-Even if injunction is right and bond is wrong that is appealable. Defendants didnt appeal this way either.
-The fact that it was not reasonably foreseeable that the law changed, doesnt mean that the injunction wasnt wrongful. The
record can be completely in good faith but incomplete.
-Posner says Rule 65 creates a principal of preference for requiring payment from the injunction bond, but it doesnt mean its
mandatory.
Rule 65( c) Security. The court may issue a preliminary injunction or a temporary restraining order only if the movant gives
security in an amount that the court considers proper to pay the costs and damages sustained by any party found to have been
wrongfully enjoined or restrained. The United States, its officers, and its agencies are not required to give security.
Situations were courts usually dont requiring posting injunction bond
-public interest group enjoins enforcement of law they feel is uncsontitutional and the public interest group doesnt
-Some district courts say they have discretion in the amount they can charge on the bond. Since they can set it in a nominal
terms they might as well set it as nothing and not require posting a bond.
-So if there is a good reason courts will waive the injunction bond
-Or if there is a good reason some courts will set the injunction bond at a deliberately low amount.
What if there is a discrepancy between preliminary injunction and permanent injunction?
-You have to show discrepancy between preliminary and permanent relief caused harm and you have to show the amount of the
harm
-Another question that comes up is about the money/fees the defendant has to spend to show the injunction was wrong? In
federal system, attorneys fees are not allowed. Some states allow recovery of the fees necessary to show that injunction was
wrong.
-Ordinarly, the bond is the only remedy you have if it turns ou the prelim is wrong.
-But sometimes the Court will waive the bond on the basis of a promise by the plaintiff to pay if there was a
wrongful injunction
-The courts will usually waive the bond if it turns out there was bad faith on the plaintiffs part for getting the
injunction. Court will do two things in this situation
-Plaintiff will have to pay full amount of the damages, and surety is only liable for the amount the
surety promised, or
-Court will impose Article 11 sanctions, or
-If the claim brought by plaintiff to get injunction amounts to malicious prosecution or abuse of
process, then defendant can bring state law claims for those, or
-If the injunction has granted an unfair benefit to the plaintiff and that can be taken away by the
court.
a. Notes on Injunction Bonds
i. Financial Impact of bonds is that it will keep out certain Ps, such as civil rights Ps or low wage
workers, who cannot perform the bond.
ii. Usual rule is that preliminary injunction merges into a permanent injunction, and that any appeal
from the preliminary injunction becomes moot.
b. Preference to allow Recovery for Harm
i. Unless good cause (e.g. failure to mitigate)
ii. Good faith is not good cause
c. Recovery up to bond amount
i. TRO bond v. PI bond
ii. Move to modify if inadequate
d. Injunction bond is not a liability policy it is a surety arrangement
i. An injunction bond is not an insurance policy
39

ii. Injunction bond allocates risk of harm, that is the harm that the injunction was wrongly ordered.
1. If it turns out to be a wrong injunction then plaintiff will pay for the harm caused by the
wrong injunction with the amount of the bond
a. The bond provides of means of having the defendant not having to bring a
separate action against the plaintiff for harming them if the injunction turns out to
be wrong, so we get the plaintiff to post money for the bond when the injunction is
issued.
b. Plaintiff usually goes to a third-party insurer (surety) that will promise to pay up
to the amount of the bond in the event of a wrongful injunction. Then all the
defendant has to prove the harm by motion, Rule 65(1) allows defendant to
proceed by motion against the surety even though the surety is not a party to the
case.
i. This provides a lot more certainty for the defendant to collect if the
injunction is harmful
c. Ultimately, the plaintiff is still on the hook, because the surety can come after
them for repayment.
i. Suretys get a fee, that is 1% of the amount they guarantee.
ii. Surety will often have a security agreement with plaintiff on the amount
of the money to get more guarantee they will be paid back.
d. TROs have bonding requirement as well.

The Procedure for Obtaining Preliminary Relief


D. Temporary Restraining Order
a. Difference from preliminary injunction
i. Timing while a preliminary injunction is decided pretty quickly into the process, a temporary
restraining order (TRO) can be brought even before a preliminary injunction in order to preserve the
status quo.
ii. Length a TRO lasts for a short time (10-15 days) until a hearing can be held. If the result of the
hearing is an extension of the TRO, it is now a preliminary injunction.
1. Immediate harms a TRO is really only to stop the most immediate of harms.
2. Ex parte a TRO can be granted ex parte, as long as the other side is given notice. In that
case, the TRO can only last about 10 days before a hearing must be held with the other
side.
i. The notice to opposing side is the day before.
b. Notice required Carroll v. President of Princess Anne 1968 pg. 459
b. Rule cannot get a TRO without notice (even if you can get a TRO ex parte,
still must provide notice). (absent showing its impossible to notify)
c. Why? The court is opposed to ex parte motions because there is no adversarial
proceeding. While the court will grant a TRO ex parte, it is too much to grant a
TRO without even notice to the other side.
d. Local city government got TRO without notice to the defendants.
e. This case comes before the Supreme Court on the question of whether the TRO
enjoining the white supremacists from protesting for 10 days is a problem
f. This proper way to test an injunction is to appeal it directly, until injunction is
vacated or stayed, you must comply with it.
g. Case concludes that TROs require notice to the other side. By notice they mean no
ex parte proceeding
i. Exception: Rule 65 says there are two ways to get TRO without notice to
the other side
1. Cant find the defendant. Have to give evidence to the court you
couldnt find them.
2. They thing you are trying to prevent the defendant from doing
will happen before you get the TRO if you give them notice.
Example, if you are worried defendant will take property they
you are enjoining them from doing, they might do it if you give
them notice. This is also common in domestic abuse situation.
h. The problem with this case is that there was no good reason to get the TRO
without notice.
E. Appealability of a TRO versus preliminary injunction Sampson v. Murray 1974 pg. 464
40

a.

2.

3.
4.

Facts was working for government agency and was dismissed based on a
memorandum that included information about something the had done in the
past.
i. District court granted TRO preventing from being terminated. This
turned into a preliminary injunction because it lasted more than 10 days.
b. This is like the Carey v. Phibus, in that the plaintiff in this case is saying that she
was suspended without a proper hearing.
i. What started as a TRO turned into a preliminary injunction of indefinite
duration.
ii. The civil service argues that this is a preliminary injunction and it is
appealable.
1. They argue that TRO can not last more than 10 days, this lasted
more than 10 days so its a preliminary injunction, and because
its a prelim injunction its appealable.
c. Holding preliminary injunction was reversed because there was no irreparable
injury because the injury was to s reputation and loss of work, both compensable
with monetary damages.
d. Issue this was a TRO, so is it appealable?
e. Rule an order issuing a TRO is NOT appealable, but a preliminary
injunction IS appealable.
i. TRO can still be reviewed for procedural deficiencies.
ii. Application this was a preliminary injunction, even though it started
out as a preliminary injunction.
f. Dissent (Marshall) this is a TRO, not a preliminary injunction.
i. A TRO with no notice can only last 10 days.
ii. BUT, a TRO with notice can last longer (FRCP 65 is ambiguous on this
issue).
There is a separate set of arguments in Sampson v. Murray about whether the substantive
granting of an preliminary injunction was correct. These arguments go to the four-factors
discussed before for granting preliminary injunction.
a. Argument about irreparable harm of being fired:
Traditionally when youre looking at injunctive relief courts are hesitant about forcing
employee to continue to work or employer to continue to employ.
Important Procedural Points: Rule 65 is the law about preliminary injunctions in the federal
courts
a. Rule 65(b) says there is a time limit on what we will call a TRO
i. The actual language says there is a time limit on TROs issued without
notice, the language doesnt address TROs with notice
1. But we treat the 14 days as the limit for all TROs and court can
renew TRO for another 14 days for exceptional circumstances
and other party agrees.
b. Granny Goose case note 2 and 3p. 465:
If an order doesnt have a time limit on it, and its otherwise consistent
with being a TRO then it will be considered a TRO
i. IF it lasts longer than 14 days it will be considered a preliminary
injunction and will be appealable.
ii. IF its a TRO there has to
iii. IF its a preliminary injunction there has to be findings and conclusions
justifying it.

I. Declaratory Judgments
J. All declaratory remedy is declaration of remedies.
a. There is a general right to declaratory relief and it it comes from statutory law.
i. All the statutes say if there is a controversy and there is uncertainty about parties
rights and obligations vis a vis one another. Then the court can declare what those
rights and obligations are.
ii. We have problem of ripeness with declaratory remedies. Nashville case deals with
this.
41

A. Declaratory Judgments
iii. What is a declaratory Judgment?
1. When a party asks the court to declare someones rights (e.g. what is the boundary,
where is the boundary, is this patent claim valid, etc.)
iv. When do declaratory judgment cases come up?
1. cases where declaratory judgment is asked for in addition to everything else
2. when declaratory relief is the best and most appropriate step.
v. Examples of places where declaratory judgments come up
1. border disputes
2. marriage determinations
3. wills
vi. Declaratory judgments are constitutional Nashville, Chattanooga, & St. Louis Railway v.
Wallace
1. Facts TN passed a gasoline tax that charged additional taxes for gas purchased outside of
TN. The gasoline is stored in the state and then sold both inside and outside the state. While
the gas is sitting in TN, it is taxed and the railway sues, claiming the TN cannot regulate
commerce in a way that unduly burdens interstate commerce.
a. Remedy sought? a declaratory judgment that the tax is unconstitutional.
2. Rule declaratory judgments are an adjudication of a real actual disputenot a
hypothetical controversy.
a. At trial court State of Tennessee wins. First question the Supreme Court asks is :
i. Do we have jurisdiction?: they conclude that they do under the
declaratory judgment statutes. They say it satisfies Artcle 3 Section
2{which requires a case or controversy for Sup. Court to have
jurisdiction} of the Constitution because there is an actual controversy
between the railroad and the state.
ii. The court says we judge whether there is an actual case or controversy on
a practical basis. As long as the controversy is real and not so general as
to amount to advisory opinion it is judiciable under Article 3 section 2.
b. There is now Federal Statute that says declaratory relief is available.
3. How to determine if dispute is ripe:
4. One of biggest areas where ripeness concerns arise is patent cases. Where defendant wants
declaration that patent is invalid or that they are not infringing.
vii. Declaratory relief is prospective, it defines rights going forward.
viii. Declaratory judgment is not coercive unlike an injunction, It just tells parties what rights and duties
are
ix. Declaratory judgements are not equitable in nature. Therefore you dont need to show that
declaratory judgment is needed to avoid irreparable injury or that there is no adequate remedy at
law.
x. Why seek a declaratory judgment instead of an injunction? Answer: Easier to get a declaratory
judgment.
1. If suing for an injunction, must show
a. imminence of the harm
b. irreparable harm
c. balancing of the equities of the parties
d. public interest
2. If suing for declaratory relief, must show
a. imminent harm and
b. a real dispute.(ripeness)
c. do not need to show irreparable harm, public interest, or balancing of the
equities of the parties.
3. Often delcratory judgment is sufficient and you often might not know exactly what the
other party will do that will cause you harm.
4. The declratory judgment laws generally say a declaratory judgment is a final judgment.
a. If a declaratory judgment is a final judgment how do you reconcile that with the
ability to get supplemental relief. So if you just got declaratory relief what about
the problems of res judicata and collateral estoppel?
i. Declaratory judgment is only loosely preclusive.
42

ii. It is determinative as to the parties for any issues actually litigated, but it
does not preclude a new lawsuit for a remedy. This is how we enforce
declaratory relief. See NOTES PAGE 577-578.
B. Standard for Declaratory Judgments - To get a declaratory judgment, plaintiff needs to show standing and that there is a
controversy. Plaintiff does not need to show irreparable harm. But a declaratory judgment only declares rights, it doesnt
tell anyone to do anything. Also, it is not backed up by contempt. If someone violates declaratory judgment, the other
party must get an injunction.
C. Cardinal Chemical v. Morton International Morton owned a patent that it claimed Cardinal infringed on. Cardinal
countersued for a declaratory judgment that its patent was invalid. Appellate court found in this case and in other cases that
if there was no infringement by the alleged infringer, it didnt need to decide the validity of the patent. So Morton
continued suing as many people as possible. US SC rejects the Appeals Courts method, so it must determine whether the
patent is valid. Declaratory judgments resolve uncertainty as to whether certain specific conduct is lawful. Party may
seek Declaratory Judgment even if no Infringement.
a. Rule a counterclaim is separate from a claim and it cannot be avoided merely because the original
claim has gone away. (
b. Ripeness in the context of a declaratory judgment.
i. Two requirements
1. Declaratory plaintiff must show that it has a ripe intention to do something that may
violate the patent AND
2. That the patent holder has a ripe intention to take enforcement action
ii. Remember mootness is a thing that destroys a case or controversy. Mootness arises from other
events that turn a case into a non-issue.
D. Supplementing the exiting of statutory declaratory relief is the range of piece-meal declaratory remedies some of
which that came from law courts and equity courts. Ex. Quiet Title and Restoration like in Newman case. Two reasons
we care about these:
i. Legal claims get evaluated by juries. Equitables claims dont
1. In federal courts there is a constitutional right to civil jury in cases where case or
controversy exceeds certain amount from Bill of Rights. Rule of Beacon Theatres says that
If you want equitable relief, if you really want a jury you need to bring some legal claim
also that requires finding of similar facts for equitable relief.
2. To get equitable relief you need to show irreparable harm and inadequate remedy at law
.

K. Quiet Title And Reformation


A. Quiet Title Removes cloud on title i.e. invalidates any document tending to prove title or lien on property in someone
other than the plaintiff.
B. Newman Machine Corp. v. Newman (you keep threatening to sue, Im gonna beat you to it and remove your title)
1. Facts Newman () owned shares in Newman Machine Co. He sold the companys shares
for $135/share and said it was an unfair price years later. Sellers remorse years later
because the price he sold for was grossly inadequate. He claims he was defrauded by the
buyer (Newman Machine Corp, ). Newman never did anything, just complained. But the
buyer (Newman Machine Corp.) has a problem later selling because of concerns about title
of the personal property (stock), so sues to quiet title.
a. Newman Machine () was suffering because of this cloud over that title because it
was hampering long range planning, there were threats of receivership.
2. Rule can bring a quiet title for real property AND personal property (AND
intellectual property).
a. Requirements for quiet title suits to quiet title may be maintained in equity
where, due to exceptional circumstances, there is no adequate remedy at law.
3. Is this a real controversy? Answer YES. A clod on title is anything that tends to prove
that title in the asset is owned by someone other than the person claiming to have title.
It removes uncertainty by declaring rights.
4. Why quiet title in this case? Answer because ejectment and trespass (which are the
most common equitable remedies in property cases) were not appropriate in this case.
a. Ejectment was essentially a restitutionary action to restore to possession
i. Here was still in possession.
b. Trespass is a damage action.
43

ii. When can quiet title be brought?


1. In nearly all states, can be brought by in or out of title.
2. In some states, in possession determines whether or not theres a right to a jury trial.
b. General Rule regarding Declaratory Relief: If rights were rights at law that are recognized in declaratory
relief then you want jury, if the rights were rights in equity then no jury.
L. Cancellation cancellation is an equitable remedy, and being so, some courts say you have to show there is no
adequate legal remedy at all.
M. -Federal Rule of Civ. Pro. 70 allows for court to appoint someone to sign something that has to be signed to transfer
property for party that is recalcitrant and refuses to transfer
C. Reformation Parties had an actual agreement and the writing doesnt reflect that agreement. The writing is
reformed to reflect the agreement. Whereas, rescission is used when there is no agreement.
a. Reformation is not meant to rewrite the agreement to make it better. It is just about conforming the
writing to what the agreement was between the parties in the first place.
b. Rescission- IF it turns out there wasnt an agreement in all, then the court will try to put the parties
back into the position they would have been before they purported to agree. This is a big difference
from reformation.
D. Hand v. Dayton-Hudson Hand was a lawyer who was fired from Dayton-Hudson. Dayton wanted a release from all
claims that could be brought by Hand. Hand altered the release agreement by excluding age discrimination claims before
he signed it and gave it back to Dayton. Dayton claims Hand defrauded them. Generally, you need mutual mistake for
reformation to be granted. i.e. you need both people to be wrong about what the contract is about. But where one side is
aware of the other sides position and commits fraud, reformation is usually granted. Here, reformation is the correct
remedy because the parties actually reached an agreement, but the contract doesnt document it correctly. Reformation is
where the court interprets what the parties meant and alters the contract. Rescission cancels the contract.
a. Rule general rule is that reformation is entitled when there is a mutual mistake.
a. EXCEPTION when there is a mistake on one side and fraud or inequitable
conduct on the other side, reformation may be decreed.
i. This must be supported by clear and convincing evidence.
ii. Why? The exception is designed primarily to combat the inequities
which naturally result from the fraudulent inducement of an innocent
party to sign a contract the guilty party knew did not reflect the other
partys intent.
b. Holding the contract was reformed to conform to the defrauded partys understanding
c. Remember there was an actual agreement in this case, but one of the other parties lied and that caused one
party appear to manifest assent to terms they didnt actually agree to.
d. Reformation or Rescission BUT NOT BOTH
E. Quo Warranto- this is where court declares that somebody is not suppose to hold public office or a corporate franchise
F. Nominal Damages as Declaratory Remedy- the granting of nominal damages allows a judge or jury to say that you
did have rights that were violated but you werent hurt. This amounts to a Declaratory Remedy.
a. Where were talking about nominal damages sometimes people sue for vindication.like in a defamation case.
If they get nominal damages its a vindication that they were wronged but if might be the case their injury
wasnt that bad so they only get nominal damages.
i. Even if they only get nominal damages the ystill might be able to get punitive damages depending
on whats allowed by law.
ii. Might be able to get attorneys fees, but Supreme Court has determined in federal causes of actions
involving rights violations like civil rights if there are only nominal damages then attorneys fees
should not be awarded.

N. Restitution: The Innocent


Restituion is about a defendant getting something they dont deserve and taking it away from them.
-Restitutional remedies comes under different labels some of those have to do with substantive rights, some of them
are equitable remedies and some are legal remedies.
-Restituion encompasses many causes of actions. It encompasses all kinds of rights that are enforceable by
restitutionary remedies/
-In any situation that a defendant receives a benefit that they shouldnt equitably allowed to keep there is a
restutionary remedy.
A. The previous cases have all dealt with putting the back in his/her rightful position from before the injury. BUT, if the
has been only minimally harmed but the received a windfall, then
a. When do you use restitution?
44

1.

When you have a contract that is not enforceable.


a. E.g.
i. Statute of frauds violation
ii. Illegal contract
iii. Against public policy
iv. Unconscionable contract
b. If a contract is not enforceable, then the contract should be rescinded.
2. When defendants gains exceed s loss.
B. Unjust enrichment (aka Quasi-Contract) is the cause the of action and defendants gains is the measure of recovery.
Where there is an intentional tort, breach of fiduciary duty, or sometimes breach of contract, restitution is often available.
Defendant must be enriched at Plaintiffs expense i.e. defendant used something that belonged to plaintiff to profit.
a. Examples are where people receive money by mistake, parties think there is a contract, but there really is not
one and one party makes partial performance, judgments paid that are subsequently vacated, emergencies,
paying more than ones share in a joint obligation, benefits taken through wrongful acts (such as theft,
conversion, etc.).
b. Two situations where restitution may be inappropriate:
1. Where the defendants wrongful act doesnt benefit the plaintiff
2. Where plaintiffs loss is equal to the defendants gain (because restitution and damages will be the same).
But where defendants gain exceeds plaintiffs loss, recovering defendants profits is most attractive.
C. Difference between restitution/rescission and unjust enrichment
3. Unjust enrichment used in torts law (e.g. infringement, misappropriation of trade
secrets, etc. is seeking ill-gotten gains that the other side has obtained from the illicit
activity.
a. Restitution/rescission rescind the contract and give back whatever everyone put in the contract.
b. Rule if there is a material misrepresentation, the court will NOT reform the contract and instead will
allow the to rescind the contract.
-A Quasi-Contract is NOT a contract. IT is a a situation where a party might get the benefit of goods and services and
decides not to pay for them. The best examples are example quantum meruit
D. Restitution for Unjust Enrichment
E. Blue Cross v. Sauer (receiving $ by mistake)
a. A man has the same name as a Blue Cross member and he begins receiving payments from the insurance
company that are not intended for him. He does not inform the company of their mistake and simply takes
the money. William is jointly and severally liable for the whole money and his father and fathers business
are jointly and severally liable for the extent they got benefits (which is any money William gave to them).
There was no contract and no tort being asserted. Unjust Enrichment is allows for them to get the money they
were entitled that defendant should not equitably retain. In
b. Rule the remedy for unjust enrichment is restitution. A person who pays money to another by mistake is
entitled to restitution from the payee or other beneficiary of the payment. This is true even if the mistaken
payment is due solely to the payors lack of care or inadvertence, or negligent mistakenness.
i. If you are consciously uncertain (not sure this is right but Ill pay them anyway) that you are
sending the check to the right person then voluntary payment defense bars claim.
ii. There is a defense called the voluntary payment purpose saying if you made the payment on purpose
knowing exactly what you were doing then your unjust enrichment claim will not work.
-Exception: If you are the defendant in an unjust enrichment claim, if you are innocent and reasonably relied on the payment
then you wont have to pay.
So there are Two Defenses for unjust enrichment: 1. Reasonable reliance on the payment 2. Voluntary payment
defense
-Other example of way to get property/money back like unjust enrichment is equitable claim of constructive trust.When you
are trying to impose a constructive trust on money you need to identify the money specifically. Like the actual physical money
itself. Without being able to identify specific item you cant have a constructive trust remedy. Unjust Enrichment is a remedy
that doesnt require you to trace like a constructive trust remedy does.
-Quantum meruit and quasi-contract are equitable remedies. These are restutionary remedies. Not all restutionary remedies are
equitable remedies though.
-Where there is a contract, it has effect on the availability of the unjust enrichment remedy. If a person is a party to a contract
they cant sue for recovery of benefits conferred and then ask court to set the benefits at fair market value that is not allowed.
-If the party cant give the benefit back because it got consumed, then you give back the price that was stated in the
contract.
-So unjust enrichment is not a means to rewrite a contract
45

Forced ExhangesF. Innocent Improver- Somerville v. Jacobs (plaintiff mistakenly builds on defendants lot)
a. P builds $17,500 warehouse on Ds $2,000 lot.
b. Rule: to prevent the unjust enrichment of defendants, and to do equity between the parties, this Court holds
that an improver of land owned by another, who through a reasonable mistake of fact and in good faith
erects a building entirely upon the land of the owner, with reasonable belief that such land was owned by the
improver, is entitled to recover the value of the improvements to the landowner and to a lien upon such
property which may be sold to enforce the payment of such a lien or in the alternative, to purchase the land
so improved
c. Nobody in this case was on notice that what was going on was wrong.
d. Difference between encroachment and improvement: An encroachment typically doesnt involve any benefit
to the victim of the encroachment, (it might not be much of a harm either see Highlander foods),
i. Typically what happens in an encroachment case, is that there is some kind of forced transaction and
its valued at some fair market valuation
ii. We typically dont require the removal of an encroachment even though it is a trespass unless it can
be done really cheaply.
iii. With improvements, the choices of remedies are make the landowner buy the improvements or
make the landowner sell the underlying land.
e. The innovation of the Somerville case and the restatement is that the landowner gets to pick which remedy
the plaintiff gets.
f. The rule of this case changes if someone is at fault:
i. If plaintiff is on notice that they are encroaching, then plaintiff on actual notice gets nothing
ii. Just constructive notice will be considered innocence, as long as they didnt act unreasonably.
g. Classic example of restitutionary damages: doctor who treats unconscious patient. Any situation where we
firuge any normal person would say help me if they could, the request is deemed to be made, but we dont
view it as a contract but we view it as a quasi-contract, that is the beneficiary of the help has received a
benefit they, in fairness, ought to pay for.
h. Tenants-in-common: they have property that needs to be fixed or maintained, and the other tenants in
common say not my problem. The tenan-in-common who preserves or adds value is entitled to restitution
from the owner coowners. A third-party that just comes over to maintain the property in the hope they get
paid is different, they should get nothing since they had knowledge.
G. Reversed Money Judgmetns- State v. ANW Seed (consumer protection suit)
a. Motion in Trial Court for Order of Restitution
b. Request: Restore Property or FMV + Loss of Use
c. Order FMV: $57,631.50
i. Court of Appeal Affd Washington Supreme Court Revd
d. Appellate Rule: Restore Property or its Value
e. Form of Restitution
f. Measure of Damage: Sale Proceeds + Interest
g. Sheriffs sales are sales that ordinarily realize less than fair market value.
h. Important to realize here that the judgment creditor here did nothing wrong, but the judgment debtor still
ended having a big loss.
i. On a money judgment, youve had the whole trial and there is an appeal. While there is a risk we will
allocate the risk differently than we do on an injunction bond. So the judgment debtor can insure the
difference in price that they get between what they think their property is worth, and what is actually
recovered by sheriffs sale.
j. If the plaintiff can show that there was a tort in collecting on the judgment (like the sheriff screwing up) then
plaintiff can get what they lossed as opposed to what defendant gained, and plaintiff can get everything back.
k. Cost of judgment bond is appealalbe
H. Remember in Restitution we focus on the benefit given the defendant. Thinking about the value of the benefit, we think of
prejudgment interested as starting to run at the moment the defendant should have become aware of plaintiffs rights. See
page 623
I. The Restatement (Third)doesnt specifically talk about the benefit conferred on the defendant it talks about increase in
defendants net worth. See Page 623
J. Issue: Defendant Benefit > Plaintiff Harm (UNJUST ENRICHMENT)
a. General Rule: recovery limited to Benefit to Defendant
K. Invalid or Unenforceable Contracts-Anderson v. Schwegel (oral contract for the repair of an antique auto)
46

a.

Anderson thinks he is paying $6000 for complete restoration; Schwegel thinks the $6000 is only for the body
work and that complete restoration is actually $9800
i. No meeting of the minds. Their was mutual mistake- the parties said the same term to each other
Restore but meant something different and neither part is at fault for the misunderstanding but no
contract was formed.
ii. One party says they want Quantum Meruit(Schwegel) and the other says the increase in market
value of the improved object(Anderson)
1. Court decides to give the reasonable value of the services done by Schwegel.
b. Trial Court decides: Quantum Meruit $4800 (defendant only paid $5000 so far, owes the plaintiff the
remaining amount for the full work done!)
c. Here, Anderson either requested the services or assented to having them performed for his own benefit.
Thus, the performance rendered may be properly valued as services, regardless of whether the services
actually enhanced the value of the automobile. We conclude that upon these facts, the magistrate did not err
in valuing the benefit conferred as the reasonable value of the services provided
d. Because the Defendant seems to have assented or chosen to allow the Plaintiff to proceed with the work,
Plaintiff must compensated a reasonable value for his services.
L. The Restatment(Third) lists four different measures of benefits conferred other than money: 1. Cost to plaintiff 2. Market
value See page 642
4. The agreed price or 4. The value in advancing the purposes of the defendant
-If the benefit is conferred at persons request or in an emergency then fair market value is recoverable.
-Plaintiff never gets more than her actual cost in restitution.
-An innocent recipient should never be liable for an amount that makes them worse off.
M. Issue: Benefit in form other than Money?
a. Can award:
i. Plaintiffs Cost
ii. Reasonable value of plaintiffs services
iii. FMV of improvement
iv. Agreed-upon price
N. Application of restitution Restituion without Restituion- Farash v. Sykes Datatronics, Inc. NY 1983 pg. 629
a. Facts (Farash) owns a building and claims entered an agreement whereby would lease the building
owned by the . agrees to make some tenant improvements. Lawsuit ensues when makes the
improvements and then the never moves in or leases the property. There was no written contract on the
agreement and statute of frauds requires leases on land over two years to be in writing. Farash argues
promissory estoppel that he reasonably relied on the promise of the Sykes to lease the property to his
detriment. (half of states say if there is a promise that would have to be in writing to support promissory
estoppel you still need a a writing). He then, most importantly, argues restitution. He then argues that he
wants restitution for his reliance interest.
i. sues seeking restitution in the cost of the improvements. Cannot sue for breach of lease because
of the statute of frauds issue.
ii. Rule: if a promisee relies on promisors representations and there is no contract, then
promisee gets to recover fair and reasonable value of partial performance (amount by which
defendant is unjustly enriched) because the defendant will be deemed unjustly enriched by
that amount
iii. Under Farash, even where there is no real benefit to the defendant, plaintiff may sue in
restitution. This isnt really reliance because there is no contract that is relied upon.
b. Holding court says can recover the money for the cost of the improvement in restitution.
i. This seems odd if you are going to undo a contract, how do you do that when the no money was
ever exchanged. Seems like what the court is really doing here is promissory estoppel.

47

1.

Statute of Frauds Exception part performance takes the case out of the statute of
frauds and makes the contract specifically enforceable. Messner Vetere Berger
McNamee Schmetterer Euro RSCF Inc. v. Aegic Group plc NY 1999 pg. 633. Part
performance shows detrimental reliance: The performance must be unambiguously
referable to the contract.
ii. When should a seek restitution and when should a seek damages?
Enforceable contract
Non-enforceable contract
Real benefit to defendant
If enforceable contract and
Sue for restitution; no contract but
benefits from the contract, sue on the benefit to (egg-washer case; there
contract for breach of contract.
was no contract).
What do you sue for? Either
expectation damages or restitution,
depending on the value of each
damage (e.g. if its a losing contract
sue for restitution).
No real benefit to defendant

If there is no benefit to the but


there is an enforceable contract, sue
on contract for breach of contract.
What do you sue for? Probably
expectation damages.

Farash problem. Two choices no


restitution because there is no benefit
(dissent) OR this is unfair and we
will find a way to get you restitution
(fictional benefit deemed to have
been received) and so restitution
must be paid.
-The outcome of this case, is caused by the State of New York refusing to follow through on its own statute of frauds laws.
-The court basically is saying even though they are not allowing promissory estoppel but they are allowing promissory
estoppel anyway.
-Why is this case in the case book?: it is from the highest court in one of the most commercially important states (NY), Its
not alone, if you dont want to call ir promissory estoppel and you want to call it restitution you have to find a benefit
conferred, or you have to call compensating plaintiffs interest restitution (which is a very significant extinction of law of
restitution). Promissory estoppel extends contract damages to places there isnt actually a contract.
-Takeaway: the boundaries of restitution are somewhat fluid. You could use this case to argue for restitution damages when
you dont actually have benefit conferred on the defendant. Argue that you want restitution on reliance interest.

RESTITUTION: MEASURING VALUE OF THE BENEFIT


Disgorging Profts
O. basic principle RECOVERING MORE THAN PLAINTIFF LIST: DISGORGIN THE PROFITS OF
CONSCIOUS WRONGDOERS
Olwell v. Nye & Nissen Co.
a. Facts Olwell had an Eggsact machine, which washes eggs. Under the terms of the agreement, Olwell sells
his interest in the company but keeps the eggsact and stores it. The company takes the eggsact out of storage
and uses it without Olwells consent for three years. Olwell finds out about it and offers to sell the eggsact to
him for half of what he paid, however the company (N&N) offers to give him $50.
i. sues for s profits ($1560 = $10 per week * 156 weeks (3 years))
ii. claims the s conduct of stealing the eggsact was wrongful and the profits earned from the stolen
machine were wrongful. So, says he is entitled to the profits.
b. Rule the theory of unjust enrichment is applicable in such a case. In cases where the (a tortfeasor)
has benefited by his wrong, the may elect to waive the tort and bring an action in contract for
restitution.
c. If had not been able to get disgorgement of profits, what could he have gotten?
i. The machine wasnt in use otherwise, so $0.
ii. Depreciation for use of the machine, so $600.
iii. Rental value, $10 a month for 36 months, so $360.
d. Rule Wrongful conduct opens up the possibility of unjust enrichment for defendants conduct.
i. Wrongful acts
1. Receiving money by mistake
2. Theft
48

3. Conversion
4. Breach of fiduciary duty
5. Misappropriation of trade secrets
ii. 3 scenarios of wrongful acts
1. Defendants wrongful act does NOT benefit the defendant at all.
a. Damages to the are larger than s unjust enrichment, so will sue for damages.
2. Defendants gain exceeds the s loss
a. Damages to the are less than s unjust enrichment, so will sue for unjust
enrichment.
3. Defendants gains are equal to s losses
a. Unclear; look to see which is better for the .
e. Ways of Looking at this Case: 1. Looking at it from the standpoint of plaintiffs damages from them taking
the egg washer(conversion).. the remedies in conversion are that youre entitled to get your thing back, and if
you cant get the thing back youre entitled to the value of the thing. (olwell can get the machine back), and
youre entitled to the market value of the use. If there is a tort measure of damages that is superior, the
plaintiff will choose the tort measure. So if the conversion measure of damages was superior he would have
proposed it, but he didnt in this case. In this case plaintiff recovers more than what he lost.
f. Why do we overcompensate plaintiffs in this situation?- We are impart looking at whats fair, we are looking
at the way these kinds of awards affect peoples behavior and we want to discourage inefficient behavior and
encourage efficient behavior. In these kinds of cases were trying to disincentivize people from wilfully
taking stuff that doesnt belong to them.
g. Very often wrongdoer doesnt profit they just hurt someone.
h. Sometimes what the defendant gets as a result of the wrong is the mirror image of what the plaintiff lsot from
the wrong.
i. This case is an example of a defendant being much better off than the plaintiff is worse off.ex. Olwell
avoided having to avoid renting an eggwashing machine
j. Waiving the Tort, and Suing in Assumpsit p. 657- in stead of bringing tort claim for tort damages, you
could sue on quasi-contract theory that defendant did something wrong and it would be unfair for him to
keep the benefit.
i. Burk says this isnt really about waiving the tort but choosing an alternative remedy, e.g.
disgorgement of profits vs. tort damages.
ii. Definition: assumpsit is just another term for quasi-contract
k. Restitutionary remedies that are not claims in contract or claims in contracts but having to do with assumpsit
or quasi-contract are restituionary remedies.
l. Profit Measure of Damage is typically used in contexts where the benefit that the defendant got is greater
than the value of whatever the defendant got.
m. There are various kinds of statutory remedies that specifically say you may recover defendants profts,
particularly in intellectual property.
n. Profits also used in cases where defendant makes more than the defendant takes
o. Profits also used in defaulting fiduciary best example is fiduciary taking stock and selling it. Fidcuaries have
higher duty than normal person. So even if fiduciary doesnt intentionally engage in self-dealing and gains
profits from something they take, they have to return the thing and return profits even without intentional
wrongdoing. Or if fiduciary takes stock and value goes down and they sell it, they have to return the whole
value of the stock they took.
p. Intermediate Fault- case where defendant is not consciously wrongdoing, but is not totally innocent either.
Like your only negligent, but your more negligent than the plaintiff.
i. Another example of innocent or negligent misrepresentation by defendants- in those situations you
probably dont get defendants profits, but you can play around with the measure of restitution to
accommodate benefit that defendant got.
P. basic principle Edwards v. Lees Administrator (CAVE CASE)
a. Facts: Edwards discovered the entrance to a cave on his land and developed it as a tourist attraction. Onethird of the cave was under Lees land.
b. Holding the court granted restitution of the profits Edwards earned from showing Lees part of the cave.
Q. when restitution? when damages? Becks v. Northern Natural Gas Co.
a. Facts natural gas stored in an underground formation owned by . The gas leaks into a second formation
also owned by the but not included in the lease. Eventually the gas in the second formation is uncovered.
sued in restitution.

49

b. Holding restitution was not appropriate and the could only get damages. sued for the profits but the
court said the was only entitled to damages, which was the rental value of the property illegitimately used
by the (gas company).
c. Rule: Culpability of defendant is a major factor: A negligent defendant will usually only pay rental
value and an intentional defendant will usually have to pay profits. Look at intentionality of the
defendant to decide defendants profits or rental value.
d. Whats the difference in these cases? Becks compared to Edwards
i. Answer look at the s conduct
1. If the conduct is bad conduct, can get restitution.
2. If the conduct is just negligent or good faith conduct, can
only get damages.
R. Maier Brewing v. Fleischmann Distilling Corp (trademark infringement of whiskey brand name for a cheap beer)
a. Plaintiffs made Black and White label whiskey. Maier began distributing beer with Black and White label
through Ralphs. Trial Court ordered that they couldnt make or sell anymore cheap beer with the Black and
White label. In addition, trial court gave defendants profits to plaintiff. Rental value in this case would be
the cost of renting the trademark. Plaintiff didnt lose any sells from the defendants using the Black and
White label. Alternative reading of the statute that the court makes here is that plaintiff can recover profits
from both the defendants, the beer makers and the retailer.
b. Issue Apportioning Profits: Is direct competition necessary to get the defendants profits from trademark
infringement?
c. Rule: Majority rule is that if companies are not competing, then plaintiff cant get defendants profits.
Minority view is that direct competition is not required in order for the plaintiff to recover defendants
profits. Ninth Circuit followed minority rule that the plaintiff may recover defendants profits.Remember
this is restitution not damages
d. Terminological point- once we determine there is a restituionary remedy available , we say the defendant
ahs to account for his profits, this is because the Defendant doesnt have to pay everything they made, they
are entitled to a credit for what they spent to get the profit.. There is a separate equitable remedy called an
accounting. That is a completely separate thing from this situation
S. Causation and Allocation-Sheldon v. Metro-Goldwyn Pictures Co (courts will apportion profits for copyright
infringement)
a. Facts: A script was stolen for a play that MGM later made into a movie. DC awarded 100% of the profits to
the P. Appeals reversed and awarded only 20% of the profits because reputation of movie maker, star quality
of actors, etc. accounted for 80% of the profits. Only 20% of the profits were attributable to the script.
Experts looked at what other scripts sell for, etc. Here, plaintiffs could also get market value of the script or
the rental value to hire script writers to write a script for you. In situations where it is difficult to allocate
different portions of the profits to plaintiff or defendant actions, the plaintiff gets 100% of the profits.
b. Holding: When measuring defendants profits, mathematical exactness is not required, but reasonable
approximation through expert witness testimony to illicit a rational separation of net profits between
plaintiff and defendant.
c. Rule: A court may apportion profits in a suit for copyright infringement based upon the actual use of the
copyrighted material in the production of revenue.
i. Suppose Leonardo painted the Mona Lisa with stolen paint. Plaintiff would argue, but for the paint,
the painting wouldnt have been created. Leonardo would argue that it was his skill that led to the
success of the painting.
d. Profits have to be reasonably approximated and not exact.
e. Lots of room for advocacy in these situations. Ex. Someone steals poem and sets it to music thats original.
How much of the profit is the music and how much is the words? indeterminate, arguable.
f. The message from this case is you do the best you can estimating damages, but it has to be a reasonable
approximation of the actual damages. If the defendant argues that its too hard to separate out the damages
they owe, they are not allowed to make that argument.
T. Three Boys Music Corp. v. Bolton (more APPORTIONING of profits for Restitution restoring profits to rightful owner)
a. Awarded $5.4 million against infringers who appear to have unconsciously borrowed from a song Love is a
wonderful thing (a minor hit by the Isley bros) they heard as teenagers a century before.
b. Jury found that Boltons song accounted for 28% of the profits from his CD and that the infringing elements
accounted for 66% of the profits from the song. 66% of the 28% of the profits from the CD, which came to
$5.4 million.
i. Ct often uses Sheldons methodology in music infringement cases to allocate profits btwn infringing
song and others on the same album or CD, and between the music and the words of the infringing
song.
50

ii. Held: Plaintiffs entitled to restitution through apportionment of profits, in the amount that their
song contributed to Boltons sales
c. Notes on Apportioning Profits
i. Ct determines the gross receipts from infringing sales and subtracts the expenses of production of
those sales
ii. P must prove only gross receipts, D has burden of proving its expenses
iii. Bought and paid for stdinfringers pay for labor and get to deduct salaries. This std is in tension
with the rationale that P should not have to pay for work done by infringers.
U. Deductions and Credits: Hamil America Inc v. GFI (Rule: Lost profits = sales costs)(case is about apportioning
overhead costs used to infringe)
a. Facts: Hamil sued GFI for copyright infringement because it copied a floral fabric pattern. GFI had SGS
manufacture it and then JC Penny sold it. Lower court said it was an exact copy. Hamil wanted lost profits
(restitutionary damages). Lost profits = sales costs. This case has to do with the costs and specifically how
much defendant can include in the costs.
b. GFI claimed actual costs of production of the infringing pattern as well as its general (fixed) overhead
expenses. DC rejected general overhead expenses. Overhead are costs you would have whether you were
doing the bad activity or not, yet doing them is essential to being able to do the bad activity.
c. All the defendants here in copyright infringement because they were engaging in unlawful distribution of the
copyrighted work.
d. Court says you are allowed to allocate a portion of your overhead.
e. Here the more the defendant can allocate to overhead the less they would have to pay in damages.
f. This is another area where advocacy can make an enormous difference.
g. 1. If you see overhead that doesnt assist in the production of the infringer you cant deduct it 2. If you see
overhead that does assist in the production of the infringement you need to allocate it.
h. You only have to have a rational means of categorizing the overhead expenses in order to allocate them.
i. You could argue salles of infringing item as a percentage of total sales is reasonable allocation of overhead,
or you could contrastingly argue that the production costs of the infringing production over total costs is
appropriate measure.
j. RULE: Appeals said COSTS should be allowed as long as they pass a two step test:
1) Determine what costs are associated with production of the infringing product
2) Come up with a fair, accurate, and practical method of allocating overhead to the
infringement. Appeals court allowed GFI to include general overhead expenses. Where the
violation is willful, the court will apply the two part test more vigorously, but will not deny
overhead costs outright. Where defendants profits are too speculative (such as enhanced goodwill),
they will not be awarded.
-Defendant will argue that those overhead expenses were attributable to the wrong activity
-Plaintiff will argue that very few of those overhead expenses are attributable to the wrong acitivty
-have to figure out how to apportion fixed costs to those parts that are attributable to infringing
activity.
-Overhead activities are general fixed-expenses of doing business that are valuable and supporting
of the infringing activity but are not exclusive to it.
-Similarly here you need a reasonable approximation of expenses
-conscious wrongdoers disgorge profits, innocent infringers pay usually by the market value of what
they got.
- Worth paying attention to in this case: Hamil complains that aside from the disagreement about
how to measure the profits, they also lost sales and they are entitled to loss sales. Court makes two
points: Sometimes your loss sales directly corresponds with sales defendant made and plaintiff has
to make choice do you want your lost sales or profits defendant made. Cant have both.
-GFI argued below that Hamil America should not recover lost profits, because the shared customers
would not have purchased the fabric at Hamil Americas above-market prices. The court noted that
Hamil could not recover both for its hypothetical sales to the shared customers and for GFIs actual
sales to those same customers. The court elected to measure GFIs actual profits from sales to the
shared customers, rather than speculate as to what Hamil might have earned had it sold to the shared
customers.
-Rule: In the absence of convincing evidence as to the volume of sales that plaintiff would
have obtained but for the infringement, the measure of lost profits may be rejected as too
speculative.
-Circuit Split:In Intellectual Property cases that authorize recovery of profits there is a split in the
circuits about how you allocate overhead:
51

Second Circuit: says you can always allocate overhead even with willful infringer
Ninth Circuit- says that overhead is deductible in calculating profits where the infringement was not
willful, conscious or deliberate.
Restatement (third)- says defendant will not be allowed to deduct expenses (such as ordinary
overhead) that would have been incurred in any event, if the result would be that defendants
wrongful activities-by defraying a portion of overall expenses-yield an increased profit from
defendants operations as a whole.
Disgorging The Profits from Opportunistic Breach
Snepp v. United States (CIA book sold behind the CIAs back, Breach of Duty = wrongdoer, therefore unjust enrichment
k. Facts Snepp worked for the CIA. He later wrote a book about his experiences, though he signed an
agreement with the CIA where he pledged not to divulge classified information and not publish any
information without prepublication approval. The CIA sues for breach of contract, though it admits (1)
didnt reveal classified information and (2) had a right to publish a book.
l. Government sought:
i. Injunction preventing publication without first obtaining prepublication approval.
ii. Profits earned from the book to be put in a constructive trust for the government.
1. Why not sue for damages? Answer no quantifiable damages to the government from the
booknot like the government was going to write a book and lost profits from s book.
iii. Government wants constructive trust on all of the proceeds of the book. The 4th circuit denies the
constructive trust since he didnt disclose any classified information. All the proceeds from the book
are from information he was entitled to use.
1. The supreme Court reverses the 4th circuit: says there are two duties involved 1. Duty to
submit to prepublication review, 2. Duty not to disclose classified information.
iv. The Court says, the breach of a contract like this usually doesnt produce damages are recoverable.
Because it is difficult to show what information is classified because the government isnt going to
say what is classified.
v. Where we are considered about bad behavior that is obviously wrong but the injury is hard to
determine, we have alternative methods to deter that behavior, in this case constructive trust.
vi. The defendant argues that the contract doesnt provide for the constructive trust remedy, and the
government has other means of punishing disclosure of classified info like criminal penalties. Court
denies those
vii. This case isnt analogous to someone wrongfully taking something and profiting from his wrong.
Snepp did something wrongful(not submitting to prepublication review) but he didnt profit from
the wrong.
viii. The majority said if you dont impose a sanction that deprives actor of the benefit of the bad acts
then it will be very difficult to deter this act.
m. Holding majority took the profits from the book and put them in a constructive trust for the government.
i. Majority saw constructive trust as a proper deterrent on this sort of future behavior.
ii. Breach of duty makes him a wrongdoer therefore, unjust enrichment (disgorgement) is
appropriate.
n. Dissent- says that tort law in its current form already contains a remedy. If you want to punish somebody for
committing a tort there is a means of doing that.
o. Generally speaking: when you have behavior that seems to be bad and you want to discourage that hasnt
created a benefit for the defendant or an idenfitiable loss for plaintiff it is hard to figure out what to do to
deter people
p. We agree that Snepp had more than a commercial contractual agreement. This is analogous to a fiduciary
relationship, and it is like a breach of a fiduciary duty which is a tort. In breaching a fiduciary duty,
disgorgement of profits can be granted regardless of intent to commit wrong.
V. May v. Muroff
a. Defendant sells Real Estate to Plaintiff. While the deal is in closing, the Defendant sells Fill from the Real
Property for $240,000. Court says: Deliberate breach Plaintiff entitled to the $240,000
b. Rule: if a deliberate breach of contract results in profit to the defaulting promisor and the available damage
remedy affords inadequate protection to the promissees contractual entitlement, the promisee has claim to
restitution of the profit realized by the promisor as a result of the breach
c. The seller's breach here was deliberate and he should not be permitted to profit by his own wrong and enjoy a
windfall profit of $117,933. The purchaser, under the facts and circumstances of this case, is entitled to the
fruits of this wrongfully received windfall. See Laurin v. DeCarolis Construction Co., 372 Mass. 688, 363
52

N.E.2d 675 (1977). The purchaser should be entitled to the value of the materials removed which is readily
determined from the record at $240,000.[1]
d. One way you can measure damages is the diminution of the market value,
e. Second way is the market value of the dirt he took from the land that he promised to give to plaintiff.
f. Third possible measure: is the cost of restoration which is specific performance remedy
g. Both and opportunistic breach and efficient breach are breaches they are encouraged to commit because they
can make more money by breaching.
h. Court could have ordered specific performance since the traditional rule is that conveyances of land by
contract can be ordered by specific performance because land is unique. The court decided not to order
restoration of the land because the cost of specific performance is grossly disproportionate to the diminution
in market value.
i. This is not a very common situation. This is because most breaches of contract are not profitable. Usually,
plaintiffs loss is commensurate with defendants gain. In this case they were not.
Rescission
-in a situation in which two parties are contracting to exchange services of some kind, sometimes it turns out that the
contract is not enforceable. Other times, even though the contract is enforceable its performance is interrupted. In both of
these kinds of situation, circumstances may have changed or parties may have changed position because the parties may
have started to perform. Circumstances might have changed that the value of the deal is different than it was earlier. In the
situation where it has been partially performed, material breach is one of the things that can excuse performance by the
other side and that gives rise to either expectation damages OR Rescission(rather than what they what they would have
gotten had the contract been performed, they want to be put back in the position they were in before the contract.)
W. Mobil v. US (contract becomes unworkable after NC changes its law to prohibit oil exploration)
a.
b.
c.
d.
e.
f.

g.
h.

i.
j.

k.
l.

This case involved a material breach. Because hes materially breached before the contract is done the
plaintiff wants rescission. He wants everyone in the position as if the agreement had never happened.
Mobil pays $156 million to US for right to explore oil off the coast of NC
This agreement becomes VOID after NC changes its laws to prohibit this type of exploration
Court said: company does not seek damages for Breach of K, but rather seeks restitution of their initial
payments which resulted in nothing
Mobil wants rescission and restitution. Mobil had started to perform after they had paid the $156 million and
they want to undue the lease from the government
Govt says even if they hadnt interfered with the federal approvals Mobil wasnt harmed because Mobil
wouldnt have gotten approval from the state of NC anyway. So the government says the money they spent
on starting to drill wouldnt be recoverable anyway(expectancy damages) since Mobil wouldnt have gotten
approval from north Carolina. Mobil agrees and says there expectancy damages are zero which is why they
want rescission (return of the $156 million they paid for the lease).
Court concludes that Mobil had reasonable expectation to believe that it would have gotten the right permits
from North Carolina. By the time we reach the point of rescission we know ex post that is was a bad bet on
Mobils part that they would get the permits (which they ended up not getting).
Rule: the law entitles companies to restitution whether the contracts would or would not ultimately have
produced a financial gain
i. Court compares to the Lottery:
1. If a lottery operator fails to deliver a purchased ticket, the purchaser can get his money
back whether or not he had a winning ticket.
Remember if there is a material breach of contract plaintiff can choose rescission or normal expectancy
based damages
This is a weird and unusual case. The plaintiff gets to choose the rescissionary remedy for breach even if it
turns out the deal would have been a bad deal. Mobil says exactly because the government shouldnt have
breached. In a situation where a party has made a bad deal typically that means it was a good deal for the
other party. In a change in circumstances situation, the party that is better off typically is not the party that
breaches. In this case the government was the one that was made better off (+$156 million) by the change in
circumstances but it was the one who breached.
Other benefits of rescission: eliminates risk of error in measuring damages by expectation measure,
Having the option of rescission means you msut have a material breach.
i. Exceptions

53

1.

In a contract forced sale on credit where the buyer doesnt pay(unsured transaction). He has
to go to court to get a judgment to reposses the car. He cant get rescission, because that
would turn every unsecured transaction into a secured transaction.
2. If you rescind part of a contract it has to be specified as a severable part. So you can only
either rescind entire contract or a specific severable part. You cant pick and choose which
parts of the contact you want to rescind
m. IF its an equitable remedy you have to show that the failure to give the equitable remedy will cause
irreparable harm and there is no other adequate remedy. You also dont get a jury trial with equitable remedie
actions.
i. So figuring out whether the remedy you get with the rescission is legal or equitable is important to
figure out what you will get and whether you can get it.
n. When do you get to choose rescission?
i. Generally speaking, you almost can always plead your remedy in the alternative. At some point, e.g,
some courts say. before trial starts, (some jurisdictions say when the jury is charged at the end of
evidence). You can change your mind, like you could choose rescission early on and the choose
damages later, UNLESS defendant has changed position in reasonable reliance on what you picked.
ii. There is a space where you can claim rescission even if you keep performing for a while.
iii. The plaintiff cant play a waiting game to see how the bets are going to play out. If there is
something siginifant that changes things, then plaintiff cant wait to see how things will change
before it picks rescission or damages.
o. What if you have a contract that goes bad where the benefits cant be returned? i.e. example of painted
house p. 692
i. In a situation where one party has substantially performed in a way that is not returnable. Ex.
Boomer v. Muir see below. Non-breaching party wants fair value of the service they provided before
completion of the contract. Where there is a big disparity between the value of the contract price and
the value of the services provided, where contract is almost completely, but not entirely, performed.
We look at the contract as strong indication of what the benefit is.
1. If the contract is fully performed, we never say well look at anything other than the agreed
contract price. But if its 95% and there is a claimed material breach the court will consider
looking at something other than the contract price.
a. if defendant argue that plaintiff s trying to renegotiate contract.
b. If plaintiff argue that defendants breach made it much more expensive to perform
than was anticipated when the contract was made.
Restituionary Rights in Specific Property
Constructive Trust Paolini v. Goldstein
1) Constructive trust is a situation where defendant has a specific pice of property. What the court can do is to say, is that we
are going to pretend you received the property consentually, you will be deemed to hold the property in trust for the benefit
of the person it belongs, to and the court will require you to return it.
a) If you made it money from the item, you will have to account for them and return the item
b) If theres any rent money collected from the defendants crony, then that money must be returned. If he wasnt
charging rent, then there is a breach of trust and the plaintiffs can get damages for them.
2) Even if other people execute judgment against the defendant before, constructive trust gives you priority over it, since the
legal fiction is that it belonged to you the whole time
3) The essential thing you have to show to get a constructive trust: that the item youre trying to impose the constructive trust
on is something that is specifically yours.
a) IT is hard to trace money because it is fungible. That means its harder to show that a money is specifically yours.
Reach of the Remedy- Ruffin v. Ruffin
-The plaintiff loses in this case because they cant trace the money to show specifically that the money that was owed
to her was used to purchase the lottery ticket. If he had broke into her house stole $20 and used that $20 bill to purchase the
lottery ticket, then that what would be different since the money is directly traceable to her and she can show that the lottery
ticket was purchased specifically with her money. To have constructive trust on money, you have to have more than unsecured
creditors claim, you cant just show it was amount of money owed to you, but that you had a direct property interest in the
thing the defendant inequitably possesses, or that he exchanged that property for something.
-If your owed money what you are typically suppose to do is get judgment against them, have sheriff go collect defendants
property under writ of execution. She didnt do any of that in this case.
-Could also easily trace, if he steals $20 bill from plaintiff and deposits it in back account where it is the only money in there.
Tracing:
i) Tracing
(1) Defining the Pool- Erie Trust
54

(2)
(3)
(4)
(5)

(a) This case asks the question: when were talking about a fungible asset like money and it gets dropped
into a pool, how do we define what the pool is?
The general rule is that the pool is defined to be the bank account or bank accounts that the plaintiffs money
has flowed into.
Erie says all deposit accounts are one fund
The more usual rule
(a) Lowest Intermediate Balance Rule
Applying the Rule- JD Services see- casenotes
(i) Takeaway from case is to do the best you can with the information you have for tracing
(b) When you put money in a deposit account its not really literally yours, though we treat it as such for
tracing purposes when imposing a constructive trust.
(i) The two kinds of money that your gets mixed with in deposit accounts is the bad guys money and
other third parties money that the bad guy ripped off.
(ii) IF defendant puts his own money into account its considered his money.
1. If the defendant puts money into his account for the purpose of replenishing plaintiffs money
that he thinks he spent then its plaintiffs money.
2. When the defendant takes money out for the purposes of benefitting himself, we say the
defendants money comes out first.
a. If the defendant takes the out more than just his money, so the plaintiffs amount in the
account is lower, but then the defendant deposits money, then the plaintiff will still only
have the lower amount disregarding the defendants later deposit. Any deficiency the
plaintiff can go get a judgment against him to collect on the deficiency that he owes him.
But the constructive trust
(iii) So defendants is considered to have come out first and then plaintiffs money, but any of
defendants money that comes in later is considered defendants money.
(iv) EXCEPTION: if defendant takes out money for the purpose of making more money, like an
investment, then that money he took out plus any profits he makes will be considered the plaintiffs
in proportion in the amount the plaintiffs money was part of the account. If the investment goes
down in value the stock will be attributed to be the defendants.
(v) Two Problems With Intermediate Balance- At the time JD Services was made, banks did not
have the ability to determine when deposits were made. Second, when youre dealing with a
defendant thats insolvent and they owe a bunch of other parties money, what happens when the
plaintiffs money is commingled with other third parties money and they all have the same kind of
constructive trust that you do.
1. Some jurisdictions say, if the defendant is insolvent the first thing you do is apply a pro-rata
claim on the whole money that is not the plaintiffs. If the money that is not the defendants is
invested and it makes money and nobody else is owed money, then the principal is distributed
pro-rata to the people with the constructive trust claims, then the profit is distributed to the
other people owed money.
a. If nobody else is owed money then the profit is distributed pro-rata among the people with
constructive trust claims.
(vi) Steps of tracing money into account
1. Define the pool (three rules of tracing: old rule: swelling of assets, Erie rule(minority)-take all
sources of liquidity that defendant has including cash and accounts, Modern Majority Rule: you
define the pool account by account. You can trace money into a particular account or other
holding arrangement and thats the pool you use. Ex. Defendant has 10 bank accounts and he
puts your money into one. Under Erie all accounts would be part of the pool, under the modern
majority rule, only the account he put money into is the pool).

X. Who gets to choose rescission?


5. Answer the plaintiff. Plaintiff can choose rescission because of its simplicity OR
because of personal preferences not reflected in market value OR because s/he has lost
confidence in defendant and the transaction.
a. Bush v. Canfield gets to choose rescission/restitution or damages 1818
pg. 624 In a losing contract, buyer () had a contract to buy 2000 barrels of flour
at $7 a barrel. While the price dropped to $5.50 (meaning the buyer was
overpaying and was going to lose $3000 on the contract), it was the seller () who
failed to perform. The seller sued for rescission and restitution of his $500 down
55

Y.
Z.

A.

B.

C.

payment. The seller () argued it should get to keep the $3000 profit even though
it delivered nothing.
a. Holding buyer was entitled to rescission and restitution of his $500 down payment.
What is interesting about rescission is when it is impossible to undue the contract and put things back to the way they
were. In that situation: We have, within reason, we have the non-breaching party pay for any benefits that cant be returned
in kind. The hard part is measuring the part of that benefit.
``
Cap on Restitution?
a. Restitution Section 38: a plaintiff entitled to a remedy for total breach of contract may prove and recover, as
one alternative measure of damages, the value to the defendant of the plaintiffs uncompensated contractual
performance, not exceeding the price of such performance as determined by reference to the parties
agreement
Mutual Benefit Life Insurance Co. v. JMR Electronics Corp - JMR lied on insurance application that he didnt smoke.
He died. Any misrepresentation in the contract would void the contract, so Mutual sued for rescission. Mutual wants a
release that says it does not to have to pay 250K and, in turn, it will pay back the premiums paid over the years plus interest.
JMR wants to pay the higher smoking rate and still keep the policy. This is rescission and restitution because k is rescinded
by way of both parties giving back what they got from the k. Court orders rescission because the misrepresentation is
material to the policy. This puts the plaintiff in the rightful position because if JMR had told the truth in the beginning, the
company might never have insured him (probably would for a higher premium). Since no one knows what might have
been, just rescind the contract.
a. Where there is: mutual mistake of fact, unilateral mistake not relied on, duress, substantial breach of
contract, or fraud, there can be rescission, but not always. The grounds for rescission must be substantial.
Cherry v. Crispin ($1500 termite fix = material breach!?) is this really substantial? (real property make a diff?)
a. Sellers hid the fact that their house had termites. It would cost $1500 to get rid of them. Plaintiffs sue for
rescission and got all their money back and defendants got the house back + rent for the time plaintiffs lived in
the house + minor improvements. In Mutual, the insured gets interest based on the same theory: the insurer must
pay for the benefit of having the use of the money.
b. IF buyer () seeks rescission, the s get the house back, the s owe the s money for rent (the time they stayed
in the home), and the s owe the s money for repairs that were done to the house when s moved out.
c. Termites = material breach, therefore the plaintiff can rescind the contract
Can restitution exceed expectation?
6. Answer yes Boomer v. Muir Cal. Ct. App. 1933 Boomer was a subcontractor and
Muir the contractor. Muir materially breached the contract by failing to provide adequate
supplies. Boomer would have lost $267,000 on the project given how much he had spent,
but the court let him sue for rescission and restitution (he was 95% done with the project
when Muir breached).
a. Holding was able to sue for the value of the nearly completed dam measured
by the reasonable cost of building it. So, he was able to recover $258,000, the
difference between what he had spent and what he had already been paid
($571,000 $313,000 = $258,000).
b. Rule a seeking restitution rescinds the contract and sues in quantum meruit,
the common count for the value of services rendered.
i. is not able to rely on the contract price because the contract has been
rescinded.
c. EXCEPTION if the has fully performed and defendant owes the money,
paintiff is limited to the contract price and CANNOT recover restitution.
i. Restitution is therefore limited in these cases to partially performed
contracts, not fully performed contracts.
d. The Boomer rule applies to contracts with the US as well (as seen in
Glendale) Acme Process Equipment Co. v. United States Ct. Cl. 1966 p. 650.
7. Restatement approach (different nomenclature, same result) Restatement 344,345,
371, 373 rejects this fiction and acknowledges: restitution of the value of the part
performance as an alternative remedy to contract remedies.
8. Courts opposed to the rule in Boomer can accept the rule but reject the numbers those
courts can accept the contract price as admissible evidence on the value of the benefit
bestowed by the , so the cannot recover more than that (and it functionally is an
expectation damages case where the loses if it is a losing contract).

O. Replevin And Ejectment


A. Replevin and ejectment are ancient writs for the recovery of property
56

ii.

iii.

iv.

v.

1. Replevin for personal property.


2. Ejectment for real estate.
Examples
1. Without court involvement Under the UCC, the lender becomes the secured part, the
borrower becomes the debtor, and the goods become collateral. If the debtor fails to pay,
the secured party can repossess without going to court, ONLY IF she is physically able
to do without a breach of the peace.
a. E.g. cars parked on the street are repossessed all the time.
2. Going to court more common, the secured party sues to replevy the collateral. If
collateral is inside the debtors house/factory/office, and the debtor is unwilling to
turn it over, the secured party cannot seize the collateral without breach of peace.
Replevin Brook v. James A Cullimore & Co. OK 1967 pg. 374
1. Facts Brook borrowed $8000 and gave items of personal property to secure the loan.
Ultimately, Brook defaults, breaching his promissory note, but his secured item was only
worth $2500. sued brook for replevin because the collateral is personal property.
a. Brooks wanted to give the $2500 rather than the personal property which was
worth less than Brooks valued the property for.
2. Rule replevin is an action to recover the personal property rather than the monetary
value of the personal property.
a. Choosing between property and value of property gets to choose between
the personal property and the monetary value of the personal property (when
the monetary value of the property is available).
3. How does this differ from an injunction?
a. Injunction must show irreparable injury.
i. If fail to follow courts injunction, is in contempt and can be thrown in
jail.
b. Replevin no requirement to show irreparable injury
i. If fail to follow replevin, is not in jail; instead, the is entitled to
TAKE the property.
ii. Replevin lies ONLY to recover goods; it does not lie to prevent a
threatened destruction or dispossession.
Ejectment Welch v. Kosasky MASS APP 1987 pg. 713
1. Facts action for the value of the property after the sale. was robbed of silver goods
($40,000 in value). Physician bought them for cheap. Court assumed he knew or should
have known) the goods were stolen:
a. Turns out that the silver actually increased in value while it was stolen.
2. Rule wrongdoer shouldnt profit; gets to decide if wants replevin or trover
(monetary value of the goods).
a. In conversion cases, the rule of damages is based on the value at the time of
conversation, but (1) the converter is credited with the value of the returned goods
at the time of their return and (2) charged with damages for loss of use of the
goods during the period of detention.
3. Application here, the court allows the to get the appreciated value ($25,000) minus the
value at time the goods are returned ($3000 because of alternations to caster), so $22,000.
forcible entry and retainer
1. Forcible entry damage action for forcibly entering land and dispossessing the occupant.
Even the true owner can be liable for forcible entry because the emphasis is on the use of
private force and the associated risk of violence.
2. Forcible detainer is the summary action for possession. It can be used only against
defendants who have no colorable claim of title.

P. Enforcing The Judgment: Contempt


Q. Contempt is the means by which you enforce an equitable remedy. If you enforce a legal remedy you use replevin in which
you you get a writ of execution to get a sheriff to go levy.
A. CONTEMPT POWER (balancing necessity versus arbitrariness)
4. Two Categories:
a. Direct: In the courtroom, in the presence of the judge.
b. Indirect: Outside the courtroom.
57

5.

Two Types:
a. Criminal
i.
ii.
iii.
iv.
v.
vi.

vii.

b.

viii.
Civil
i.
ii.

Punishes for violations of injunctions.


In order to convict, there must be constitutional protections such as jury trial and proof beyond a
reasonable doubt.
Criminal contempt is prosecuted in the name of the sovereign; the civil plaintiff is in the position of
a complaining witness.
Only a willful violation can be prosecuted under criminal contempt
The sanction is criminal.
Criminal contempt is subject to the collateral bar rule. Collateral bar rule says you can not defend a
charge of criminal contempt by arguing that the injunction was wrong. So you can not defend it by
violating it and then defending it in the criminal proceeding. The way you challenge the injunction
is by appealing it not by violating it.
Criminal contempt is a crime and is is given regular criminal protections like right to counsel, due
process, and right to jury trial for crimes involving more than 6 motnhs in jail and substantial
fines, all though it is not clear what substantial means in order to trigger a jury. Example note 2.b. p.
766.
Fines are payable to the government.

Purpose is remedial and not retributive i.e. to get someone to comply with a court order
Must be avoidable by doing what the court asks you to Key to the jail is in your pocket. When
you comply, punishment ends.
iii.
Dont need jury trial
iv.
Usually filed in the name of the plaintiff
v.
Not subject to collateral bar rule.
vi.
Civil contempt can be predicated on completely inadvertent violation of injunction. So you dont
even have to be negligent to be liable, all though the sanction may be
vii.
Procedurally the Bill of Rigths is not relevant. You have to follow due process and give defendants
opportunity to be heard but no juries required.
6. To bring civil contempt proceeding, has to file motion to show cause, and will argue that defendant violated injunction
and the court will order the defendant to show up to a proceeding to argue that he didnt violate the injunction or that
he was justified in doing so
7. You can have both civil and compensatory remedies in the same hearing.
8. In civil contempt the question of whether you violated the court order Is clear and convincing proof, and the standard
for proving damages is preponderance of the evidence.
9. Two Types of Civil Contempt
a. Compensatory Civil Compensates plaintiff for a defendants violation of court order if the plaintiff suffers
losses (damages are measured against the position the plaintiff would have been in had the defendant not been in
contempt of the injunction).. Attorneys fees are generally awarded. California doesnt have this type. Defendant
can be disgorged of any benefit they received from violating the order (restitution).
i. These are contradictory because the whole point of getting the injunction in
the first place that was violated, was that it had to be shown that damages
were inadequate and now they are asking for damages for the violation of the
injunction.
b. Coercive Civil Purpose is to force compliance rather than punish for past non-compliance.
i. Coercive civil contempt is a conditional remedy for contempt.
ii. Idenfies a specific thing for the plaintiff to do or stop doing in order to stop
the penalty, like fines or imprisonment.
iii. Defendants wealth is relevant to the amount of the fine. Set a fine amount
necessary to coerce the defendant
iv. Coercive fines are payable to the government
10. Remedies for Contempt
a. Criminal
i.
Jail or fine imposed for past disobedience nothing that D can do to get out other than by serving the
term, cannot get out earlier by compliance
ii.
Fines payable to the government, otherwise go to jail
b. Coercive Civil

58

i.

Court may take person who has violated court order and confine them until they comply or pay a fine
until they comply (i.e. each day there is a fine). This is called purging the contempt because the
contempt may be purged by complying with the court order.
11. Defenses to Contempt
a. Contemptuous conduct did not occur
b. Court order was vague
c. Inability to comply
d. Excuse or Justification
e. Order is unconstitutional or unlawful
B. International Union v. Bagwell
a. Union went on strike and obstructed the road and blocked access to the road. Bagwell gets the injunction,
but Union violates it. Court finds them in contempt, finds 72 violations and orders $642,000 fine. Second,
court orders union to pay $100K per future violent breach of injury and 20K for non-violent breach. Court
orders union to pay 64 million for additional violations, $12 million to Bagwell and $54 million to the state.
Trial court vacates the $12 million to Bagwell because the parties settled, but the court refuses to vacate $54
million to the state because there was no such settlement.
b. Criminal contempt penalty may only be imposed if the person was afforded Constitutional protections.
Civil contempt sanctions are designed to compel future compliance with a court order and are
avoidable through obedience. They may be imposed upon notice and an opportunity to be heard.
c. SC finds that the $54 million is a criminal contempt order because, although this order has civil and criminal
characteristics, the union couldnt purge the fine. It was imposed on the parties, so they couldnt avoid the
punishment by doing what the court wanted them to do.
d. Judges have attempted to circumvent the requirement that civil contempt penalties be avoidable by stating
that defendants could avoid the penalties if they obeyed the injunction and published an affirmation of intent
to abide by its terms.
e. Criminal says that the fines are so huge and there is much dependent on the fact that after the fact someone
has to prove that somebody violated the injunction that the $54 milion fine will be considered a criminal
contempt fine and requires jury and due process.
f. So if you deal with a more seriously large civil contempt fine then you have more room to argue that its
really a criminal contempt fine.
C. US v. City of Yonkers (contempt to force a City Council to change votes; fines on individuals = unnecessary)
a. Court ordered City of Yonkers to build 800 units of government housing in white neighborhood, but Yonkers
refused. Instead of meeting to plan how to build the units, the city met to figure out how to pay the escalating
fines ($500/day for individual board members and $100/day for the city doubling everyday until it was
implemented). Appellate Ct affirmed $1 million/day limit. Finally, two board members changed their votes.
b. S Ct said: stayed against individuals unnecessary
D. Walker v. City of Birmingham (cant claim a TRO is invalid afterwards; should have filed opposition papers)
a. Petitioners are black ministers who wanted to march on Birmingham. State Court order a TRO that
prevented them from marching. Petitioners violated the order. Court imposed contempt charge of 5 days in
jail and $50 fine because before they violated the order, they should have challenged it. But US SC says it is
a courts decision to determine the validity of the law and not an individuals. Majority says the injunction
would probably not have been upheld because there was no notice and it prevented freedom of speech.
b. Warren Dissent focused on Constitutional problems of the TRO. Douglas dissent discussed courts not having
jurisdiction if it is an unconstitutional order and it gives too much power to local court decisions.
c. Collateral Bar Rule prevents petitioners from making subsequent arguments against the TRO that
they didnt make prior to the violation of the TRO. This rule makes sure people go through the proper
procedures and rules of the court and dont take the law into their own hands. People are not
constitutionally free to ignore the court because of their impatient commitment to the cause. Cant attack an
order after you violate it, must attack its validity before you violate it. Only by following the process of law
will you give any meaning to the rights you are seeking to enforce. MLK responds it is difficult to wait when
you have seen what he has seen.
d. Injunction may be disobeyed where:
a. It is transparently invalid
b. Court lacks jurisdiction over the person

R. Collecting Money Judgments


A. Money Judgments
- A money judgment is a declaration of rights that says who owes what to whom, it is NOT an obligation that the
defendant must pay the plaintiff.
59

- Judgment Creditor is the successful plaintiff who gets a right to collect on the judgment
- Judgment Debtor is the defendant
A. Judgment Enforcement Steps:
1. Find the judgment debtors assets
a. Asset search before you sue/investigation
b. Depositions (judgment debtor examination) of Judgment Debtor and Third Parties
c. Appoint a receiver to take over business
2. Abstract of Judgment i.e. lien on real property. This is a piece of paper that prevents people from selling it
(freezes it) because if they do, you get the money. Record it at the county recorders office.
3. Writ of Execution - Piece of paper you get from the court that says the bank must pay you the money in
defendants bank account. Also, includes sheriff instructions.
4. Garnishment of Judgment Debtors Wages (Federal state and CA statute limits to 25% of monthly income)
5. Personal Property get a writ of execution to take furniture, computers, car, etc.
6. Fight off Judgment Debtors claims of exemption e.g. exemption of personal jewelry of up to 5000, food,
retirement, pension, social security, homestead exemption (equity in house).
7. Fight off Government and other judgment creditors
8. Fight off Bankruptcy Filing
B. Coercive Collection of Money Writ of Execution and Garnishment
Execution is used to get tangible property and garnishment is used to get simple money debts owed to the judgment
creditor.
C. Execution Court issues a writ, the writ is delivered to the sheriff, he levies on the debtors property, and eventually
sells the property on which he levied. Proceeds of the sale go first to pay the sheriff and then to pay off liens on the
property.
1. Credit Bureau v. Moninger Credit Bureau obtained a money judgment against Moninger. Moninger
subsequently borrowed more money from the Broken Bow Bank, but failed to sign a security agreement giving
bank an interest in his truck. Credit bureau obtained a writ of execution and got the sheriff to execute it. Sheriff
held the door handle of Moningers truck and said, I execute the writ for the state and county. Sheriff did not
take physical possession of the truck. Then Moninger signed the security agreement with the bank. Sheriff then
seized the truck and sold it. Bank argued that the Sheriffs execution of the Credit Bureaus writ was improper
and their security agreement should take priority. Court in NB, a writ is served by expressly asserting
dominion over the vehicle is enough. Therefore, the Credit Bureaus writ takes priority. In CA (the
majority rule), when you execute a writ, you must seize the property and take it away. Most states require
the sheriff to assert effective control over the property.
D. Garnishment Independent action against a third party who owes money to the judgment debtor. Common
garnishees are banks and employers; bank accounts and wages are attractive sources of payment. Garnishee can
defend on the ground that it doesnt owe the judgment debtor, but it cannot question the underlying judgment. If the
garnishee answers that it does not owe the judgment debtor, judgment is entered against the garnishee, it pays the
judgment creditor, and its liability to the judgment debtor is discharged.
12. Dixie National Bank v. Chase - Garnishing wages for child support. If you have to pay child support or alimony, you
better pay it, because if you spend it on something else the judge will trace the money and get the money to your wife.
Garnishees (banks or employers) can incur double liability or worse if they ignore or mishandle the
garnishment. In many states, a garnishee who fails to answer at all becomes liable for the entire judgment
against the judgment debtor. Here the bank was careless in following its own procedures with respect to accounts in
trade names, bank just looked at Jimmy Gore account and not also Jim Gore. The creditor then got another writ
of garnishment served for Jim or Jimmy Gore, but in the meantime, Jim Gore had taken his money out of that
account. Judgment entered against the bank for the entire amount fo the judgment debtors debt to the judgment
creditor.
B. Exemptions to Enforcing Money Judgments Judgment debtors can keep their exempt property even if the judgment
remains unpaid. All states exempt some property, but there is enormous variation. E.g. Florida has a very high homestead
exemption. Where there are generous exemptions, execution is ineffective against ordinary citizens.
1. CA Exemptions (Handout):
a. 704.010 motor vehicle up to $1900 (if you sell it, you must give the first $1900 to the debtor).
b. 704.020 a household furnishings, appliances, provisions, wearing apparel, personal effects, but only if
theyre ordinarily and reasonably necessary to live on. i.e. wine cooler, big screen television, are not
ordinary and reasonable.
60

c.
d.
e.
f.

704.030 material about to be used to repair your home up to 2000.


704.040 jewelry, heirlooms, and works of art up to 5000.
704.050 health aids, prosthetics (no dollar limit),
704.060 personal property thats necessary and used in your trade or business up to 5000. e.g. carpenters
tools, but not tools for a lawyer.
g. 704.070 paid earnings and wage garnishment: b 2 75% of the paid earnings are exempt, but under
subsection 1 all the earnings are exempt if they are subject to an earnings assignment order for support (child
support payments).
h. 704.080 social security benefits exempt up to 1000.
i. 704.115 private retirement plans
j. 704.120 unemployment insurance
k. 704.140 personal injury causes of action. i.e. if you get hit by a truck and get 100K, that is exempt because
that money is there to make you whole.
l. Other Provisions Wrongful death, workers compensation, relocation benefits for displacement, judgment
debtor and spouses cemetery plot.
m. 704.730 Homestead Exemption:
i. Up to 50K if you live alone
ii. Up to 75K if you live with someone in community property home
iii. Up to 125000 if you live with spouse in community property home and you or your spouse are
disabled or over 65 years old
n. 706.010 Wage Garnishment: you can only take out a limited amount so people dont starve. Pg 865
Judgment creditor can only gtake maximum of 25% of salary or amount by which his pay exceeds 30 times
the minimum hourly rate.
II.
PRE-JUDGMENT REMEDIES Attachment, TOP & Receivership
A. Attachment Attachment is a levy or garnishment before judgment that ties up assets.
1. Primary purpose is to preserve access to assets because they may be about to disappear.
2. It is a piece of paper that gives you a right of attachment order and a writ order. You get copies of them and you go to
the bank and you go to the defendant and give them copies. This doesnt get you money, but ties up the money. If the
defendant or bank violates the order after this, the defendant will go to court and get a contempt citation.
3. Attachment is similar to replevin, except that, with replevin, plaintiff has a pre-existing claim to the collateral (i.e.
security interest and entitlement). Attachment is more invasive because it can reach any nonexempt assets defendant
owns without an adjudicated claim.
1. City of New York v. Citisource - NY City was seeking damages against two civil defendants who had previously been
convicted of bribing a city official to get a $22 million contract. Before the city sued, they first had to figure out how much
money the defendants had at their disposal. Once you found money in the bank accounts, you want to make sure that the
money stays in the accounts and defendants dont try to hide the money. City asked the court to order the bank not to release
the money to the defendants and issue an order preventing access to the money for the defendants. Court provisionally granted
a TRO but then vacated the attachment. City appealed. Showing in NY to get the attachment is different than in CA.
In NY, it is difficult to get an attachment, in CA it is easier, in Maine it is easy.
In NY, to get an attachment, plaintiff must show:
1) Intent to defraud or frustrate the enforcement of a judgment
2) Defendant assigned, disposed of, encumbered, or removed asset from the state or is ABOUT TO DO SO
3) Probability of success on the merits (maybe 75% certain)
4) Amount demanded is greater than all known counter claims (because why should the court waste its time if the defendants
claims exceed the plaintiffs).
IN CA, TO GET AN ATTACHMENT, PLAINTIFF MUST SHOW (POS-A-TRACTION CASE PG 4):
1) Claim upon which the attachment is based is one where an attachment can be issued. Such as a claim for a specific
amount of money that is readily ascertainable or fixed.
2) Probable Success on the Merits more likely than not it will obtain a judgment against the defendant.
3) Proper Purpose for the Attachment cannot be sought for a purpose other than the recovery. i.e. you cant get an
attachment just because you hate the other person.
4) Amount to be secured by the attachment is greater than zero. Doesnt include all counterclaims, just good counterclaims.
However, practically, judges typically consider all counterclaims.
In Maine, to get an attachment, plaintiff must show:
1) More likely than not, plaintiff will win (and recover judgment)
***Dont have to show intent to defraud, or attempt to remove assets, etc.
61

In this case, there was a fear that the defendants were trying to defraud because they called the bank asking how they could
withdraw their money. Defendants said that they were merely inquiring about the procedure of how to withdraw from the
account. Questions was whether the plaintiffs proved that the defendants were about to withdraw. Court says that this shows
they were about to withdraw and orders the attachment.
C. Receivership Receivership is a very sophisticated form of attachment where someone is appointed to manage/run a
business. Prejudgment receiver manages the property for the court, preserving it for delivery to the true owner once the
court decides who the true owner is. Ideally they are business people, but most often judges will just appoint receivers who
are their friends. They charge a lot of money (hundreds or thousands/hour) and can cost more than the business brings in.
1.

Erikson Construction v. Congress-Kenilworth Water slide case. Erikson was hired to build the water slide for
Thunder Mountain Rapids Corp., who owns Congress-Kenilworth. C-K only partially paid Erikson for the work. CK claimed it had no assets, while Thunder Mountain Corp collected hundreds of thousands from the park. When the
court appoints a receiver, it deprives legal ownership, so it should exercise this power with care.
To get a receivership, plaintiff must show:
1. A clear right to the property itself or has some lien upon it (OR that the property constitutes a special fund
that he has a right to, such as the money C-K made from the water park in the summer)
2. Defendant obtained the property by fraud or Property, or the income arising from it, is in danger of loss
from neglect, waste, misconduct, or insolvency
Court said there was a special fund (income from the park) that Erikson had a right to because it had a promissory
note, but found there was not sufficient evidence that Erikson was, or would be, deprived of any proceeds from the
management agreement between Thunder Mountain and C-K. Speculative fear is not enough for the court to appoint
receivership. Court will give you a receivership, but plaintiff must do a better job in showing they are entitled to it.

S. Attorneys Fees
A. Notes on the American Rule (fear of discouraging litigation)
a. American rule- each party to a lawsuit ordinarily bears its own attys fees; recovery of attys fees viewed as an
ancillary matter, separate from the merits
b. Since litigation uncertain, one should not be penalized for defending or prosecuting a lawsuit.
i. England- if you lose, you pay other sides atty fees. Theory- stops frivolous lawsuits.
c. Policy:
1. Downside of the American Rule recovery is never complete because have to get money
for attorney fees.
2. Upside of American Rule preserves access to the courts because if the always was
responsible for paying the attorneys fees of victorious-s, then the would be disinclined
to file suit.
d. Notes on Exceptions to the American Rule
1. Statutory exceptions
2. K exception- K that says you get atty fees if win
a. CA statute provides that if any contract provides fees for one side, then either
party can recover fees if it prevails. Cal Civ Code 1717
3. Family Law
4. Collateral Action
5. Bad-Faith
6. Private AG
7. Frivolous litigation
8. Contempt of court exception- (Compensatory civilyou violated order and caused harm,
now you must pay attys fees)
9. Common Fund
10. Tort of another doctrine- when Ds wrong involves P in collateral litigation, P can recover
the expenses of that litigation, incl attys fees, as consequential damages
a. Where broker makes a misrepresentation to an insured and then the insured sues
the insurer, the insured can collect attys fees from broker.
B. City of Riverside v. Rivera (awarding reasonable attorneys fees = Lodestar: Number of Hours x Reasonable Rate)
a. Facts: Police broke up a party at Rivera house. Arrested people using unnecessary physical force. DC
awarded 33K in compensatory damages total and 13K for state and 20 for Federal Claims. Court awarded
250K in attorneys fees because section 1988 gave attorneys fees to prevailing parties. Defendant contested
62

the 250K in attorneys fees. Defendants argue that the amount is unrelated to the amount of compensatory
award.
b. SC upheld the award because attorneys fees dont have to be related to compensatory damages. There
is nothing in section 1988 or the courts history that says that is the way it should be done. SC says the
proper test is the Lodestar: Number of hours reasonably expended x Reasonable Hourly Rate.
Number of hours must be proved, if cant be proved, then dont get them. Hourly rate determined by
the market rate for attorneys with comparable skills and experience, even if the lawyers are public interest
and work at below-market rates. Here, the attorneys charged roughly $400/hour and spent 2000 hours.
Dissent said $400/hour is not a reasonable rate for lawyers out of school for 2 years.
a. Section 1988 specifically entitles plaintiffs to attorneys fees even where there are no monetary damages, so even
if all they got was an injunction, they would still get attorneys fees.
b. Pg 921 Where fees may be awarded to a prevailing party, if the plaintiffs prevail on most of their claims, then it
maybe ok to award all fees, but where only prevailed on some claims, then that maybe excessive. Unrelated
Claims should be treated as separate lawsuits, so the fees from unsuccessful unrelated claims should not count in
the attorneys fees number of hours analysis.
c. Pg 921 Plaintiff sues for an injunction and the defendant voluntarily stops. In Federal Court, there is no
enforceable judgment on the merits and the plaintiff is not a prevailing party. Therefore, the attorneys cannot get
fees. In CA state court, attorneys fees will be awarded where the plaintiff has been the catalyst in bringing
about the relief sought by the litigation. But where the plaintiff has just been the activator it is to be decided by
the CA SC soon.
d. Fees and proportionality to recovery? not in this case (huge difference in compensatory v. attorneys fees)
a. Do nominal damages support fees? Should they?
e. Rule there is NO per se rule that attorneys fees in excess of damages are unreasonable.
a. Relevant (Johnson) factors in determining whether or not attorneys fees are
reasonable
i. Time and labor required
ii. Novelty and difficulty of the questions
iii. The skill requisite to perform the legal service properly
iv. The preclusion of employment by the attorney due to acceptance of the
case
v. The customary fee
vi. Whether the fee is fixed or contingent
vii. Time limitations imposed by the client or the circumstances
viii. The amount involved and the results obtained
ix. The experience, reputation, and ability of the attorneys
x. The undesirability of the case
xi. The nature and length of the professional relationship with the client
xii. Awards in similar cases.
b. Multiplier these factors will be used to determine whether or not the attorneys
fees should be multiplied because of the nature of the case.
C. Common Fund Cases
i. Attorneys fees in common fund cases should be market price In re Cabletron
1. Facts litigation against Cabletron, a securities fraud class action. Settlement = $10.5
million, Plaintiff motions for fees/costs (3.15 million fees plus 900k costs)
2. Plaintiffs motion in support:
a. Big Risk
b. Most contingency fees are 1/3 (as in this case)
c. Skill/Experience of Attorneys
d. This settlement is GREATER than most other similar cases
e. Public Interest in pursuing these claims (and Lodestar would have been $8 million
Riverside)
3. Rule when deciding on the appropriate levels in common-fund cases, courts must do
their best to award counsel the market price for legal services in light of the risk of
nonpayment and the normal rate of compensation in the market at the time.
a. The POF method is preferred in common fund cases because "it allows courts to
award fees from the fund 'in a manner that rewards counsel for success and
penalizes it for failure.'"

63

b.

ii.

A lodestar cross-check may also be useful; however, it is unclear to this Court


where the precise lines of "reasonableness" would be drawn if the lodestar crosscheck was mandatory
c. HOLDING: Having considered at length the import of this data, the Court
concludes that a reasonable percentage in this case shall be calculated by
averaging the 17 percent figure from the market-based cases with the 26 percent
figure derived from the various studies. Therefore, this Court finds that, in light of
all the circumstances, a fee award on a POF basis of 21.5 percent, or $ 2,257,500
is reasonable. (Court finds a percentage that reflects market price)
4. How do you determine market price?
a. Look at actual agreement
b. Legal fees date from similar suits
c. Risk of nonpayment of fees increases the rate to recoup those funds
In these common fund case, the fee is taken out of what defendant owes the class on the merits.

T. Equitable Defenses I
U. Equitable Defenses II

64

Vous aimerez peut-être aussi